[d | an-b-bro-fr-gf-hr-l-m-maid-med-mi-mu-ne-o-old_o-p-ph-r-s-sci-sp-t-tran-tv-w-x | bg-vg | au-mo-tr | a-aa-abe-azu-c-dn-fi-hau-jp-ls-ma-me-rm-sos-tan-to-vn | misc-tenma-vndev | dev-stat]
[Burichan] [Futaba] [Gurochan] [Tomorrow] [Архив-Каталог-RSS] [Главная]

Файл: cirnok200.png -(122 KB, 1172x613, cirnok200.png)
122 No.4946080  

Про физику говорить можно тоже, но только совсем чуть-чуть!

>> No.4946088  

Что будет если всеопознающий всёопознаватель столкнётся с неопознаваемой неопознанностью?

>> No.4946093  
Файл: cirnok400.jpeg -(8 KB, 300x168, cirnok400.jpeg)
8

>>4946088
Будет ужасно!

>> No.4946097  

Поясните номер треда. Вике пишет, что там все Z_2 по диагонали. Как это понять, почему так?

>> No.4946099  

>>4946080

Я случайно отложил мешок кирпичей от "очевидных" объяснений в бамплитный тред!:
>>4946095

>> No.4946106  
Файл: cirnok500.jpg -(44 KB, 500x500, cirnok500.jpg)
44

>>4946097
Это Freudenthal suspension theorem: \pi_{n+k)(S^n) стабилизируются когда n>=k+2, доказывается очень просто кстати, через выписывание LES по довольно просто устроенному вложению.

>> No.4946115  

Уй точно, это же по диагонали и есть. Мне казалось, что FST про стабилизацию по вертикали. Моэ-моэ-моэ

>> No.4946202  
Файл: cirnok600.jpg -(154 KB, 800x450, cirnok600.jpg)
154

>>4946099
Может чуть позже отвечу подробнее, но постараюсь ещё раз объяснить про 2) но теперь по-другому. То во сколько площадь одной фигуры больше другой не зависит от масштаба в которых эти фигуры нарисованы, потому что если площадь одной фигуры A а площадь второй фигуры B и мы увеличили масштаб в k раз, то площадь первой фигуры будет k^2 A а площадь второй k^2 B но их отношение (k^2 A)/(k^2 B) = A/B останется тем же. Поэтому у любых двух подобных фигур (не обязательно прямоугольных треугольников, не обязательно треугольников вообще) отношение квадрата любого линейного размера (скажем, периметра или диаметра) и площади будет одинаково, вне зависимости от того во сколько раз одна из этих подобных фигур больше другой.

>> No.4946612  
Файл: 0B44 омайгод.png -(873 KB, 1360x544, 0B44 омайгод.png)
873

>>4946202

> Может чуть позже отвечу подробнее, но постараюсь ещё раз объяснить про 2) но теперь по-другому. То во сколько площадь одной фигуры больше другой не зависит от масштаба в которых эти фигуры нарисованы, потому что если площадь одной фигуры A а площадь второй фигуры B и мы увеличили масштаб в k раз, то площадь первой фигуры будет k^2 A а площадь второй k^2 B но их отношение (k^2 A)/(k^2 B) = A/B останется тем же. Поэтому у любых двух подобных фигур (не обязательно прямоугольных треугольников, не обязательно треугольников вообще) отношение квадрата любого линейного размера (скажем, периметра или диаметра) и площади будет одинаково, вне зависимости от того во сколько раз одна из этих подобных фигур больше другой.

Я словно читаю инопланетную газету или отчёт с именами (позывными) секретных агентов. Каждый из них мне очень слабо знаком. Мне нужно составить досье на каждого! И мозг отчаянно пытается связать ниточки с помощью дедукции и размытого (для него) контекста:

  • Агент "квадрат/отношение квадратов" судя по отчёту является каким-то самостоятельным мемом, так как упоминается без упоминания Пифагора. Но мне надо выяснить его отношения с Пифагором, так что от этого факта мне пока не холодно не жарко... (1)
  • Снова pop up новый (для меня) агент, по имени "Масштаб/ы". У меня есть слабая теория, что "Масштабы" и "фигуры на сторонах" это одна и та же личность somehow... (это создало бы ниточку к Пифагору)(2)
  • Я начинаю сомневаться, приведут ли разборки с этой новой кликой бандитов к действительно новой информации о синдикате Пифагора (моей первейшей реакцией на твой первый ответ была мысль, что ты неправильно понял вопрос "откуда квадраты", однако потом решил что это недостаточно ясное дело, а теперь совсем неясное: может ты объясняешь что вообще не нужно объяснять, может нет. может пытаешься оправдать действие Эйнштейна, когда к нему вопросов нет!)
  • 100-ое прочтение документа выявило содержание утверждения, что иерархия в секте "Подобные" не зависит от количества превосходства в размере, что очевидно противоречит информации о секте "Пифагоровы Штаны", в которой отношение превосходства строго контролируется. Теперь надо интерферировать и проверять кучи гипотез о взаимоотношении этих двух, уже очевидно различных, структур "Подобные" и "Filthy Пифагор"... они торгуют местами-рангами в своей пирамиде (похожий случай был в Флатленде в 1884 г.), чтобы вступить в оборот надо только стать "квадратом" (сотником/десятником), но пока не очень ясно как течёт эта "валюта"... когда я спрашивал "откуда квадраты", то даже не ожидал что всплывёт столько нестиранных портянок (вопрос засиял совсем новыми красками)... возможно, за многими преступлениями на самом деле скрывалась теневая фигура "Площадь/Безразмерье" (кажется, что-то похожее было в формулах круга... но тут же нет: не все "отношения" в сети посредников обязаны быть "квадратами")(3)

Каждый шаг после (1) требовал ощутимого мысленного усилия, был "потным"! Но именно это мне приходиться делать для попытки (неверной!) понять смысл хотя бы одной фразы объяснения! И это не шутка! В попытках проверить (3) дошёл до уравнений парабол и эллипсов, чтобы хоть где-то встретить соотношения, требующие возведение величин в квадраты, хотя это должно быть явно куда-то не туда...

https://en.wikipedia.org/wiki/Seven_Bridges_of_K%C3%B6nigsberg#Euler's_analysis
Хотел сказать, что 7 мостов это ещё один пример непонятного (для меня) доказательства, но теперь вроде пофигу стало!

Известно ли что-нибудь интересное о задаче про 8 ферзей?

Фейнман говорил, что математика позволяет связывать разные факты. Сколько математики надо для связывания разных фактов о скоростях? Постоянство скорости света, максимальность, одностороннесть/двустороннесть, связь скорости и массы... Ещё может не понимаю самого обычного сложения скоростей.

>> No.4946613  

>>4946612
Отошло!:

Непонимание вызывало то, что обоснование слишком тривиальное (причём тут графы? это просто какой-то факт о количествах) и что "граф" это какое-то размытое понятие (из графа можно вынуть другой граф, можно одной линией соединить два совершенно не связанных по сути графа, можно пририсовать к любому графу хвостик) — в итоге опять непонятно, о чём именно утверждение

С площадью треугольника немного похожая ситуация, но там всё ещё достаточно просто: можешь разбивать любой треугольник на два прямоугольных, можешь не разбивать (можно считать, если хочешь, что дело не в сложной структуре под названием "треугольник", а в свойстве наклонной линии: "треугольник" же представляет собой просто грибницу/петлицу разных сущностей, слитых в одно, и по удачному стечению обстоятельств сохраняющих одно свойство — с графом же не очень понятно, о какой более простой сущности речь; в других понятных доказательствах тоже вроде бы понятные структуры, где список/доска/мельница)

>> No.4946641  
Файл: noncirno.png -(219 KB, 460x460, noncirno.png)
219

>>4946612

>может пытаешься оправдать действие Эйнштейна, когда к нему вопросов нет!

Ну видимо пытаюсь, просто мне показатлось, что его последний пас про то что отношение площади к квадрату гипотенузы у подобных фигур одно и то же может выглядеть непонятным. Хотел показать, что гипотенузы тут не при чём, и тем же самым аргументом что, например, сумма площадей вписанных окружностей двух маленьких треугольников равна площади вписанной окружности большого. Вроде известно что задача "можно ли потавить n ферзей на доске n на n чтобы не били друг друга" NP-полная, то есть иначе кроме как (оптимизированным) перебором решить её нельзя. Ещё мне один раз давали её как на phone interview то ли в гугл то ли в фейсбук как задачу по программированию, во времена когда я ещё программистом был.

>Фейнман говорил, что математика позволяет связывать разные факты. Сколько математики надо для связывания разных фактов о скоростях? Постоянство скорости света, максимальность, одностороннесть/двустороннесть, связь скорости и массы... Ещё может не понимаю самого обычного сложения скоростей.

Для самого обычного сложения скоростей совсем немного, для постоянства скорости света и прочей теории относительности в принципе тоже немного, но наверное лучше с обычной кинематикой сперва разобраться. Со всякими релятивистскими массами/массами покоя и E=mc^2 я сам не разобрался, вроде физики их как-то различают, но я не очень разницу понимаю.

>> No.4946642  

>>4946088
Ты же опознал ее уже, как неопознаваемую неопознанность. Переформулируй вопрос.

>> No.4946644  
Файл: a.jpg -(8 KB, 191x264, a.jpg)
8

>>4946641

> Со всякими релятивистскими массами/массами покоя и E=mc^2 я сам не разобрался, вроде физики их как-то различают, но я не очень разницу понимаю.
> релятивистскими массами

Забудь, такой массы нету. Есть только масса покоя.

> E=mc^2

Это на самом деле очень легко выводится (для простоты изложения поставим с=1):

  1. Из-за постоянства скорости света можно вывести инвариант: ds^2 = -dt^2 + dx^2, где ds^2 является инвариантным "отрезком" (который иногда отрицателен, ага), dt -- отрезок по времени, dx -- отрезок по пространству. Инвариантность нужно полагать для смены системы координат.
  2. Таким же образом можно вывести инвариант для энергии: dm^2 = -dp^2 + dE^2, где p -- это момент инерции. То есть инвариантный отрезок превращается в инвариантную массу (т.е. массу покоя; очевидно что инвариантный отрезок это "отрезок покоя").
  3. Ставим момент инерции dp=0 и добавляем "с" назад чтобы единицы измерения работали всегда: E=mc.
>> No.4946646  

>>4946088
Этого априори не может произойти по следующим причинам:

  1. Он не является всезнающим реальности, если столкнётся с этим.
  2. Либо он является частью той самой реальности, которую невозможно понять. Т.е. живёт постоянно в ней, его либо не существует, либо он и есть эта реальность
>> No.4946729  
Файл: cirnok470.png -(217 KB, 530x200, cirnok470.png)
217

>>4946644
Не, ну это совсем интеллектуально нечестный вывод, потому что формула в пункте 2 это обобщение E=mc^2, и говорить "можно вывести" не очень красиво, вообще даже для того чтобы переопределить правильным образом что такое E и что такое p уже нужны некоторые аргументы. Я аргумент Эйнштейна про E=mc^2/sqrt(...) примерно знаю (предположить что энергия зависит от скорости и массы, и проанализировать в двух разных фреймах ситуацию когда большое тело разделяется на два маленьких и летят в разные стороны, из этого получится функциональное уравнение на функцию E(m,v) которое потом можно будет решить чисто алгебраически) но я бы сказал что не очень подлежащую философию вокруг него понимаю, ну например мне вообще не очевидно что энергия должна зависеть только от массы и скорости. А про "релятивистскую массу" и "массу покоя" я помнил, что там как-то традиция терминологии сменилась, в любом случае как бы величину "m" в E=mc^2/sqrt(...) не называть она не полный аналог той массы, которая в обычной механике. На самом деле мне просто тип мышления физиков и их аргументы в духе "элементов Евклида" (когда самоочевидным предполагаются некоторые принципы) не очень нравятся и вряд ли когда-нибудь понравятся.

>> No.4946866  

>>4946729

Хорошо. Формула 2, как и 1, выводится из симметрий.

В до-релятивистской физике, симметрии это перемещение в пространстве, перемещение во времени, и вращение (все можно представить в виде 4x4 матриц). В релятивистской физике перемещения в пространстве и времени объединяются в одну общую симметрию (хотя это происходит уже в классическом электромагнетизме).

Далее, используем эту новую симметрию на пространственных точках и выводим формулу 1. Или используем ее в фазовом пространстве (пространстве скоростей) и выводим формулу 2. Тут конечно каким-то магическим образом момент инерции превращается в энергию и обратно. На самом деле, просто исторически сложилось, что величина, сохраняемая при перемещении во времени называется энергией, а при перемещении в пространстве -- моментом инерции. Очевидно, что в теории относительности ввиду объединения симметрий, эти две вещи тоже должны объединятся.

> аргумент Эйнштейна

Ух ты. Первый раз вижу. Спасибо.

> подлежащую философию

Обычно просто используется интуиция.

> как бы величину "m" в E=mc^2/sqrt(...) не называть она не полный аналог той массы, которая в обычной механике

Почему это?

>> No.4946924  
Файл: 0B45 Pythagoras_by_pentagons.png -(51 KB, 815x1024, 0B45 Pythagoras_by_pentagons.png)
51

>>4946641

> Ну видимо пытаюсь, просто мне показатлось, что его последний пас про то что отношение площади к квадрату гипотенузы у подобных фигур одно и то же может выглядеть непонятным.

Я человек совсем простой!: я просто верю, что верное для треугольников верно и для квадратов. А что такое "отношение площади к квадрату" (безумный какой-то параметр вообще! что он может значить?) даже не представляю! Чей площади? Возможно из-за этой простой непонятки ничего и не понимаю. Почему единственная сущность "штука на стороне OR квадрат/шмадрат стороны" стала вдруг двумя сущностями с каким-то отношением к чему-то (квадрат же и есть площадь, вы чего)?

На самом деле прости меня: я просто не дочитал до последнего предложения на википедии, думал ты какую-то свою информацию мне даёшь (не понимал, в какие дебри масштабов и единиц измерения мы улетели). Перечитываю то последнее предложение третий раз, что-то брезжит, но всё равно тяжело... Там 3 разных гипотенузы, 3 не квадрата, 3 разных квадрата, площадь (по сути тоже не одна)... и отношение чего-то из этого к чему-то из этого сохраняется для того-то из этого и сякого из этого, это просто hopeless (понять это всё равно что случайно собрать часы, бросив детальки на землю. Разве можно считать это понятным, так обращаться с людьми?)

> Хотел показать, что гипотенузы тут не при чём, и тем же самым аргументом что, например, сумма площадей вписанных окружностей двух маленьких треугольников равна площади вписанной окружности большого.

Понимаю/верю в это, как в ясно выраженный для моего ума факт! Но где-то мне казалось, что ты говоришь уже вообще не про наш главный (пифагоровый) треугольник и что-то "не зависит от того, во сколько раз оно больше другого" (10 интерпретаций у этой фразы может быть и выбрал видимо не ту)

> Вроде известно что задача "можно ли потавить n ферзей на доске n на n чтобы не били друг друга" NP-полная, то есть иначе кроме как (оптимизированным) перебором решить её нельзя. Ещё мне один раз давали её как на phone interview то ли в гугл то ли в фейсбук как задачу по программированию, во времена когда я ещё программистом был.

Раньше на википедии был какой-то странный алгоритм, который делил какие-то числа, какие-то остатки брал... может быть он проверял, можно ли решить доску с такими-то параметрами "лесенкой" (Г-ходами коня).

> Для самого обычного сложения скоростей совсем немного, для постоянства скорости света и прочей теории относительности в принципе тоже немного, но наверное лучше с обычной кинематикой сперва разобраться.

Просто у меня непонятка о разнице разных схожих идей. Как понял:
1) Если пространство абсолютно, то о сложении скоростей можно почти не говорить кроме как для объяснения факта того, что люди не падают с поездов? Если ты двигаешься в сторону чего-то, ты просто становишься ближе к этому, ничья скорость не меняется?
2) Если пространтсво не абсолютно, то ты не можешь знать, изменил ли ты положение, поэтому тебе остаётся лишь констратировать изменение скорости движущегося объекта или своего приближения к нему? Хотя как это измеряется тогда?
3) Если время не абсолютно, то ты не можешь знать даже изменение скорости, то что разные вещи дошли до тебя в разные моменты времени можно списать на то что на самом деле это было одновременно, просто одновременность относительна?
Где тут (или совсем не тут) ещё эффекты замедления времени и проч.?

Просто у меня такое непонимание: если ты стоишь перед светом, а потом идёшь на свет, то какой эффект останавливает/избегает сложение скоростей? Как не спутать/спутать абсолютное пространство с абсолютным расстоянием или чем-то ещё, скалярами всякими, световыми зайчиками (где скорость, где не скорость)?

https://physics.stackexchange.com/questions/200982/special-relativity-two-beams-of-light-in-opposite-direction
https://www.physicsforums.com/threads/what-is-happening-when-travelling-towards-light.333774/#post-2329462

Ты-то "не разобрался" в чём-то умном, в отличии от!

Или вот ещё что в статье не находил:
https://en.wikipedia.org/wiki/Pythagorean_theorem#Similar_figures_on_the_three_sides

Увы, особо понятнее не стало! Нужно создавать в уме какие-то зацепки, всё заново делать...

>> No.4946928  
Файл: cirnok480.jpeg -(60 KB, 529x600, cirnok480.jpeg)
60

>>4946866
Я если честно немного потерял нить уже, я не спорил с тем что если правильным образом определить 4-импульс, то он будет конечно же сохранятся относительно преобразований и вывод любых соотношений будет тривиальным.

>Почему это?

Ну потому что в ньютоновской механике есть закон сохранения массы, инвариантность массы и аддитивность и в Лоренцевой нельзя ввести концепт массы который удовлетворяет всем трём свойствам, но можно повводить три концепта, каждый из которых удовлетворяет двум из трём, по-моему решить какой из них "фундаментальнее" принципиально нельзя.

>> No.4946961  

>>4946928

Ну, как бы ты не определил массу, на малых скоростях она все равно должна быть той же, что и в Ньютоновской механике.

>> No.4946995  

>>4946928
Встречал где-то интересное пояснение понятия массы. Масса - коэффициент при линейном члене в уравнении поля, следовательно соответствует источнику. То есть массовый член, добавленный к безмассовому уравнению поля, создаёт, как пропорциональный полю источник, переотражения. И это даёт в целом локализованное поведение, подобное массивной частице

>> No.4947012  
Файл: cirnok490.png -(148 KB, 490x585, cirnok490.png)
148

>>4946924
Чуть позже отвечу.
>>4946961
Да, конечно.
>>4946995
Да, это стандартный способ восстановить массу имея на руках тензор энергии-импульса, если я правильно понял что ты хотел сказать.

>> No.4947290  
Файл: 0B47 1557649959178154844.jpg -(376 KB, 1600x2133, 0B47 1557649959178154844.jpg)
376

>>4946928

> я не спорил с тем что если правильным образом определить 4-импульс, то он будет конечно же сохранятся относительно преобразований и вывод любых соотношений будет тривиальным.

У меня именно такое чувство от некоторых математических доказательств, добавляющих в доказательство какие-то "специальные" понятия ("базирующиеся" на наших договорённостях)! Типа "антропный принцип": если бы это не работало, то выбрали бы другое (будь античастиц больше частиц, они не звались бы античастицами, будь наша вселенная неприемлемой для жизни, она не была бы нашей). Мне совсем не интересно КТО (подобие/кусочки/диффереанциал; в твоём случае тебе не интересен некий "4-импульс") и даже не интересно КАК (как и тебе не интересны выводы-выкладки, происходящие после правильного определения), мне интересно КАКОЙ (чем особенны именно эти подобие/кусочки/дифф., почему именно в этой Пифагоровой ситуации наши доказательства доказывают то что нам нужно). Мне без мотивации (и качественных "counterfactuals") никуда!

Ну и определения каких-то сущностей вроде 4-импульса типа "площадь делить на квадрат гипотенузы" (?!) могут просто сбить. Мне думаешь очевидно, что площадь надо делить на квадрат линейного размера? Для меня это как класть сосиски, переломленные пополам, в цветочную вазу!

Может быть эта аналогия станет ключом к взаимопониманию (если представим, что математика как физика)?

Просто я же буквально (упрощая!) в такой же ситуации как ты — вот тебе дали детальки (масса/энергия/проч.) и ты не знаешь как их скомбинировать, для тебя это как случайное подбрасывание в надежде, что нечто само соберётся, ты не видишь мотивации вязать одну деталь к другой. Ты можешь пытаться искать причины/обоснования своей неспособности или отличать одну область от другой (математику от физики), но это либо немного лицемерно, либо не отменит факта, что психологически ситуации могут быть идентичны всё равно

По-моему зря ты не припомнил эту тему постов N назад!

>> No.4947301  

>>4947290

> 4-импульс

Как-бы он пришел из электромагнетизма.

>> No.4947331  
Файл: cirnok520.png -(702 KB, 500x600, cirnok520.png)
702

>>4946924
>>4947290
Про сложения скоростей и абсолютность/относительность, это можно объяснять разными способами. Один из способов - это думать что вообще всё относительно, то есть чтобы определить хоть что-нибудь тебе сперва нужно зафиксировать "из каких глаз" ты на это смотришь. Даже чтобы сросить такие элементарные вопросы как "сколько сейчас времени" или "одновременно ли произошли события А и В" нужно сперва определиться из чьих глаз ты смотришь. Посмотри какие-нибудь видео по СТО на ютубе, их вроде много хороших.

На самом деле зря я сказал что в чём-то не разобрался, потому что эту мою фразу трактовали слишком прямо и начали объяснять чуть ли не азбучные вещи, но с другой стороны и обижаться на такое не следует, если кто-то хочет объяснить - то пусть объясняет. Да мне кажется я и так изначально твое непонимание хорошо понимал, может конечно я много о себе думаю. Поэтому и хотел связать этот последний ход чуть ли не с бытовой интуицией. Суть в том что ты делишь две площади "площадь прямоугольного треугольника" и "площадь квадрата нарисованного на гипотенузе", при этом у тебя три копии этой ситуации: большой треугольник и два внутренних маленьких, но ведь то во сколько одна площадь больше другой не зависит от того насколько большая или маленькая картинка в которой обе эти площади нарисованы. На самом деле объяснить ИРЛ такое было бы намного проще, просто что-то нарисовав на доске, потому что это правда аргумент очень близкий к бытовой интуиции и, по-моему, "вскрывающий внутреннюю историю" этой теоремы.

>> No.4947380  

>>4947012
Не, я имел в виду другое. Например, возьмём скалярное поле, если в уравнении есть массовый член, то можно прочитать его как уравнение безмассового поля с источником, пропорциональным полю. И то, что волны перестают распространяться со скоростью света - эффект присутствия такого источника, он как бы отражает волну обратно в каждой точке.

>> No.4947418  
Файл: 0B46 AA A AA AA A AA.png -(513 KB, 1348x505, 0B46 AA A AA AA A AA.png)
513

>>4947331

> Про сложения скоростей и абсолютность/относительность, это можно объяснять разными способами. Один из способов - это думать что вообще всё относительно, то есть чтобы определить хоть что-нибудь тебе сперва нужно зафиксировать "из каких глаз" ты на это смотришь. Даже чтобы сросить такие элементарные вопросы как "сколько сейчас времени" или "одновременно ли произошли события А и В" нужно сперва определиться из чьих глаз ты смотришь. Посмотри какие-нибудь видео по СТО на ютубе, их вроде много хороших.

Одно дело быть моральным релятивистом, и другое дело верить что моральные нормы сжимаются и растягиваются... наверное. То есть бытовая интуиция не помогает понять, как к таким идеям можно придти чисто философски/незнанием!

https://www.youtube.com/watch?v=AInCqm5nCzw
https://www.youtube.com/watch?v=hTxWAQGgeQw

Такие виде? "Сны Эйнштейна" читал! В Braid играл (хотя там, говорят, наоборот относительность не действует!)

Объяснения в духе "просто добавь ещё одну координату" немного раздражают, но может штука, которую собрали в минутной физике, будет понятнее! Раздражают потому, что описывают ЧТО (4 измерения), но не описывают КАКИЕ (во-первых, не понятно отличие от ньютоно-подобного взгляда, во-вторых, не говорит о том, что для особых эффектов в 4-ом измерении нужно выполнить особые условия)

> Да мне кажется я и так изначально твое непонимание хорошо понимал, может конечно я много о себе думаю. Поэтому и хотел связать этот последний ход чуть ли не с бытовой интуицией.

Но это понимание не влияет на твои действия, ты отвечаешь примерно так как если бы тебе был известен лишь факт непонимания (ты выбрал свой специфический опыт как гипотезу непонимания, но для такого выбора тебе в принципе даже не требовались мои слова). Ты продолжаешь использовать какие-то абсолютные универсальные понятия/качества типа "интуитивной понятности". Может ты видишь истинную причину моего непонимания, которая не связана с моими словами. Может мне так кажется из-за того что я ещё не понял твоё объяснение. Может ты обрезаешь не попадающее в твою гипотезу

Мне нужна не столько интуитивная понятность, мне нужно сравнение и исключение вариантов, обзор пространства вариантов, обзор того как могло бы быть, counterFUCtuals (об этом целый текст накатан). Простая идея — если что-то может быть спутано, оно будет спутано, не важно, из-за бытовой ли интуиции (это всё левые стандартные понятия, которые не важны и ничего не объясняют!), из-за какой-то другой привычки (можно забыть/не знать, что температура не аддитивна, а интенсивна), из-за особенности характера/мышления или ещё чего-то или просто потому что, причём это ещё динамично (возможность запутаться связана с тем, сколько опций тебе предлагают, она будет отличаться от ситуации к ситуации: понятность дважды и трижды относительна). Все стандартные объяснения и доказательства основываются на попытке вбить в голову/усилить один-единственный вариант (связать с бытом, сделать 100 иллюстраций и графиков, разбить на дотошные шаги), дать какое-то "абсолютное понимание" какого-то одного-единственного феномена, который задан абсолютно и безотносительно других феноменов: это колосс, пытающийся устоять на острие кнопки, как его не укрепляй — его сдует слабейшим ветерком/выводом из равновесия. И если бедняга ученик не понял, значит надо заново вдалбливать абсолютную задачу этого одного понятия (делается вывод, что колосс недостаточно сильно давит на острие этой иглы, поэтому "не стоит"). Ты вроде этому не внимал, когда речь была о топологических понятиях (думал, что путаница одного понятия с другим означает полное непонимание понятия или что-то типа того/что не достаточно понятна разница между одним понятием и другим — разница понятий тоже не абсолютна), потом где-то думал что мне нужны для понятности какие-то весёлые байки, и ссылка на текст была возможностью вернуться в частности к этой теме — но мы уже не вернулись (вроде тебе не интересно ни обсуждать вопрос понятности (варианты объяснения непонятности), ни обсуждать вопрос того что может быть "историей" доказательства (разные варианты) — а интересно только "сгладить" эти вопросы к уже подуманным мыслям, запирая себя по сути в рамках первой реакции на феномен и первой пришедшей на ум теории)

> Суть в том что ты делишь две площади "площадь прямоугольного треугольника" и "площадь квадрата нарисованного на гипотенузе", при этом у тебя три копии этой ситуации: большой треугольник и два внутренних маленьких, но ведь то во сколько одна площадь больше другой не зависит от того насколько большая или маленькая картинка в которой обе эти площади нарисованы. На самом деле объяснить ИРЛ такое было бы намного проще, просто что-то нарисовав на доске, потому что это правда аргумент очень близкий к бытовой интуиции и, по-моему, "вскрывающий внутреннюю историю" этой теоремы.

Очень хочется понять ради тебя! Вот всё что могу сделать:

Представляю прямоугольный треугольник, который занимает половинку квадрата. Квадраты катетов равны квадрату. Квадрат гипотенузы, значит, должен быть вдвое больше квадрата! Это звучит интересно (но не знаю, зачем это)(A + B = 2A = C = 4 треугольника)
Если на сторонах будут подобные-прямоугольные вместо квадратов, то мы получим A1/2 + B1/2 = A = C = 2 треугольника (буквы это площади!)(кажется уже что-то не то вышло, что должно быть)

Если менять размер всей конструкции, то ничего не будет меняться, раз это пропорции

Если катеты будут разными, то сложно представить что будет, что за величины и пропорции! A^2/AB(1/2) + B^2/AB(1/2) = C^2/AB, 1/2(A/B + B/A) = C^2/AB, A/2B + B/2A = C^2/AB, A/2 + B^2/2A = C^2/A, A^2/2 + B^2/2 = C^2... (испугался, что что-то совсем не то вышло, но я вроде просто 1/2 одну забыл)

Не понимаю, что делал, с какой целью, чего добился и вообще!

> The basic idea behind this generalization is that the area of a plane figure is proportional to the square of any linear dimension, and in particular is proportional to the square of the length of any side. Thus, if similar figures with areas A, B and C are erected on sides with corresponding lengths a, b and c then:

Площадь фигуры пропорциональна любому квадрату, особенно(? что значит особенно?) квадрату любой стороны (даже не уточняют, что за сторона).

Конечно, любое число можно поделить на любое (не понятно (всё ещё), о пропорциональности чего чему говрится)... и наверное фигуры пропорциональны, это мы и должны доказать (или что пропорциональность именно квадратное? или что нужно именно складывать?)

Ну какая интуиция может быть у меня о результате деления чего-то на что-то? Можно вообще не использовать понятие "деления", как засланный спец. термин? Ведь это, как я понял, просто числовое выражение какой-то идеи

> но ведь то во сколько одна площадь больше другой не зависит от того насколько большая или маленькая картинка в которой обе эти площади нарисованы.

Если понимаю правильно (такие объяснения заставляю спрашивать реальность), это самая банальная шняга, другое дело что совсем не понятно что ей доказано? Где размер, где форма, где связь чего с чем...

Эти "односложные положняки" только множат сущности и запутывают!.. типа "кирпич плывёт (простите, тонет!). А заяц прыгает. А попугай говорит..." что почём, просто мозг взрывается! Зачем плывёт? Куда прыгает? Что говорит, кому?

"То что кирпич тонет это интуитивно понятно?" может быть понятно, может быть нет, без контекста вообще не понимаю зачем и о чём мы говорим, на односложные утверждения мне пофигу с высокой колокольни!

> Because the ratio of the area of a right triangle to the square of its hypotenuse is the same for similar triangles, the relationship between the areas of the three triangles holds for the squares of the sides of the large triangle as well.

Я не понимаю, почему because, в первой части ничего не говорится о компонентах второй (это спрятано в формулах? надо сказать без формул)

>> No.4947422  

>>4947331

> Because the ratio of the area of a right triangle to the square of its hypotenuse is the same for similar triangles, the relationship between the areas of the three triangles holds for the squares of the sides of the large triangle as well.
> при этом у тебя три копии этой ситуации: большой треугольник и два внутренних маленьких, но ведь то во сколько одна площадь больше другой не зависит от того насколько большая или маленькая картинка в которой обе эти площади нарисованы.

Кажется понял что-то, прорвалось! Но всё ещё не до конца: пропорции-то разные и речь даже о разных частях площади большого треугольника, где мы что соединили? Иначе же у нас получится что все стороны равны друг другу (или подобная дичь: не ясно ни откуда квадрат, ни откуда сложение) или утверждение про площадь треугольника, а не стороны... Неужели нельзя прямее сказать?

https://www.youtube.com/watch?v=vOOHIpxy9yM
https://www.youtube.com/watch?v=FhvAE1--DEo

>> No.4948071  

>>4947331

> На самом деле зря я сказал что в чём-то не разобрался, потому что эту мою фразу трактовали слишком прямо и начали объяснять чуть ли не азбучные вещи, но с другой стороны и обижаться на такое не следует, если кто-то хочет объяснить - то пусть объясняет.

Ведёшь какой-то рассказ, с A и B и C, связанными между собой, а отвечают на какую вещь отдельно?

Хочешь затронуть какой-то нюанс A, а тебе объясняют A само по себе?

Но чем твоя собственная реакция порой отличается от этого? Не замечал никогда разрывающих перескоков в собственных постах (сам особо не замечал, если что)? Ну или как бывало с некоторыми ответами задающим здесь вопросы (не знаю, что там на самом деле происходило, бывало ты менял ответ или даже в чём-то менял мнение 1 раз, про какие-то редуксы/редукции/индукции)

> Because the ratio of the area of a right triangle to the square of its hypotenuse is the same for similar triangles, the relationship between the areas of the three triangles holds for the squares of the sides of the large triangle as well.
> при этом у тебя три копии этой ситуации: большой треугольник и два внутренних маленьких, но ведь то во сколько одна площадь больше другой не зависит от того насколько большая или маленькая картинка в которой обе эти площади нарисованы.

Неужели для понятности просто нельзя хотя бы полностью проговорить логическое следствие?

Пока не представляю даже какая вообще логика (даже асбурдная) могла бы дополнить это предложение: разве что если бы деление любой площади на площадь треугольника давало одно и то же число... Может дело в том что площадь на сторонах превращается в саму площадь треугольника? Но всё равно не понимаю

Мне кажется, много чего можно сделать и без всяких бытовых интуиций!

>> No.4949337  

>>4946080

> Про физику говорить можно тоже

Можно ли уронить электрон с одной из орбиталей на ядро, подействовав на атом электромагнитным полем или ударив подходящей частицей с нужным импульсом?

>> No.4949339  
Файл: Pixiv 2552299 21794427.jpg -(656 KB, 1600x1200, Pixiv 2552299 21794427.jpg)
656

>>4949337
Собственно, вопрос «Какого хрена электрон не падает на ядро?» вытек в квантовую физику.

>> No.4953580  
Файл: 71093851_p0.jpg -(491 KB, 759x1000, 71093851_p0.jpg)
491

Пусть с потолка взято утверждение:

>существует такое x, что x ≠ 1 и x не является последующим ни для какого натурального числа

Можно ли доказать, что это x заведомо не существует/не натуральное, опираясь только на 5 аксиом Пеано?

>> No.4953600  

>>4949337
https://ru.wikipedia.org/wiki/Электронный_захват
Как-то так?
Как копировать ссылки с википедии шоб русскими буквами было сразу из адресной строки блин?

>> No.4953602  

>>4953600
Поставь пробел после адреса и копируй.

>> No.4953605  
Файл: 7abcd2f147dfd061b5e0a394e1131984 -(842 KB, 1000x1355, 7abcd2f147dfd061b5e0a394e1131984)
842

>>4953600
Переходишь в адресную строку, добавляешь пробел и копипастишь уже с пробелом.Но делать так нястоятельно не рикамендую, можно словить проблемы с кодировками.

>> No.4953616  

>>4949337>>4953600
Там не совсем верно написано. Не знаю как изложить правильно в рамках официально действующей теории. В общем, процесс характерен только для нестабильных элементов, имеющих дисбаланс внутриядерных процессов за счёт избытка протонов. И "захват электрона" я бы назвал скорее не причиной, а следствием распада. Иначе было бы трудно объяснить, почему уравновешивание ядра приводит к его распаду. Хотя, само по себе наличие по близости электронов является подталкивающим фактором. Там об этом частично говорится. Большего сказать боюсь, чтоб не взболтнуть случайно какой крамолы. Полагаю >>4949337 хотел узнать скорее про то, можно ли столкнуть протон с электроном раз они так любят друг-друга. И почему такого не происходит. Можно же электронов натолкать почти без меры. Ответ будет крайне интересным, но не от меня. Я не умею аккуратно в официальную науку. Не хочу чтоб тред снова вспучило.

>> No.4953645  
Файл: cirno.jpg -(8 KB, 199x253, cirno.jpg)
8

Я если что вернулся, в связи с началом учебного года меня снова начали тут мучать, но могу отвечать на ответы. Про столкновение электрона с протоном вряд ли чего умного могу сказать: если cтолкнуть протон с электроном и при этом не произойдёт электронного захвата - не образуется атом водорода, то вроде должно произойти электрон-протонное рассеяние с выделением энергии, либо elastic либо т.н. deep в зависимости от угла столкновения. Про то можно ли как-то соорудить экспериментальную установку обстреливающую атом протонами точно не знаю, но наверное можно.

>>4948071
Мне кажется тут эффективнее будет постараться тебе разобраться самому, я просто уже не совсем помню что я тебе объяснял, в любом случае судя по объемам текста который мы нагенерировали, первоначальное доказательство, видимо, куда проще любых моих попыток объяснения.

Всех с днём сырно!

>> No.4953996  
Файл: 0B51 Treehouse_of_Horror_XIX.jpg -(179 KB, 450x653, 0B51 Treehouse_of_Horror_XIX.jpg)
179

Я решил, что пора уходить, раз я неспособен здесь ничему научиться! (И не могу повлиять на стиль обучения)
>>4953645

Могу только попробовать несколько конкретных вопросов задать по местам текста, которые были посвящены пониманию:

Обучаем ли такой человек? Вот такой:

> Ударения — возможно, из-за "комбинаторики" у меня проблема с ударениями в словах, знаю как слово правильно читается но не могу связать это с "символом" (мне понятно, что ударение меняется, но совсем непонятно какую букву к этому привязывать: привязка кажется абсолютно искусственной). (...)

(У нас есть много вариантов как прочитать слово (и плохо ещё, что эти варианты не всегда отличаются ударением) и много букв, которые можно выделить (много "символов") — от этого мне тяжело.)

> Счёт — у меня проблемы даже с базовым применением счёта (простого сложения) для узнавания, какое число было в такой-то день недели. Мы считаем сами дни или переходы между ними? (...)

(У нас есть много разных действий, которые можно считать, много разных соответствий между числами и днями (например, номер дня в неделе vs. расстояние между днями) — от этого мне тяжело.)

Понятна ли тебе моя гипотеза, что "понятность", грубо говоря, связана тупо и механически только с (не)возможностью спутать один вариант с другим? Что понятность зависит тупо от комбинаторности? Не от очевидности, наглядности или связи с бытовым опытом или другой "материи понимания"? Ты не веришь в эту версию, ты не хочешь о ней думать или попытаться опробовать её, строя объяснение исходя из неё(?) (первый шаг/первая мысль)

(второй шаг) Понятна ли тебе такая идея: положительное описание ("то-то есть то-то и из этого следует..."), назовём его "внутренней логикой", на практике не всегда эквивалентно негативному ("происходит так, а не эдак: происходит вот это, а не то"), назовём "внешней/глобальной логикой", исключающему альтернативы? Аналогия с "физикой": можно навводить сущностей и наделать каких-то выводов внутри собственной логики, а потом обнаружить что ты сделал 0 предсказаний о реальности

Или могу дать ссылку на новый текст:
http://www.gotai.net/forum/default.aspx?threadid=283802

Вот суть:

  1. Есть "слои объектов" ("уровни реальности"), этими объектами может быть что угодно: это могут быть мыслительные объекты, типа понятий или образов для распознавания... могут быть и физические объекты, наверное... могут быть и какие-то "биологические объекты", наверное (но мне есть что-то сказать только о понятиях и образах! поэтому буду говорить понятия)
  2. У каждого понятия есть бесконечность специализированных копий ("синонимы"), в пределе вообще любое понятие является синонимом любого другого... в каком-то смысле всё абсолютно одинаково, а в каком-то смысле всё абсолютно различно
  3. Чтобы происходил какой-то процесс, например "логический вывод" или "распознавание" (или развитие/эволюция организма или рождение каких-то структур благодаря физическим законам, но тут мне нечего сказать), нужен механизм, который будет какие-то понятия соединять, а какие-то разделять, делать выбор конкретной версии понятия/вносить дисбаланс (например, моральная аксиома или просто жизненный опыт, который придаёт разным понятиям разный "вес"/"важность") — какие-то понятия делать существующими, а какие-то несуществующими
  4. То есть существует некое состояние, в котором (например) "логики не существует" и нельзя сделать никакого выбора — логика не существуют сама по себе и может рождаться только из какого-то дисбаланса
  5. Любой выбор понятия (в пределе) зависит абсолютно от всех других понятий: чтобы выбрать одну версию понятия, тебе должна быть не важна другая версия.
  6. Между уровнями понятий может рождаться циклическая связь. И, возможно, от одного понятия нельзя перейти к любому другому (понятия начинают занимать какие-то отдельные друг от друга "ниши")(это просто об интересных маленьких штучках пишу)
  7. Понятие может обладать свойством "масштабируемости" (это, например, когда одно понятие может применяться к разным уровням других понятий, когда можно свободно перемещаться между уровнями/или когда понятие порождает рекурсивную структуру, которая одинакова куда не сдвигай свой взор/или когда понятие независимо от некоего абсолютного-универсального фундамента/или...), это аналог понятия "симметрии" в физике

Что думаешь? Мог ли бы что-то из математики или физики изложить в подобном стиле?

> Мне кажется тут эффективнее будет постараться тебе разобраться самому

Но ведь нюанс: всё началось с того, что я думал что понимал доказательство.

https://www.youtube.com/watch?v=z9YVoPRdOwc
https://www.youtube.com/watch?v=KQhc4pdan5s
https://youtu.be/bbrfxUiq3GQ?t=111
https://www.youtube.com/watch?v=SKJ_dDQu3pc
https://www.youtube.com/watch?v=NnMIhxWRGNw
https://www.youtube.com/watch?v=EmBjt0b2BKE
https://www.youtube.com/watch?v=itfXCKpmQPs (?)

Там пару раз упоминается "метод размерностей"

>> No.4954039  
Файл: best-30-cirno-fun-on-9gag-52078860.png -(139 KB, 500x750, best-30-cirno-fun-on-9gag-52078860.png)
139

>>4953996
Жаль что решил, ты мне определенно нравился и мне кажется твой стиль мышления довольно сильно похож на мой.

Мне понятны обе твои гипотезы, но кажется что довольно сложно очертить пространство всех возможностей просто речью и сказать какие возможности данным рассуждением отсекаются, обычно в математическом доказательстве говорят "голое рассуждение" оставляя остальную работу по построению пространства всех возможностей и выделению из них тех которые данным логическим шагом отсекаются на проработку изучающему.

Лично я, кстати, не всегда стремлюсь за той моделью понимания, построенном на отсечении вариантов, которое ты описываешь. Ну потому что оно слишком трудоёмкое, а ресурсы памяти конечны, иногда даже дать удачное название или визуальный образ какому-то трюку, который поможет его просто хотя бы запомнить (и, возможно, реконструировать то комбинаторное понимание о котором говоришь ты при случае если тебя попросят этот трюк объяснить) уже большая удача.

Но попробовать можно конечно, чисто в качестве эксперимента хотя бы. Тогда я подумаю какую теорему выбрать и как её можно в таком стиле рассказать.

Кстати, чтобы подчеркнуть насколько мы похожи, я как-то говорил что у меня есть свой "список моделей" и я то что ты описываешь на ГотАИ назвал в этом списке "априорной равноправностью всех позиций", ну только у меня почему-то не хватало энергии это настолько подробно расписывать, выдернуть из себя хлёсткое название для какой-то модели-картинки и потом по нему её реконструировать - максимум на что я способен.

>> No.4954203  
Файл: 0B53 c3b.png -(47 KB, 320x320, 0B53 c3b.png)
47

>>4954039
Может на совсем не ухожу! Просто какой-то мой вопрос приведёт только к мучению всех, "кампании" по топологии/Пифагору/физике провалились

(Ещё боюсь, что слишком формалистом себя выставил последним постом. "Перегнул палку" тупости где-то, может быть)

> Мне понятны обе твои гипотезы, но кажется что довольно сложно очертить пространство всех возможностей просто речью и сказать какие возможности данным рассуждением отсекаются, обычно в математическом доказательстве говорят "голое рассуждение" оставляя остальную работу по построению пространства всех возможностей и выделению из них тех которые данным логическим шагом отсекаются на проработку изучающему.

Может что-то можно сделать, если решать эту проблему не на формальном уровне/не полностью? Ведь подобная проблема много где возникает всё равно — например, в комментировании шахматных партий (хотя ты не играешь)

...

> Лично я, кстати, не всегда стремлюсь за той моделью понимания, построенном на отсечении вариантов, которое ты описываешь. Ну потому что оно слишком трудоёмкое, а ресурсы памяти конечны, иногда даже дать удачное название или визуальный образ какому-то трюку, который поможет его просто хотя бы запомнить (и, возможно, реконструировать то комбинаторное понимание о котором говоришь ты при случае если тебя попросят этот трюк объяснить) уже большая удача.

Понял!

> Но попробовать можно конечно, чисто в качестве эксперимента хотя бы. Тогда я подумаю какую теорему выбрать и как её можно в таком стиле рассказать.

Если что, можем ещё обсудить что-нибудь об этом, чтобы не потратить попытку впустую!

А что думаешь в виде представления рассуждения в виде "слоёв/уровней" (каких-то условных групп утверждений)? Я вот в последнее время немного улучшил запоминание шахматных партий, став всегда отмечать в какую область доски (а не просто на какое поле) фигура делает ход — ассоциировать каждый объективный ход/объективное конкретное поле (куда пошла фигура) с каким-то субъективно-выделенным множеством полей. В доказательстве теоремы Пифагора подобными треугольниками, например, одним "уровнем" могут быть факты подобия, а другим "уровнем" соответствующие фактам численные отношения и чисто арифметические манипуляции... можно о разных уровнях подумать, в каком-то доказательстве (Rearrangement) "уровнем" может быть множество конструкций, которые можно собрать из треугольников... "уровнем" может быть и условно выбранное множество фигур, с которыми хотим сравнить треугольник... можно вводить и "выдуманные" мнимые неформальные уровни для чего-то... и в идеале мы должны на каждом уровне найти что-то "особенное", а потом соединить эти "особенности"/дисбалансы/симметрии (обычно же в рассуждении быстро скачат между разными уровнями и не "обозревают" их)(но может из моей затеи ничего и не выйдет)

Есть такая идея — сначала говоришь избыточно (!), а потом сокращаешь до того что было

Читал книгу Фейнмана о КЭД? Он рассказывает (с какими-то упрощениями) о суммировании по траекториям, как нарисовать эти траектории и сделать рассчёт. И рисует их для феномена за феноменом. У меня есть идея более простого стиля объяснения, вот мой пересказ книги:

  1. В пространстве есть сгустки [вероятностей] (постулируем уровень сгустков)
  2. Сгустки зависят от таких-то предметов в пространстве. Поэтому добавление датчика или изменение размера зеркала или щели может изменить световую картину (постулируем уровень каких-то предметов и какую-то его связь с уровнем сгустков; мы уже можем предложить какие-то предсказания, хоть мы их и не выводили)
  3. И вот в самую последнюю очередь можно уже поговорить о каких-то конкретных способах считать конкретные картины!

Что думаешь? Может оказаться совсем не трудоёмко

Мне такое помогает снимать психологические барьеры и отвлекаться от red herrings: после того как поверил сразу во всё, в избыточное количество вещей, проще потом поверить во что-то конкретное

> Кстати, чтобы подчеркнуть насколько мы похожи, я как-то говорил что у меня есть свой "список моделей" и я то что ты описываешь на ГотАИ назвал в этом списке "априорной равноправностью всех позиций", ну только у меня почему-то не хватало энергии это настолько подробно расписывать, выдернуть из себя хлёсткое название для какой-то модели-картинки и потом по нему её реконструировать - максимум на что я способен.

Здорово! Но может стоит поискать и различия: (просто чтобы знать, могу ли сообщить нечто новое)

Я придумывал идеологию (это наложило специфику), которая помогает выбирать удобные распределения/определения понятий. Поэтому говорю именно о понятиях/версиях понятий
Придумывал классификацию (тоже специфика), в которой есть абсолютно отличные объекты (отличающиеся любым свойством; или похожие всеми своими свойствами, с другой перспективы). Поэтому (опять) говорю о специализированных версиях понятий и всякой "синонимичности" (если у каждого понятия есть [немного] отличающийся синоним, можно представить две штуки, состоящие из похожих, но во всём [немного] отличающихся понятий: как галактика из материи и галактика из анти-материи)(если у понятий в каких-то ситуациях есть какие-то "измеримые" отличия, можно представить некий механизм, который будет создавать/собирать некую штуку именно из каких-то определённых версий понятий, или понятие которое будет уподоблять себе другие понятия, как Лёд-девять)

Короче хотел сказать что-то о структуре разных "точек зрения", даже если эта структура не содержится в них самих. Ввёл всякие версии/уровни чтобы можно было рассмотреть разные интересные (специфические) объекты или процессы

>> No.4956971  

>>4953580
А что всё-таки насчёт этого вопроса, его как-то заигнорили, мне тоже интересно.

Насколько я понимаю, ответ на него всё-таки нет и оно и есть те самые нестандартные модели?

>> No.4957480  
Файл: cirno1.png -(18 KB, 240x240, cirno1.png)
18

>>4956971
Можно доказать что "если х не последующее ни для какого натурального числа, то х=1", или, иначе "\forall n. S(n) \neq 1". Просто индукцией для 1 это верно (S(1) \neq 1), если S(n) \neq 1 то S(S(n)) \neq 1. Порядковый тип нестандартных моделей N_0 + (Z x D) где D это некоторый плотный порядок без концов (очень легко показать) откуда в принципе тоже видно что таких элементов нет.

>> No.4957525  

>>4953580

> существует такое x, что x ≠ 1 и x не является последующим ни для какого натурального числа
> Можно ли доказать, что это x заведомо не существует/не натуральное, опираясь только на 5 аксиом Пеано?

По аксиомам Пеано получается так, что все натуральные числа это единица и то, что можно получить прибавлением единицы к единице. Т.е. 1, 1+1, (1+1)+1, ((1+1)+1)+1 и так далее. Если нет никакого числа, которое идет перед этим x и если само x не равно 1 то получается что такое x невозможно получить из единицы путем ее увеличения на 1, так что этот x натуральным числом не является.

>> No.4957570  
Файл: cirno2.jpg -(340 KB, 840x1222, cirno2.jpg)
340

>>4957525

>По аксиомам Пеано получается так, что все натуральные числа это единица и то, что можно получить прибавлением единицы к единице. Т.е. 1, 1+1, (1+1)+1, ((1+1)+1)+1 и так далее

Не, этого как раз не получается (по крайней мере если брать первопорядковые аксиомы), потому что предикат "P(n) = n может быть представлено в виде конечной суммы единиц" невыразим в п.п. логике, из теорем о полноте+неполноте элементарно следует существование нестандартных моделей и тд и тд
Более того, что тоже легко доказать, любая счётная нестандартная модель имеет порядковый тип N + (Z x Q), то есть там заведомо есть элементы которые больше любой конечной суммы единиц.

>>4954203
Я о нашем эксперименте помню, просто никак не возьмусь, на самом деле я думаю расписать тебе теорему Лагранжа (о конечных группах) по многим причинам, во-первых это чистая аксиоматическая математика, то есть честный вывод утверждения из аксиом, во-вторых конечные группы это простой объект кодирующийся конечным числом информации, и не возникнет всяких сложностей с бесконечностями, в-третьих она априори неочевидно и не возникнет сложностей с тем что утверждение очевиднее своего доказательства, в-четвёртых её следствия очень важны в криптографии, как приятный бонус (малая теорема Ферма и RSA), в-пятых за этим всем стоит красивая геометрическая картинка позволяющая красиво доказательство запомнить. Почитай где-нибудь что-нибудь о конечных группах сперва, я конечно сам попытаюсь рассказать, но может ты с этим справишься быстрее и этого не потребуется.

>> No.4957573  
Файл: cirno4.jpg -(10 KB, 180x279, cirno4.jpg)
10

>>4957570

>Более того, что тоже легко доказать, любая счётная нестандартная модель имеет порядковый тип N + (Z x Q), то есть там заведомо есть элементы которые больше любой конечной суммы единиц.

Ну и да, вообще чтобы увидеть что есть модели в которых есть элементы которые больше любой суммы единиц знать точный порядковый тип не нужно, достаточно теоремы о компактности, ведь в наборе утверждений
c > 1
c > 1+1
c > 1+1+1
...
(где "c" некоторый новый константный символ) любой конечный поднабор имеет модель (стандартную, например), значит и весь набор имеет модель, в которой "с" будет элементом большим любой конечной суммы единиц.

>> No.4957915  
Файл: 196_lychrel_thread.jpg -(21 KB, 401x592, 196_lychrel_thread.jpg)
21

Можно ли, не имея доказательства чего либо, знать что это доказательство вообще существует? Вот например проблема 196, когда число отзеркаливают, само с собой складывают и надо понять, получим ли мы палиндром. Если палиндром мы когда-нибудь получим, то доказательство этому точно будет т.к. этим доказательством будет сам ход вычислений. А если нет, т.е. если палиндрома никогда не получается? Можем ли мы доказать для подобных случаев, что да, такое доказательство (для данного примера - доказательство невозможности получения палиндрома если палиндром действительно невозможно получить) точно существует для данных аксиом, но при этом самого доказательства пока что нет?

>> No.4957938  
Файл: cirno4.jpg -(242 KB, 1139x1600, cirno4.jpg)
242

>>4957915
Ну тут от некоторых философских позиций зависит, считать ли доказательство того что доказательство существует доказательством. Чисто формально, если ты можешь доказать доказуемость Р то ты можешь доказать и Р - это теорема Лёба.

Но, скажем, теорема Гёделя о полноте тебе говорит о том что если у тебя нечто семантически следует из утверждений, то должно следовать и синтаксически из них же, но при этом она тебе не даёт никакого способа это доказательство построить из аксиом чисто правилами вывода. Хотя, с другой стороны, если у тебя теория эффективная, то "перебирать все доказательства пока не найдётся нужное" тоже способ.

Конкретно про 196 не знаю, не думаю что можно (потому что, как я и говорил, с некоторых позиций доказательство доказательства само по себе доказательство), обычно все примеры такого типа, типа тех что я называл выше, довольно искуственные.

>> No.4957976  

>>4957938

>Ну тут от некоторых философских позиций зависит, считать ли доказательство того что доказательство существует доказательством.

Ну тут по-моему путаница какая-то. Вот конкретно проблема 196 - если итерация числа 196 (отзеркалить-сложить) когда-нибудь приведет к палиндрому, это можно доказать, доказательство точно существует если это так, ведь можно просто проделать цепочку вычислений и показать это. Но если мы знаем что если (и только если) 196 никогда не придет к палиндрому, то существует доказательство этого - само по себе это не доказательство, и оно не равноценно ему, из этого еще не следует, что число 196 к палиндрому никогда не придет. Ну зато мы тогда точно поймем, что эта проблема разрешима - можно одновременно и число 196 прогонять через цикл отразить/суммировать, и перебирать все возможные доказательства.

>> No.4957997  
Файл: cirno.jpg -(2591 KB, 1600x2196, cirno.jpg)
2591

>>4957976
Почему путаница? По-моему и я и ты нечто тривиальное говорим, но разное. Ты наверное хочешь, если пользоваться языком provability logic, что "из (P -> box P) не выводится P" это и правда так, я лишь хотел сказать что доказуемость P и непротиворечивость арифметики влечёт семантическую истинность P. Поэтому, с некоторых позиций (с тех в которых верим в непротиворечивость арифметики, например) доказать что нечто доказуемо это то же самое что и просто доказать.

>> No.4958403  
Файл: Screenshot_2019-09-24_13-40-31.png -(89 KB, 795x216, Screenshot_2019-09-24_13-40-31.png)
89

Вот есть аксиома http://us.metamath.org/ileuni/ax-1.html из Metamath но я не очень понимаю что она значит

⊢ (φ → (ψ → φ))
φ ψ это некие well formed formula (wff). Там ссылаются на теорему 2.02 из Principia Mathematica - вот та теорема. Какой вообще смысл? Если утверждение φ верно то из какого-то (любого? ) утверждения ψ следует верность утверждения φ? Или это значит что-то другое?

>> No.4958416  

Вторая аксиома кстати вполне понятна http://us.metamath.org/ileuni/ax-2.html
⊢ ((φ → (ψ → χ)) → ((φ → ψ) → (φ → χ)))
Если (если φ = TRUE, то (если ψ = TRUE то χ = TRUE) ), то (если (если φ = TRUE то ψ = TRUE), то (если φ = TRUE то χ = TRUE) ).

>> No.4958456  
Файл: cirnoo.jpg -(8 KB, 179x281, cirnoo.jpg)
8

>>4958403
Да, но нужно не забывать что импликация материальная, то есть \psi -> \phi буквально значит "(не psi) или phi". Можно считать так же что это обещаение, условный оператор, если psi выполнено, то и phi выполнено. Поэтому phi -> (psi -> phi) это буквально значит "если phi выполнено, то если psi выполнено, то phi выполнено" или, если записыать проще "если phi и psi выполнено, то phi выполнено", что уже действительно интуитивно тавтологией ощущается.

>> No.4958524  

Сырночки, объясните мне, на какашках и палках, что за вещь такая тензоры, для чего нужно и как в физике этим пользоваться, если и такие тут есть. С меня тонны нефтечая.

>> No.4958577  

Чем по смыслу отличаются ⊢, ⊨ и → ?
В чем разница между этими тремя утверждениями:
A → B ⊢ ¬B → ¬A
A → B ⊨ ¬B → ¬A
(A → B) → (¬B → ¬A)
?

>> No.4958617  
Файл: ыясчясячся.jpg -(140 KB, 900x1291, ыясчясячся.jpg)
140

>>4958524
Это очень обширный вопрос, который сложно раскрыть в рамках какашек и палок даже не зная твой уровень, но чтобы ты не ушел с пустыми руками советую загуглить "Гравитация Ч. Мизнер", можно скачать в интернете бесплатно без смс, книга старая и можно сказать даже слегка устаревшая, но ценность её в том, что автор доступно знакомить читателя со всем необходимым математическим аппаратом, включая основной инструмент теории относительности тензорны, а уж если ты интересуешься физикой, то этот талмуд доступно ответит и на многие другие твои вопросы.
Если хочется узнать о тензорах побыстрее то начинай читать сразу со второй главы, про тензоры там сразу после векторов.

>> No.4958835  
Файл: cirno123.jpg -(32 KB, 400x400, cirno123.jpg)
32

>>4958524
Ну я уже отвечал, есть полилинейные функции, т.е. линейные по каждому аргументу, это даёт некоторый очень простой и очень вычислимый способ их считать и строить вокруг них достаточно простую теорию. Основной инсайт математического анализа в том, что почти все сколь-угодно сложные отображения и функции на маленьком масштабе выглядят в нулевом как константа, в первом приближении как константа+линейная, а во втором как константа + линейная + квадратичная (т.е. полилинейные ранга 2) и тд, таким образом полилинейные функции естественно встречаются в физике когда мы хотим приближать обычные функции или отображения. Тензоры это способ рассматривать полилинейные функции как линейные, но на более сложном пространстве, ну примерно так.
>>4958577
Ну на пропозиционной логике отличия очень плохо будут видны, но окей. "A → B" это утверждение на объектном языке, это такая же логическая связка как и "А /\ В" или "А \/ В", в то время как "А ⊢ В" и "А ⊨ В" это утверждения на метаязыке. "А ⊢ В" означает синтаксическую доказуемость, что ты принципиально из аксиом подлежащей логики и утверждения А можешь пользуясь синтаксическими правилами вывода разрешенными в твоём языке вывести В, читается как. "А ⊨ В" можно читать как "семантически следует", оно говорит что в любой модели (из того класса моделей которые мы позволяем себе рассматривать для данного языка) в которой верно утверждение А верно и утверждение В. Иногда выделяют некоторый подкласс моделей С и пишут А ⊨_С В что значит что в любой модели из С из истинности А следует истинность В.

Если наша логическая система (= язык + семантика (выбор класса допустимых моделей)) такова, что "А ⊢ В" влечёт "А ⊨ В" то говорят что она обладает свойством корректности (по англ. soundness), если такова, что "А ⊨ В" влечёт "А ⊢ В" то говорят, что она обладает свойством полноты. Пропозиционная логика и предикатная логика первого порядка корректны и полны. В классических логиках (пропозиционной и предикатной первого порядка) выполняется так же "теорема о дедукции" то есть верно что "A ⊢ B" тогда и только тогда когда "⊢ А→В".

Думаю объяснил непонятно, но задавайте уточняющие вопросы если что.

>> No.4958836  

>>4958835

> читается как

читается как "из А доказывается В"

>> No.4959034  
Файл: slide_11.jpg -(59 KB, 960x720, slide_11.jpg)
59

С аксиомами логики высказываний для классической и для интуиционистской логикой что-то не очень понятно
http://ru.metamath.org/mpeuni/mmtheorems1.html#mm5s
Вот тут для классической логики введены 4 аксиомы:
1) ⊢ 𝜑 & ⊢ (𝜑 → 𝜓) ⇒ ⊢ 𝜓
Тут все просто, если верно 𝜑 и если из верности 𝜑 следует верность 𝜓 то верно 𝜓. Просто правило вывода
Эта аксиома есть и в интуиционистской логике.

2) ⊢ (𝜑 → (𝜓 → 𝜑))
С этой уже разобрались в >>4958403 >>4958456
Эта аксиома есть и в интуиционистской логике.

3) ⊢ ((𝜑 → (𝜓 → 𝜒)) → ((𝜑 → 𝜓) → (𝜑 → 𝜒)))
Тут тоже все ясно, >>4958416 и эта аксиома тоже есть в интуиционистской логике.

4) ⊢ ((¬ 𝜑 → ¬ 𝜓) → (𝜓 → 𝜑))
А вот эта аксиома есть только в классической логике. В интуиционистской логике ее нет : http://ru.metamath.org/ileuni/mmtheorems1.html#mm5s
С точки зрения булевой алгебры, это выражение всегда истинное (т.е. тавтология). Можно посмотреть таблицу истинности для логической импликации и для логического отрицания, построить таблицу истинности - получится что всегда TRUE какие б 𝜑 𝜓 туда не подставили.

В интуиционистской логике отрицание ¬ было введено в этой вот аксиоме http://ru.metamath.org/ileuni/ax-in1.html и там есть ссылка на теорему http://ru.metamath.org/ileuni/pm2.01.html которая такая же как и эта аксиома, названа она "доведение до абсурда"
⊢ ((φ → ¬ φ) → ¬ φ)
В классический логике это уже является нормальной теоремой, для доказательства которой понадобилась в т.ч. и аксиома ((¬ 𝜑 → ¬ 𝜓) → (𝜓 → 𝜑)) : http://ru.metamath.org/mpeuni/pm2.01.html
По смыслу это "если из верности φ следует ложность φ то тогда φ точно ложно". Ну да, истинным оно быть не может, можно построить таблицу истиности для такого выражения, используя логическую импликацию и отрицание - да, оно всегда истинное.

Какая таблица истинности будет для импликации в интуиционистской логике? Там что-то может быть одновременно правдой и неправдой? Или там какая-то многозначная логика? Таблицу истинности каких-то логических связок из интуиционистской логики вообще можно построить? Похоже что для классической логики все доказанные теоремы в этом metamath для пропозиционной логики это тавтологии с т.з. булевой алгебры. А есть ли какая-то такая алгебра, для которой тавтологией будут те и только те выражения, которые в пропозиционной логики являются истинными (т.е. доказуемыми теоремами)?

>> No.4959037  
Файл: cirnokk.jpg -(173 KB, 431x500, cirnokk.jpg)
173

>>4959034
В интуиционистской логике семантика чуть сложнее чем булевы алгебры и таблицы истинности. Чтобы получить полную корректную семантику интуиционистской пропозиционной логикой, нужно брать либо семантику во всех алгебрах Гейтинга (интуиционистский аналог булевых алгебр), либо семантику в мирах Крипке (интуиционистский аналог таблиц истинности). Это много где хорошо написано, например тут https://www.ps.uni-saarland.de/~dang/ri-lab/model/imodel.pdf неплохо вроде, но завтра, может, получше источники поищу.

>Похоже что для классической логики все доказанные теоремы в этом metamath для пропозиционной логики это тавтологии с т.з. булевой алгебры.

Ну это не очень интересное наблюдение, было бы, мягко говоря, странно если бы в metamath была доказана некая общезначимая пропозиционная формула не являющаяся тавтологией.

> А есть ли какая-то такая алгебра, для которой тавтологией будут те и только те выражения, которые в пропозиционной логики являются истинными (т.е. доказуемыми теоремами)?

Ну я полагаю что уже ответил выше, но вообще язык у тебя немного режет глаз. В алгебрах не бывает тавтологий, в них бывают элементы и операции над ними, тавтологии бывают в дедуктивных системах, истинность и доказуемость вещи разные, правильно твой вопрос нужно было бы сформулировать так "Существует ли некоторая полная и корректная семантика интуиционистского пропозиционного исчисления где моделями были бы некоторые алгебраические структуры, аналогичные булевым алгебрам в классическом пропозиционном исчислении?" (сори, решил в учителя поиграть)

>> No.4959057  
Файл: 0B57 jim-kay1.jpg -(448 KB, 1600x1194, 0B57 jim-kay1.jpg)
448

Я ВЕРНУЛСЯ! Не сразу увидел твой пост и не хотел писать до того, как будет готов текст, который может оказаться критически (не)важен для всего...

>>4957570

> Почитай где-нибудь что-нибудь о конечных группах сперва, я конечно сам попытаюсь рассказать, но может ты с этим справишься быстрее и этого не потребуется.

А из этого можно создать ИИ, если вместо нейронов будут группы и они будут общаться созданиями/ломаниями симметрий? Простите. Просто ты как-то упоминал очень страшные понятия из теории категорий, удивительно/ интересно применяются ли они к этому делу

Для меня это сложно отличить от какого-нибудь другого математического понятия, например топологии (какие-то штуки, и нам нельзя с помощью операции с ними выйти за какие-то пределы). Часы без минут, стрелку которых можно поворачивать на 0 ("единица") или 12 часов, это группа?

> в-пятых за этим всем стоит красивая геометрическая картинка позволяющая красиво доказательство запомнить

Жду!

Я ещё сам надеюсь, что смогу оторваться от "комбинаторного понимания" (для обучаемости)
https://www.cut-the-knot.org/proofs/Ants.shtml

Я смог психологически принять задачу так: над "слоем" наших объектов (муравьи) мы вводим "слой" более абстрактных объектов (разноцветные "флаги"), более абстрактных но более "симметричных"

Вот мой пост (9 пунктов), в котором попытался выпучить специфику своей идеи (отличия от "равноправия"/добавки к этому), если она есть:
https://dvuobyomnyi.livejournal.com/2607.html

Что думаешь, что-то оттуда может помочь эксперименту?

Этим по сути сжал весь ужасный-монструозный текст о "моделях"

>>4958835 этот пост заставил задуматься о том, к чему могло бы привести наше "соединение". Ты бы зачищал 90% математики и 10% словоблудия, я бы зачищал 90% словоблудия и 10% математики. Мы бы "догоняли" друг друга, двигаясь на самом деле по своим собственным направлениям, как небесные тела

которые падают друг на друга, но в то же время двигаются в разные стороны вокруг центра масс

> Ну я уже отвечал, есть полилинейные функции, т.е. линейные по каждому аргументу, это даёт некоторый очень простой и очень вычислимый способ их считать и строить вокруг них достаточно простую теорию. Основной инсайт математического анализа в том, что почти все сколь-угодно сложные отображения и функции на маленьком масштабе выглядят в нулевом как константа, в первом приближении как константа+линейная, а во втором как константа + линейная + квадратичная (т.е. полилинейные ранга 2) и тд, таким образом полилинейные функции естественно встречаются в физике когда мы хотим приближать обычные функции или отображения. Тензоры это способ рассматривать полилинейные функции как линейные, но на более сложном пространстве, ну примерно так.

Это не я спрашивал про тензоры (второй раз), но теперь это читается совсем иначе (чем когда в мыслях кирпичи)!

> "А ⊨ В" можно читать как "семантически следует", оно говорит что в любой модели (из того класса моделей которые мы позволяем себе рассматривать для данного языка)
> Думаю объяснил непонятно, но задавайте уточняющие вопросы если что.

А текст может выполнять роль уточняющего вопроса? (будет время, сам тоже честно подумаю)

>> No.4959058  
Файл: 0B57 jim-kay1.jpg -(448 KB, 1600x1194, 0B57 jim-kay1.jpg)
448

Я ВЕРНУЛСЯ! Не сразу увидел твой пост и не хотел писать до того, как будет готов текст, который может оказаться критически (не)важен для всего...

>>4957570

> Почитай где-нибудь что-нибудь о конечных группах сперва, я конечно сам попытаюсь рассказать, но может ты с этим справишься быстрее и этого не потребуется.

А из этого можно создать ИИ, если вместо нейронов будут группы и они будут общаться созданиями/ломаниями симметрий? Простите. Просто ты как-то упоминал очень страшные понятия из теории категорий, удивительно/ интересно применяются ли они к этому делу

Для меня это сложно отличить от какого-нибудь другого математического понятия, например топологии (какие-то штуки, и нам нельзя с помощью операции с ними выйти за какие-то пределы). Часы без минут, стрелку которых можно поворачивать на 0 ("единица") или 12 часов, это группа?

> в-пятых за этим всем стоит красивая геометрическая картинка позволяющая красиво доказательство запомнить

Жду!

Я ещё сам надеюсь, что смогу оторваться от "комбинаторного понимания" (для обучаемости)
https://www.cut-the-knot.org/proofs/Ants.shtml

Я смог психологически принять задачу так: над "слоем" наших объектов (муравьи) мы вводим "слой" более абстрактных объектов (разноцветные "флаги"), более абстрактных но более "симметричных"

Вот мой пост (9 пунктов), в котором попытался выпучить специфику своей идеи (отличия от "равноправия"/добавки к этому), если она есть:
https://dvuobyomnyi.livejournal.com/2607.html

Что думаешь, что-то оттуда может помочь эксперименту?

Этим по сути сжал весь ужасный-монструозный текст о "моделях"

>>4958835 этот пост заставил задуматься о том, к чему могло бы привести наше "соединение". Ты бы зачищал 90% математики и 10% словоблудия, я бы зачищал 90% словоблудия и 10% математики. Мы бы "догоняли" друг друга, двигаясь на самом деле по своим собственным направлениям, как небесные тела

которые падают друг на друга, но в то же время двигаются в разные стороны вокруг центра масс

> Ну я уже отвечал, есть полилинейные функции, т.е. линейные по каждому аргументу, это даёт некоторый очень простой и очень вычислимый способ их считать и строить вокруг них достаточно простую теорию. Основной инсайт математического анализа в том, что почти все сколь-угодно сложные отображения и функции на маленьком масштабе выглядят в нулевом как константа, в первом приближении как константа+линейная, а во втором как константа + линейная + квадратичная (т.е. полилинейные ранга 2) и тд, таким образом полилинейные функции естественно встречаются в физике когда мы хотим приближать обычные функции или отображения. Тензоры это способ рассматривать полилинейные функции как линейные, но на более сложном пространстве, ну примерно так.

Это не я спрашивал про тензоры (второй раз), но теперь это читается совсем иначе (чем когда в мыслях кирпичи)!

> "А ⊨ В" можно читать как "семантически следует", оно говорит что в любой модели (из того класса моделей которые мы позволяем себе рассматривать для данного языка)
> Думаю объяснил непонятно, но задавайте уточняющие вопросы если что.

А текст может выполнять роль уточняющего вопроса? (будет время, сам тоже честно подумаю)

>> No.4959151  
Файл: cirnok.jpg -(792 KB, 1920x1080, cirnok.jpg)
792
>А из этого можно создать ИИ, если вместо нейронов будут группы и они будут общаться созданиями/ломаниями симметрий? Простите. Просто ты как-то упоминал очень страшные понятия из теории категорий, удивительно/ интересно применяются ли они к этому делу

Не знаю, но мне всё же разумной в таких вопросах кажется позиция Посицельского "теоремы нужны для того, чтобы доказывать другие теоремы, исключения крайне редки и на дороге не валяются". Из желания типа "а давайте-ка прикрутим вот эту вот математику к этой вот задаче" редко даже хорошая математика получается, не говоря уже о продуктивных приложениях.

>А текст может выполнять роль уточняющего вопроса? (будет время, сам тоже честно подумаю)

Конечно!

>Для меня это сложно отличить от какого-нибудь другого математического понятия, например топологии (какие-то штуки, и нам нельзя с помощью операции с ними выйти за какие-то пределы). Часы без минут, стрелку которых можно поворачивать на 0 ("единица") или 12 часов, это группа?

Группой скорее будет не стрелка и не цифры на циферблате, а операция "Повернуть стрелку на k часов" соответственно единицей будет операция "Повернуть стрелку на 0 часов (не поворачивать стрелку)".

Я твой текст обязательно прочитаю, и обязательно напишу пять-шесть абзацев о том что такое конечная группа, пытаясь писать излишне подробно и очерчивать явно "пространство вариантов и какие варианты теми или иными условиями отсекаются", но точно не сегодня и наверное не завтра.

>Это не я спрашивал про тензоры (второй раз), но теперь это читается совсем иначе (чем когда в мыслях кирпичи)!

Я рад! Ну это в некотором смысле из-за того что сейчас я "полуправду" написал, а предыдущим постом пытался писать "правду", ну энивей, овладение любым математическим понятием в полной мере происходит через работу с ним и извлечение его смысла из контекста употребления .

>> No.4959214  

>>4959037

>Это много где хорошо написано, например тут https://www.ps.uni-saarland.de/~dang/ri-lab/model/imodel.pdf неплохо вроде, но завтра, может, получше источники поищу.

Нет, что-то не очень понятно мне.
Вот начал я читать... Heyting algebra is a preorder - что такое preorder? Пошел гуглить - похоже что это такое множество, и там между элементами введено бинарное соотношение ≤ которое обладает какими-то свойствами

https://en.wikipedia.org/wiki/Preorder
Consider some set P and a binary relation ≤ on P. Then ≤ is a preorder, or quasiorder, if it is reflexive and transitive; i.e., for all a, b and c in P, we have that:
a ≤ a (reflexivity)
if a ≤ b and b ≤ c then a ≤ c (transitivity)

Если сказано что это set т.е. множество, то на основе какой теории множеств это строится? Или это неважно? Есть же ZFC теория, в том же metamath есть IZF которая построена на интуиционистской алгебре высказываний. Ну и кроме ZFC и IZF есть всякие другие разновидности теорий множества.
Получается, алгебра Гейтинга как-то описывает пропозиционные исчисления, но для описания этой алгебры Гейтинга нужно знать что такое preorder, для описания preorder нужна какая-то теория множеств, которая в свою очередь должна описываться на некоем пропозиционном исчислении... Какой-то замкнутый круг. Как это все начать распутывать?

>> No.4959228  
Файл: cirno9.jpg -(62 KB, 1100x488, cirno9.jpg)
62

>>4959214
Забавно, что ты знаешь что такое IZF и не знаешь что такое предпорядок :3 Можешь считать, что ZFC, вообще более-менее не зависит от ТМ оснований.

Алгебры Гейтинга не "описывают", а "являются семантикой" для инт. пропозиционного исчисления, а семантика она и должна быть "внешней" по отношению к тому синтаксису для которого она является семантикой. С булевыми алгебрами и классическим пропозиционным исчислением ситуация точно такая же, чтобы определить модель (классического) пропозиционного исчисления, тебе нужно выбрать булеву алгебру (которая, вообще говоря, "множество элементов + 5 операций на нём") и выбрать отображение которое ставит в соответствие каждой переменной некоторый элемент булевой алгебры и ты можешь это сделать только исходя из некоторого "внешнего взгляда" на синтаксис.

Формально строго доказывать теоремы про семантику каких-то языков (чтобы это не было философией) можно только в каком-то "более высоком синтаксисе" (скажем, в той же ZFC которую ты упомянул) который мощный в достаточной степени чтобы промоделировать внутри себя инт. пропозиционное исчисление и алгебры Гейтинга и отношение между пропозиционным исчислением и алгебрами Гейтинга. Но конечно, как ты наверное верно заметил "пропозиционное исчисление" в этот момент раздваивается, есть пропозиционное исчисление которое кусок синтаксиса ZFC а есть пропозиционное исчисление которое объектный язык, определённый в рамках самой ZFC, и что-то доказывать/формулировать мы можем только про этот объектный язык, ну и ничего страшного.

Вообще ты задал достаточно хороший и тонкий вопрос, который уже тут задавали и который требует некоторого самостоятельного продумывания, но почитай ещё этот тред, может найдёшь там что-то для себя:

https://math.stackexchange.com/questions/1334678/does-mathematics-become-circular-at-the-bottom-what-is-at-the-bottom-of-mathema

>> No.4959237  
Файл: 0B63 loose-cannon-2516-9ce425f34157bfa72(...).jpg -(28 KB, 360x270, 0B63 loose-cannon-2516-9ce425f34157bfa72(...).jpg)
28

>>4959151

Могу ли я дать какую-то полезную информацию по ходу дела? Прости за длинный ответ из 3-х частей, это идеи о понятности из нового текста (но вне текста они унылы)

(Почти досмотрел "Интро" к специальной теории относительности, Парадокс близнецов остался. Докладываю)

> Из желания типа "а давайте-ка прикрутим вот эту вот математику к этой вот задаче" редко даже хорошая математика получается, не говоря уже о продуктивных приложениях.

Это была двухходовка! Та же тема поднимется в тексте.

Хотя вообще пролетаю как фанерка в этом треде, скрываю оставшиеся живые места за иронией и безумными вбросами

> Часы без минут, стрелку которых можно поворачивать на 0 ("единица") или 12 часов, это группа?
> поворачивать на 0 ("единица") или 12 часов
> Группой скорее будет не стрелка и не цифры на циферблате, а операция "Повернуть стрелку на k часов" соответственно единицей будет операция "Повернуть стрелку на 0 часов (не поворачивать стрелку)".
> "Повернуть стрелку на k часов"
> Группой

Я не назвал группой цифры!.. хотя (с другой стороны) получилось, видимо, именно так, что назвал. Подумал о каком-то неправильном (хреновом) ограничении. Уже полезли из под земли какие-то "комбинаторные собаки" (ЧАСТЬ 1)

Меня сбило с толку, что бывают конечные и бесконечные группы. И нам нужна именно конечная, что-то конечное

Ограничивать повороты конкретными часами конечно, очень тупо! "Придумывай сам, такого бы не сделал" хочется сказать. Тем более задался бы вопросом, какой это имеет смысл, если вокруг часов можно создать дополнительную систему, которая вращает сами часы (хотя может это как раз и не аргумент, но суть не в этом, а в показании путаницы!) — но вспомнив, что в прошлой дискусии (топология) это больше вело лишь к нудятине, оставил вопрос при себе

Ещё сбило с толку какое-то видео, где выписывали "таблицу умножения элементов"
https://youtu.be/wA8Wa2zAJ3E?t=191

Видимо, есть какое-то более умное (менее ограничивающее) ограничение
https://ru.wikipedia.org/wiki/Конечная_группа#Конечные_циклические_группы
(Хотел что-то сказать по этому поводу, но понял что это уже не важно:)

Кажется, понимаю что ты хотел сказать, если конечность уже есть в элементах, она не нужна в операциях

> Я твой текст обязательно прочитаю, и обязательно напишу пять-шесть абзацев о том что такое конечная группа, пытаясь писать излишне подробно и очерчивать явно "пространство вариантов и какие варианты теми или иными условиями отсекаются", но точно не сегодня и наверное не завтра.

Возможно, может ещё помочь описание действий: что мы хотим делать с часами, какого поведения от них хотим, а какого не хотим. И уже потом лепить на это "математическую разметку" (это есть то, то-то есть сё)(ЧАСТЬ 2)(ЧАСТЬ 3:)

Ты как-то вспоминал Вигтенштйена, помнишь его мем про крутящееся колесо? В новом тексте есть идея "соответствия понятия себе/силы связи"

Пример из морали: можно очень любить людей, но не считать никого человеком/не распознавать никого как человека/быстро переставать считать кого-то человеком. Это значить, что понятие не соответствует самому себе, слишком свободно крутится

Идея в том, что любой выбор это на самом деле два выбора, иначе никак

Полу-абстрактный пример: можно представить стрелку на диске и ограничить её движение по нему. Но это может иметь мало смысла, если можно крутить сам диск (закрепили лишь один слой) — получается большая пушка, которая вместо неуклюжести стремительно летает по палубе (loose cannon)

Или что-то, связанное с относительностью:
https://ru.wikipedia.org/wiki/Пространство-время_теории_Ньютона#Математическая_модель
https://www.physicsforums.com/threads/affine-space-or-fibre-bundle-spacetime-formalism-in-newtonian-mechanics.186908/

Какие-то проблемы в понимании ещё могут быть от ощущения "слабости связи" разных слоёв/непонимания этой связи, что могло быть и с Топологией например:

  • Какой смысл упоминать о какой-то связи множеств и меняющихся параметров, если множества не меняются
  • Насколько опасно иметь объекты, "забывающие" своё содержание (не начнут ли рождаться новые сущности из ничего и прочая ересь? ты успокоил, что нет. но меня это пугало как loose cannon)
  • Какой смысл говорить, бывают ли элементы без множества, если не оговаривать бывает ли нечто кроме элементов/как они появляются? Иначе по факту окажется что вполне себе бывают, просто меняют название (как в моральном примере). Опять loose cannon. Опять отсутствие двойного выбора. В принципе ничего страшного, но немного сбивает непонимание "модальности" каких-то утверждений об элементах и множествах
  • Возможно, где-то близко к этому и причина непонимания механики пустого множества (будто это что-то типа Quine atom) и топологии и группы с первого раза, но сложнее
> Но конечно, как ты наверное верно заметил "пропозиционное исчисление" в этот момент раздваивается, есть пропозиционное исчисление которое кусок синтаксиса ZFC а есть пропозиционное исчисление которое объектный язык, определённый в рамках самой ZFC, и что-то доказывать/формулировать мы можем только про этот объектный язык, ну и ничего страшного.
> Вообще ты задал достаточно хороший и тонкий вопрос, который уже тут задавали и который требует некоторого самостоятельного продумывания, но почитай ещё этот тред, может найдёшь там что-то для себя:
> circular

Зацикленная логика это одна из тем текста, связанная с принципом "двух выборов" (выбор должен быть предопределён заранее, грубо говоря)... хотя может никакой связи нет

>> No.4959695  

>>4959228
Ну, если проблемы с зацикленностью как бы нет (или они неважны) можно ли описать исчисление высказываний через empty theory, uninterpreted function? Кто-нибудь так делал?

>> No.4959750  
Файл: cba96f43cf66ad7a1e62abcca37dab44.jpg -(74 KB, 564x564, cba96f43cf66ad7a1e62abcca37dab44.jpg)
74

Несмотря на хорошие способности к простейшей арифметике в начальных классах, где-то на квадратных уравнениях или даже дробях я застопорился в своём развитии и даже пошёл вниз, наверно, из-за чего превратился в безнадёжного гуманитария.
Ну или я резко потупел
Помогут ли вот эти книги исправить данную неисправность?

Савватеев - Математика для гуманитариев

Кечеджан - Математика для безнадёжных гуманитариев

>> No.4959755  

>>4959750
Потому что арифметика и алгебра - совершенно разные вещи. Во всяком случае, при тех методах которыми пользуются в образовательных учреждениях. До определённого момента тебе поможет осознание того что все операции осуществляются по таблицам, которые нужно тупо зубрить. Не только умножение, все. Дальше ты можешь запнутся на перетасовке формул. Кто не может нутром чувствовать как правильно их тасовать по формулам чтоб получить ответ обычно фейлится без грамотного объяснения. Грамотного объяснения мне пока не попадалось, только примеры. И пусть тот кто возразит что учить только на примерах это нормально и не как в каменном веке, никогда и никого не обучает.

>> No.4959825  
Файл: cirnooo.jpeg -(5 KB, 224x225, cirnooo.jpeg)
5

>>4959237
Сори, прочитаю обзательно, просто это не коротко.
>>4959695
Зацикленности нет.

Ты что-то путаешь, пустая теория - это всё равно теория (в, скажем, пропозиционном или предикатном исчислении) и она предполагает под собой наличие некоторй подлежащей логики (пропозиционной в том числе). То есть вопрос вообще некорректен, потому что в "пустой теории" уже "зашито" пропозиционное исчисление (или некоторая другая подлежащая логика).

>>4959750
Ну, всем подходят разные книжки и у меня мнение что читать надо всё что нравится читать, но лично мне такой стиль как в этих книжках кажется сильно не продуктивным для изучения чего-либо. Попробуй Гельфанд Шень "Алгебра", Шень вообще прекрасные книжки пишет мне кажется, а Гельфанд это литералли классик науки и с его вкладом ознакомлен каждый более-менее образованный математик (ну, кроме тех кто анализом совсем уж брезгует).

>> No.4960244  
Файл: 1024px-Rieger-Nishimura.svg.png -(38 KB, 1024x878, 1024px-Rieger-Nishimura.svg.png)
38

Я кажется не до конца понимаю то, как работает этот Metamath.
Вот есть аксиома modus ponens:
http://us.metamath.org/ileuni/ax-mp.html
Hypotheses:
⊢ φ
⊢ (φ → ψ)
Assertion:
⊢ ψ
А есть такая вот тероема, которая через что-то там доказывается и зависит от той аксиомы modus ponens:
http://us.metamath.org/ileuni/pm3.35.html
Assertion:
⊢ ((φ ∧ (φ → ψ)) → ψ)

Вроде бы то же самое. Кажется что смысл в том, что эта ax-mp означает:
вот если у тебя есть "φ" и "(φ → ψ)" выражения в некоем наборе, то можно из него сделать "ψ" т.е. мы можем еще и добавить к набору выражений "ψ". И эти φ ψ это какие-то wff.

А теорема pm3.35 это значит "для любых двух wff φ, ψ можно записать выражение ((φ ∧ (φ → ψ)) → ψ) и потом что-то с ним делать"

Там еще есть две теоремы
http://us.metamath.org/ileuni/simpli.html :
Hypothesis
⊢ (φ ∧ ψ)
Assertion
⊢ φ
и эта
http://us.metamath.org/ileuni/simpri.html :
Hypothesis
⊢ (φ ∧ ψ)
Assertion
⊢ ψ

которые выводятся через modus ponens аксиому и две соответствующие аксиомы для "and"
http://us.metamath.org/ileuni/ax-ia1.html
http://us.metamath.org/ileuni/ax-ia2.html

Т.е. там есть какой-то свой внутренний "and" и отдельно есть описанный через саму эту машинерию "and"... Можно ли сделать как-нибудь саморефлективно это, чтобы был один "and"? И вообще, чтоб сама машинерия себя же и описывала? Есть ли подобный proof checker/proof assistant?

>> No.4960328  
Файл: cirno.png -(84 KB, 360x600, cirno.png)
84

>>4960244
Не очень понял о чём ты, я вижу только один and, который /\, о том как это реализовано "внутри" не наше дело ведь, вообще любая программа запускается на компьютере, процессор которого суть сложная транзисторная схема, в которой тоже есть куча элементов, которые по сути and.

>вот если у тебя есть "φ" и "(φ → ψ)" выражения в некоем наборе, то можно из него сделать "ψ" т.е. мы можем еще и добавить к набору выражений "ψ". И эти φ ψ это какие-то wff.
>А теорема pm3.35 это значит "для любых двух wff φ, ψ можно записать выражение ((φ ∧ (φ → ψ)) → ψ) и потом что-то с ним делать"

Вроде всё правильно сказал!

>> No.4960445  

>>4960328

>Не очень понял о чём ты, я вижу только один and, который /\, о том как это реализовано "внутри" не наше дело ведь

Я о том, что есть т.н. closed form, а есть правило вывода
Вот например две теоремы:
http://us.metamath.org/ileuni/2th.html
http://us.metamath.org/ileuni/pm5.1im.html
Первая это "если мы видим некие wff φ ψ то можем еще сделать (φ ↔ ψ)"
А второе это "мы можем записать (φ → (ψ → (φ ↔ ψ))) подставив в φ ψ какие-то wff".
По сути то же самое. Мне вот хочется чтоб не надо было делать две такие штуки, а чтоб была только одна, которая и говорила и про то что можно сделать такую-то подстановку, и про синтез из двух wff новой wff. В общем как-то так

>> No.4960835  

А чем вообще лучше это интуиционистское исчисление высказываний в сравнении с классическим, если рассматривать практичные вещи? Ведь математика это не только какие-то непонятно зачем нужные теоретические построения и доказывания всяких теорем вроде теоремы Ферма. Есть прикладная криптография, есть всякие инженерные расчеты типа сопромата, гидрогазодинамики, есть всякие системы помехоустойчивого кодирования. Что-то меняется, если строить (описывать) математику на другом фундаменте? Может быть если строить на интуиционистском исчислении высказываний, какие-то теоремы проще доказываются (и доказав какую-то теорему можно допустим доказать криптостойкость какого-то алгоритма шифрования)? По-моему наоборот, доказывать там сложнее.

>> No.4960920  
Файл: cirno1234.jpg -(724 KB, 2115x1254, cirno1234.jpg)
724

>>4960445
Ну теоретически можно сделать пруфчекер с уже "вшитой" теоремой о дедукции так, скажем, сделал Тао в своей обучающей программе QED https://www.math.ucla.edu/~tao/QED/QED.html но не помню чтобы в серьезных пруфчекерах такое было. То что пара теорем пропозиционной логики раздвоилось, это ещё не самое страшное что может с пруфчекером случиться.

>>4960835
Если практичные вещи, то ни в чём. На интуиционистской логике ничего проще доказываться не может, потому что это подмножество классической (то есть любая интуиционистская теорема она одновременно и классическая, а в от наооброт нет), её используют обычно чтобы избежать "неконструктивности" то есть доказательств существования объектов, которые явно его не предъявляют.

>> No.4960977  
Файл: 138f6n.jpg -(52 KB, 546x855, 138f6n.jpg)
52

>>4960920

>то есть любая интуиционистская теорема она одновременно и классическая, а в от наооброт нет

А можно ли как-нибудь узнать процент теорем, которые в классической логике выводимы и являются правильными теоремами (тавтологиями), а в интуиционистской невыводимы из аксиом?
Хотя тут надо еще знать порядок перечислений счетнобесконечного множества. Ну скажем если мы хотим узнать, какой процент натуральных чисел являются четными, то мы можем идти подряд, 1,2,3,4 -> 1 нечет, 2 чет, 3 нечет, 4 чет. И тут получится что четных 50% и нечетных тоже 50%
А можем каким-нибудь запутанным образом. Например, 1, 3, 2, 5, 7, 4, 9, 11, 6, 13, 15, 8 ... т.е. два нечетных и один четный, и таким образом мы тоже перебираем все натуральные числа, но если мы так будем считать процент четных натуральных чисел, результат будет другим. А каким образом правильно перечислять выводимые теоремы классической логики, чтоб потом узнать процент тех, что в интуиционистской невыводимы?

>> No.4960979  

>>4959755
А вот давай не надо, меня аж в дрожь бросило. Какое зазубриваение таблиц? Все что в математике есть обьяснено и как дважды два понятно, за исключением теории чисел, но в ней и зазубривать ничего не надо. Кто умеет шевелить тот математику поймет и таблицы сами по себе запомнятся без мучительного зазубривания. Бака

>> No.4961015  
Файл: cirnokl.jpg -(62 KB, 200x438, cirnokl.jpg)
62

>>4960977
Думаю нужно смотреть количество классических теорем длины <= n и количество интуиционистских той же длины и смотреть на предел. Вопрос забавный, ответ скорее всего либо 0 либо 1 по каким-нибудь тривиальным причинам, отпишу если придумаю.

>> No.4962072  

Есть один матан начинайко-уровня и есть одна девочка, которая ненавидит рисовать скучные графики, таблицы и прочие диаграммы. Сильно ли нерисование особо скучных графиков увеличит баковость в будущем?

>> No.4962537  
Файл: 0B62 Breaking_Bad_logo.png -(81 KB, 1280x763, 0B62 Breaking_Bad_logo.png)
81

>>4961015

До октября я хотел сказать, что буду в октябре идти к врачу, проверять падения оптимизма... Но думал что в любом случае дождусь поста, а получилось как в анекдоте каком-то

> Не знаю, но мне всё же разумной в таких вопросах кажется позиция Посицельского "теоремы нужны для того, чтобы доказывать другие теоремы, исключения крайне редки и на дороге не валяются". Из желания типа "а давайте-ка прикрутим вот эту вот математику к этой вот задаче" редко даже хорошая математика получается, не говоря уже о продуктивных приложениях.

Даю любую часть тела на отсечение. Вот так вот! Иначе всё равно будет то же самое или даже ещё хуже... Больше не время для меня оперировать гипотетическими критериями разумности или адекватности, я иду "Во все тяжкие"

Я собираюсь "штурмовать" все области знания

Сначала буду (вечером) штурмовать Реддит рационалистов, на r/HPMOR меня уже забанили...

Вот пока бесполезная ссылка для штурма:
https://boiinthenut-rationalityisbad.blogspot.com/

Потом буду создавать тред, в который попрошу тащить знание из всех областей знаний и разбираться в симметрических идеях

>> No.4963235  
Файл: cirnoguitar.jpg -(431 KB, 1200x1200, cirnoguitar.jpg)
431

>>4962537
Не обижайся, просто ИРЛ столько всего происходит НЕПРИЯТНОГО, что хоть галоперидолом ставься, и тут не до текстов на несколько экранов на иичане (хотя иичан очень люблю и тексты на несколько экранов писать тоже люблю). Ну давай попробую объяснить про конечные группы, моё объяснение будет немного отличаться от большинства классических способов объяснять, и начну я совсем издалека.

Итак, начать надо с того что любое определение в математике можно подвести под одну и ту же схему - это данные и условия которым эти данные должны удовлетворять. Данные чаще всего это несколько множеств (возможно бесконечных, но у нас всё будет конечно) и какие-то операции и отношения на них. Условия чаще всего - это некоторый набор аксиом которым должны удовлетворять данные чтобы удовлетворять определению. Про то что чисто формально отношения и отображения - это тоже множества думать чаще всего НЕ НАДО, их именно нужно воспринимать как отдельные сущности. Определение чаще всего задаёт не один объект, а целый класс объектов, любой набор данных удовлетворяющий аксиомам будет подходить под определение и будет являться экземпляром этого класса (терминология нестандартная).

Пример математического определения 1. Отношение эквивалентности ∽ на Х. Данные: множество X и отношение ∽ на нём (опять же, формально отношение это подмножество X x X но думать так про это не надо, нужно думать что отношение это просто некоторая формальная машинка которой дают два элемента из Х на вход и на выходе она говорит, находятся ли они в отношении или нет)
Условия на данные: для любых x, y, z из Х должно выполняться

  1. x ∽ x
  2. x ∽ y тогда и только тогда, когда y ∽ x
  3. если x ∽ y и y ∽ z, то x ∽ z

Пример экземпляра 1: X={a,b,c}, a∽a, b∽b, c∽c, a≁b, b≁a, a≁c, c≁a, b≁c, c≁b (каждая буква состоит в отношении только с самим собой и ни с чем больше) такое отношение эквивалентности называется "дискретное отношение эквивалентности на множестве X"
Пример экземпляра 2: X={a,b,c}, a∽a, b∽b, c∽c, a∽b, b∽a, a∽c, c∽a, b∽c, c∽b (каждая буква состоит в отношении с любой другой буквой) такое отношение эквивалентности называется "антидискретное отношение эквивалентности на множестве X"
Пример экземпляра 3: X={a,b,c}, a∽a, b∽b, c∽c, a∽b, b∽a, a≁c, c≁a, b≁c, c≁b (a и b состоят в отношении друг с другом и с самими собой, а c не состоит в отношении с a и b)
Непример экземпляра 1: X={a,b,c} a∽a, b∽b, c≁c, a≁b, b≁a, a≁c, c≁a, b≁c, c≁b это НЕ пример отношения эквивалентности, так как тут нарушена аксиома 1, c не состоит в в отношении с самим собой, а по аксиоме 1 любой элемент множества Х должен состоять в отношении с самим собой, чтобы иметь право называться отношением эквивалентности
Непример экземпляра 2: X={a,b,c} a∽a, b∽b, c∽c, a∽b, b≁a, a≁c, c≁a, b≁c, c≁b это НЕ пример отношения эквивалентности, так как тут нарушена аксиома 2, а состоит в отношении с b, но вот b с a в отношении не состоит, а по аксиоме 2 для любых двух элементов x и y должно выполняться: если x∽y то и y∽x
Непример экземпляра 3: X={a,b,c} a∽a, b∽b, c∽c, a∽b, b∽a, a∽c, c∽a, b≁c, c≁b это НЕ пример отношения эквивалентности, так как тут нарушена аксиома 3, ведь a состоит в отношении с b и b состоит в отношении с с, а вот а вместе с с не состоят в отношении, а по аксиоме для любых элементов x, y, z должно быть верно, что если x∽y, y∽z то и x ∽ z
Пример экземпляра 4 (менее тривиальный):
Х={...,-2,-1,0,1,2,3,4,...} (множество всех целых чисел), определим ∽ следующим образом, скажем что a∽b если и только если b-a чётное и a≁b если и только если b-a нечётное. Легко видеть, что это определение корректно (то есть не может быть ситуации когда одновременно a∽b и a≁b или когда для какой-то пары элементов ни a∽b, ни a≁b не выполняются) действительно, ведь b-a либо чётное, либо нечётное, оно не может быть чётным и нечётным одновременно или не быть чётным и не быть нечётным одновременно. Легко можно проверить что это отношение эквивалентности, и правда, пусть x,y,z это любые элементы из X (то есть любая тройка целых чисел)

  1. x-x в нашем случае это означает что x-x=0 чётное, но 0 действительно чётное, значит эта аксиома для нашего набора данных (X,∽) выполнена
  2. x-y тогда и только тогда, когда y-x в нашем случае это означает что если x-y чётное, то и y-x=-(x-y) чётное, но и правда ведь, если некоторое целое число (x-y) чётное, то и его противоположное число -(x-y) тоже чётное, потому что это по абсолютному значению то же самое число, но с противоположным знаком, а чётность не зависит от знака.
  3. если x-y и y-z то и x-z, в нашем случае это значит что если y-x чётное и z-y чётное, то и z-x чётное, но и правда ведь z-x=(y-x)+(z-y) если мы знаем что y-x и z-y чётные, то и z-x чётное, так как сумма чётных чисел чётна.

Скажи, хорошо ли тебе понятно определение и разобранные примеры?

>> No.4963236  
Файл: cirnostrange.jpg -(70 KB, 700x700, cirnostrange.jpg)
70

>>4962072
Нет, совсем не сильно, это довольно специфическое и аутичное занятие, которое многих просто отторгает своей специфичностью и аутичностью.

>> No.4963241  

>>4963235
У-у-у, ну я там нафакапил в примерах немного, но надеюсь ты всё равно разберешься.
Пример 3:

>a∽a, b∽b, c∽c, a∽b, b∽a, a∽c, c∽a, b≁c, c≁b

должно быть
a∽a, b∽b, c∽c, a∽b, b∽a, a≁c, c≁a, b∽c, c∽b

в Примере 4: везде вначале должны быть тильды вместо минусов, т.е. вместо

>x-x в нашем случае

должно быть x∽x

>> No.4963265  

>>4963235

Ну тогда тем более не хочу тебя/себя мучить, раз это ещё на фоне таких событий! (Спасибо, что всё равно написал) Мне ничего не мешает, но психологически тяжело от того что кажется проигрался с идеями (и некуда их нести)

> НЕ пример отношения эквивалентности, так как тут нарушена аксиома 3

Транизитивность по-моему уже в третьем примере была нарушена, либо не хочу тратить твоё время (и даже узнавать, что на самом деле не была — это всё равно как обычный учебник. "Если надо объяснять, то не надо объяснять")

> Скажи, хорошо ли тебе понятно определение и разобранные примеры?

Я не знаю, в учебнике топологии тоже было всё более-менее "понятно", а потом оказалось... (это всё зависит от контекста, который потом будет наслаиваться)

Я думал, ты хочешь сказать что чётные числа чётны в обе стороны и по ассоциации... то что ты перешёл к вычитанию и складыванию было не понятно (видимо, дело в том что на больших множествах надо как-то дополнительно определять эквивалентность, связывать её с чем-то, иначе она не имеет смысла, или что?)

Но это всё фигня по-моему — чем мелководнее, тем непонятнее, может Лагранж с исключениями вариантов сработал бы?

>> No.4964004  

Как понимать математику?
Как не бояться математики?

>> No.4964028  
Файл: chibicirno.jpg -(25 KB, 466x466, chibicirno.jpg)
25

>>4964004
Больше времени вливать, как того когда ты непосредственно что-то изучаешь / разбираешь / пытаешься решить / делаешь конспекты, так и того когда ты стоя в душе или находясь в очереди на маршрутку пытаешься отрефлексировать или как-то реструктурировать уже то, что ты знаешь. По-другому никак, думаю.

>> No.4964034  

>>4963265
Пример 4 можешь не разбирать если непонятно, это была некоторая подводка к определению конечной группы и к Лагранжу, тонкие детали всегда сложны и менее понятны, потому что завязаны на конкретные вычисления, а вычисления зачастую комментировать сложно.

>> No.4964310  

>>4964034

Спасибо! Я пока создал тред ещё >>4964046... жду Лагранжа, как/если захочешь!

>> No.4965214  

Вопрос ОПу: считаешь ли ты, что мнимые (или вымышленные, если по-английски) числа — это плохое название?

>> No.4965225  
Файл: cirnokk.jpg -(9 KB, 189x267, cirnokk.jpg)
9

>>4965214
Нет, думаю что хорошее, вообще люблю когда заигрывают с терминологией и не относятся к неймингу супер академично-научно серьезно, как минимум потому что запоминать легче и спутать тяжелее.

>> No.4965240  

>>4965225
Вроде же говорят, что их стали называть вымышленными, чтобы насмехаться над ними, не значит ли это, что это название оскорбительно?

На самом деле я встречал, кажется, не один раз утверждения, что вымышленные — плохое слово, потому что мнимые числа настолько же реальны, как и действительные или, как говорилось в других примерах, как и отрицательные. Я согласен с тем, что они настолько же реальны, но только наоборот, я считаю, что это остальные числа, помимо натуральных, тоже вымышленные. Но раз уж название сложилось, то хотя бы мнимым пусть будет оставлен вымышленный статус.

Я вообще проникся древними греками, которые разделяли числа, то есть натуральные числа, которые есть числа в исходном смысле этого слова, и отрезки и прочую геометрию. Натуральные числа естественны и реальны, может быть один протон или два протона, и гелий с двумя качественно отличается от водорода с одним. Точно так же реальны отрезки и прочие геометрические построения, потому что мы живём в 3+1-мерном пространстве-времени, которое подчиняется геометрическим законом.
Все остальные "числа" (кроме, возможно, отрицательных) возникли из-за попытки настоящие числа подружить с отрезками. С отрезками можно делать многое то, что и с числами, например, складывать. Также можно умножать отрезок на число, сложив его столько раз. Если мы говорим, что длина отрезка равна такому-то числу, то мы же на самом деле говорим, что если умножить эталонный отрезок на это число, то он будет равен данному отрезку. Делить отрезок на равные тоже можно, и таким образом у нас появляется первая искусственная конструкция из чисел, которую мы принимаем за число, — натуральная дробь. Ведь на самом деле, когда мы пишем a/b=x/y, мы имеем в виду, что a·y=x·b.

Пифагорейцы хотели подружить числа с отрезками тупым образом, используя только натуральные дроби, но у них не получилось, так что можно понять (хоть они и неправы и морально, и научно), почему они убили того беднягу, которым первым доказал несоизмеримость диагонали и стороны квадрата. Прошло ещё много веков, прежде, чем была придумана подходящая, но весьма сложная конструкция, которую мы теперь называем действительными числами, которая представляет из себя бесконечную последовательность натуральных дробей.

Где-то посередине были придуманы ещё и отрицательные числа, и они как раз были придуманы не из геометрии, а ради алгебраической симметрии, но по-факту-то это тоже придуманная конструкция. Везде, где в реальной жизни применяются отрицательные числа, они просто означают, что что-то там "в другую сторону". Заряд либо электрона, либо протона не является концептуально отрицательным, просто так уж повезло, что видов зарядов два, и поле действует на них силой, направленной в противоположную сторону. Значит мы можем приписать одному +, второму - и упростить вычисления. То, как вектор умножается на отрицательное число — это тоже просто сокращение для "сначала перевернуть, а потом уже умножить". Классический пример с долгами тоже туда же, реально никаких отрицательных денег-то нет.

Введение действительных чисел позволило наконец подружить геометрию с алгеброй, и красиво расположить все-все новоизобретённые "числа" на прямой. Но это всё обман, которым обманывают всех школьников, которым не говорят сначала о тех самых сложных конструкциях, которые за этими "числами" скрываются. И мало того, из современной математики полностью исключили те самые древнегреческие отрезки, полностью подменив их конструкциями из чисел. Да, они соотносятся один к одному, а числами и их конструкциями, то есть символами, оперировать гораздо удобнее. Но всё равно, обидно!
Натуральные — это, конечно, хорошее название для настоящих чисел, но я отныне отказываюсь считать все остальные числа по-настоящему настоящими.

P.S. Если кто-то скажет, что в реальности есть иррациональные числа, не являющиеся соотношениями отрезков, например, соотношение масс протона и электрона, то нет, даже если оно иррационально, то это всего лишь означает, что есть бесконечный класс бесконечных последовательность чисел x₁ᵢ, y₁ᵢ, x₂ᵢ, y₂ᵢ, удовлетворяющих каким-то сложным свойствам, такая, что если взять x₁ᵢ электронов, то они будут легче y₁ᵢ протонов, а если x₂ᵢ электронов, то легче y₂ᵢ протонов.
Массу мы не наблюдаем непосредственно, а вот отрезки времени и пространства наблюдаем, поэтому отрезки имеют больше моральных прав на то, чтобы сидеть на платоническом небе рядом с настоящими натуральными числами.

>> No.4965253  
Файл: cirnokt.jpg -(120 KB, 850x601, cirnokt.jpg)
120

>>4965240
Ну наверное, я просто как-то не ощущаю в себе внутреннего желания рассуждать об онтологическом статусе вещественных и натуральных чисел, поэтому думаю и школьникам рассказывать об этом идея плохая, ведь они этого желания тоже могут не ощущать. Почитай "Что такое число?" Кириллова, это небольшая брошюра на 40-50 страниц про основные конструкции всяких экзотических числовых систем в математике, может тебе покажется интересным.

>> No.4965400  

>>4965240

Понятия это "формочки" для мира (для нашей игры в песочнице/мире), которые мы выбираем. Можно выбирать формочки, основываясь на мнении других людей. Можно на основе своих принципов или интересов. Но можно и начать деградировать, вцепившись в идею, кажущуюся "простой", и начав отрезать всё что за её пределами (пропагандировать свои квадратно-гнездовые методы)

> То, как вектор умножается на отрицательное число — это тоже просто сокращение для "сначала перевернуть, а потом уже умножить". Классический пример с долгами тоже туда же, реально никаких отрицательных денег-то нет.

И любви тоже никакой нет, только "три движения тазом вперёд, потом два назад, повернуть/перевернуть..." — начать раскладывать на свои собственные понятия можно всё что угодно
>>4965253

У меня тред стал активным/накопил постов! Надеюсь тебя не мучает жизнь

> Ну наверное, я просто как-то не ощущаю в себе внутреннего желания рассуждать об онтологическом статусе вещественных и натуральных чисел, поэтому думаю и школьникам рассказывать об этом идея плохая, ведь они этого желания тоже могут не ощущать.

Но такой (>>4965240) школьник ощутил, и был даже не единственным в треде, целой прослойке приходящих сюда важна именно такая онтология, был вопрос "Объясни почему направления два" например, или онтологический вопрос о зацикленности математики...

Вывод о неважности онтологии может быть подобен ошибке выжившего: тем кто в математике "выжил" она обычно не важна, но есть куча невыживших, которым что-то надо было такое...

>> No.4965439  

>>4965400
Забавно, но про два направления писал я же. Про зацикленность уже кто-то другой начал, но я точно потом докинул сверху ещё вопросов, правда не помню уже, каких.

Раскладывать в голове понятия на составляющие крайне полезно. И в курсе матана, например, про те же действительные числа всё раскладывается понятно и однозначно.

Вопросы про реальность или нереальность чисел — это баковство и к реально-жизненной математики имеет не очень много отношения, но у нас тут всё-таки бака-тред или как.

Сейчас я пытаюсь изучать урывками философию и истории философии, когда изучу, тогда ещё раз попробую перечитать твои тексты, может тогда получится.

>> No.4965615  
Файл: 0B83 180px-Feynman_Drums.png -(22 KB, 180x178, 0B83 180px-Feynman_Drums.png)
22

>>4965439

Это не забавно, это чудо подтверждения теорий! Если немного закрыть глаза на факт, что я не делал предсказания. И что я по сути назвал (чуть ли не) вообще всё что было в этом треде и таким образом "попал"

Вообще, если задуматься, это подорвало (причём сразу в двух местах) мой политический посыл, что кому-то нужна эта онтология... кстати, ответ ОПа тогда тоже странный немного, получается он тоже серьёзно воспринял? иначе в чём смысл бака-треда если нет желания баковать

> Вопросы про реальность или нереальность чисел — это баковство и к реально-жизненной математики имеет не очень много отношения, но у нас тут всё-таки бака-тред или как.

Прости! Привык воспринимать все эти предложения абсолютно серьёзно.

> Сейчас я пытаюсь изучать урывками философию и истории философии, когда изучу, тогда ещё раз попробую перечитать твои тексты, может тогда получится.

Я сам считаю, что лучше много маленьких усилий (например, вопросов)(мало усилий, зато много), чем одно большое (и потом тишина).

>> No.4970424  

Бамп есть?... а если найду?

>> No.4970443  

>>4970424
Вот читаю этот тред, ну профессора сидят, аж страшно стает. Такие умы пропадают! Их надо бы в какой-нибудь Гарвард бампнуть. Надежда всей планеты!

>> No.4971199  

>>4970443

Имеешь в виду этот >>4958617 (судя по автарке. с подобной был один и как-то переспорил математика)? Я согласен что >>4958835 и >>4958617 это кандидаты.

Они даже про anti de sitter correspondence говорили. AdS/CFT correspondence... не знаю, что-то из этого!

>> No.4974930  
Файл: 598px-K33.png -(64 KB, 598x600, 598px-K33.png)
64

Для графов есть понятие планарности, например полный двудольный граф К 3,3 нельзя на плоскости изобразить без пересечения ребер. А есть ли понятие планарности для 3-х и более измерений? Ну например ребрами могут быть не прямые, а некие плоскости, а вершины графов будут допустим прямые, и такую "планарность" в трехмерном пространстве будет считаться как непересечение плоскостей, соединяющих вершины. Есть ли какое-то такое обобщение?

>> No.4976547  

>>4976541

>Чисто теоретически может, ибо у ребёнка есть половина генов каждого из родителей и при зачатии от него может перейти именно она

Ну если родители были почти идеальными клонами, чтоб все хромосомы кроме половых (аутосомы) были попарно одинаковыми, и с одинаковым митохондриальным ДНК (она наследуется только по женской линии) то тогда такие родители будут делать клон себя при размножении (если не будет случайных мутаций). Только такие организмы обычно нежизнеспособны, такая гомозиготность говорит о кровосмешении.

>> No.4976550  

>>4976547

> эдипов комплекс

Ну так речь-то идёт не о AA+BB=AA, а про AB+BB=AB, что вполне возможно

> Чисто теоретически

На практике, даже если вероятность отсечь и мутации отсечь, да, несильно осуществимо.

>> No.4977889  
Файл: cirnoq0.jpg -(129 KB, 700x1000, cirnoq0.jpg)
129

Хочу чтобы тред немножко ожил! Напоминаю что вопросы можно задавать самые глупые! Там про гипотезу Пуанкаре или деление на 0 или что-то такое!
>>4976530
На самом деле нотаций для обозначения регулярных замощений и прочих правильных геометрических конфигураций очень много, наверняка такая существует но я о ней не знаю. Попробую нагуглить, если получится - то напишу.
>>4976536
Теоретически, общая теория относительности не запрещает существование машин времени, так называемых "замкнутых времениподобных петель" которые возникают в решениях уравнений Эйнштейна называемых "вормхолами", но экспериментально никаких объектов которые ведут себя похожим образом не обнаружено, и из-за их фантастичности многие относятся к ним как к некоторому теоретическому артефакту, чем как к какому-то объекту который мог бы существовать и пока не обнаружен. Философский тезис, который утверждает что будущее не может влиять на прошлое называют "принцип причинности", иногда ссылаются на него чтобы "доказать" что вормхолов существовать не может.

>> No.4977922  

>>4905555
https://www.youtube.com/watch?v=xPzR_D9qKeo

>> No.4977928  
Файл: cirnoq1.jpeg -(21 KB, 268x268, cirnoq1.jpeg)
21

>>4977922
Смешно! Ну фигуры вроде "нужно наложить пару технических требований на функции в контексте чтобы всё стало правильным" и "заменить функцию аппроксимацией на каком-то этапе вычислений" терпеть вообще можно, но фигуры вроде "не совпадают тип операции и аргумента" как в случае с обращением неквадратных матриц - мне кажется уже нельзя, потому что просто перестаешь понимать историю которую тебе пытаются рассказать.

>> No.4977955  

>>4977928
Даже на лекциях по математике отступления от формализма встречаются постоянно. Потому что. Во-первых, в унике учат на специальность, а не заморачиваются строгостью. И во-вторых, учат идеям, а не их формальному воплощению.

>> No.4977969  
Файл: 77806430_p0.jpg -(1406 KB, 2041x2857, 77806430_p0.jpg)
1406

/sci/ слишком мёртвый, напишу здесь.

Существуют ли области науки, не затронутые машинным обучением?

Почему за столько лет существования всяких нейросетей и прочих работ по ИИ они только сейчас стали жутко популярны и востребованны чуть ли не на каждом шагу? Что вызвало этот всплеск?

>> No.4978006  
Файл: goat-739756_960_720.jpg -(172 KB, 960x640, goat-739756_960_720.jpg)
172

>>4977969
Мода. Один делает сенсацию, другие бросаются повторять - и иногда формируется целая индустрия. И для этого даже не нужно чего-то нового.
Это как яМобилко, который в принципе не привнёс ничего нового. Но задал модный тренд, отходы которого в виде гигантских лопат торчат отовсюду. Или Илон Маск со своей Теслой, который не сделал ничего для разрешения проблемы безопасности (Дешёвый аккумулятор пожаровзрывоопасен и токсичен, для защиты требуется броня, броня увеличивает массу автомобиля, размер аккумулятора приходится наращивать, что требует ещё большей брони).

>> No.4978009  
Файл: cirnoq2.jpeg -(6 KB, 186x271, cirnoq2.jpeg)
6

>>4977955
Спасибо! Буду знать теперь!
>>4977969
Думаю почти все не затронуты, в математике, например, есть несколько забавных казусов, как аналитические решения каких-то диффуров или замкнутую форму интегралов фейнмана нейронками находят, но это скорее забавности, имеющие мало теоретического и практического смысла.

>> No.4978014  

>>4977969
Бстроотвечу по второму: Так то вся база была заложена ещё в 70-е, и до сих пор на тех теориях и моделях и работает практически без модификаций. Но одно дело прогнать через перцептрон 100 картинок на i386-16МБ, и другое терабайты данных на i9-16ГБ.
Ну и общий объём индустрии вовлечение её экономику и количество специалистов на рынке тоже сыграло пинка.

>> No.4978071  
Файл: 0B107 eebcf9ae403dec53114c2aa4cdd0b1a5.jpg -(59 KB, 320x640, 0B107 eebcf9ae403dec53114c2aa4cdd0b1a5.jpg)
59

>>4978009

Я собираюсь напирать на статистическую проверку своей шахматной гипотезы. Уже обсуждал в Slack'е с рационалистами (с единственным оставшимся в диалоге со мной рационалистом то есть). Буду создавать темы на форумах, даже на соционическом. Буду рассказывать свою историю (кто я такой, откуда вылез), потому что для меня это последний крестовый поход с единственным возможным исходом. [буду рассказывать] Про стариков, про побеждающую время любовь... Всё будет происходить как в боевике. Возможно, жизнь сыграет мной "последний" гамбит ...

Я хочу узнать, могу ли положиться на тебя в том случае, если нечто (уже, само, даже без тебя!) пойдёт хорошо. Чтобы ты мог загнать мяч в лузу, если речь вдруг пойдёт о математике или физике. Донести последний снайперский выстрел.

Вот мой план:

1) Шахматная гипотеза доказана прямой статистикой. Открытие уровня Дарвина или Менделя или Менделеева совершено.

2) Открыта новая задача в области распознания (ИИ) и статистики. Обогащён список задач.

3) На основе специфических свойств найденной в шахматах статистики изобретают новые инженерные ИИ-решения и новые статистические методы. И последнее будет по-любому, даже если вся нужная математика уже есть, потому что подобной задачи никогда ещё не было. Также автоматически совершён прорыв в Stylometry и Writer invariant (кстати, это дополнительно обосновывает предыдущее утверждение, что таких задач ещё никогда не было)
https://en.wikipedia.org/wiki/Stylometry
https://en.wikipedia.org/wiki/Stylometry#Data_and_methods
https://en.wikipedia.org/wiki/Writer_invariant

4) Это откроет новую эру работы с информацией и идеями. Физика и математика и биология и всё остальное будут затронуты как минимум так же, как и шахматная культура.

5) Будет выход на общую теорию интеллекта и сознания. Если понять восприятие "шахматного ландшафта", можно понять любой "ландшафт" в мысли, от "ландшафта" настроения до "ландшафта" залежей памяти...

6) Происходит некое вторжение в физику. Потому что:

а) Снова сделано открытие типа Менделеевского. Как это может не заинтересовать учёных?

б) Физика и математика это частности происходящего в мышлении и частности приходящих в мозг идей. У меня есть общая информация о строительстве теорий (если моя верна).

в) Может быть немного натренировался в подоборе теорий во время создания своих теорйи. (в общем подборе теорий)

(пояснение б) Будет позже!

Мне интересно (хотел спросить), можешь ли ты вторгнуться где-то после удачи пунктов 1—3?

Чтобы реализовать сюжет "И тут из за угла выезжают наши танки"... а?

Объясняю шахматную гипотезу ещё раз:

Есть слой объективной реальности, это шахматная позиция и конкретное положение фигур на ней.

Над этим слоем формируется новый слой, который можно назвать, например, "текстурой" позиции. "Текстура", в отличии от реальной позиции, одинакова куда не посмотри. Новый слой выражает симметрию данных, новый слой воплощает "инвариант" (нечто, применимое не только к этой конкретной позиции или не разрушающее при определённых изменениях позиции)... (!) В нахождении инвариантов и симметрий суть мышления, любое свойство является симметрией или инвариантов, иначе оно мгновенно бы растворилось в реальности или других мыслях (или даже не могло бы быть выделено изначально).

Заметки: "текстура" не обязана быть чем-то визуальным, она может состоять из ощущений; слою объективной реальности не обязательно существовать, "реальность" может быть условно-выделяемой частью слоя, в который входят и реальность и "текстура" и всё остальное.

Текстуру можно условно разделить на части, все эти части будут подобны друг другу.

Существуют текстуры, которые соответствуют "стилям шахматистов", можно назвать их "цветами". Может быть много шахматистов одинаковых или похожих цветов, но шахматист не может менять свой цвет. Если сравнить два цвета, в них будут найдены взаимно-исключающие паттерны (или статистические перекосы в положении шахмат).

Сами "цвета" тоже образуют слой, который везде подобен (или обладает другой особой "симметрией"), "спектр". Спектр это особое пространство (слой = пространство) свойств, не-комбинаторное, где каждой точке соответствует реальный объект и у каждого свойства есть "хозяин"...

На прагматическом уровне: 1) можно угадывать шахматистов по [универсальным] паттернам в их позициях (удивительно) 2) эти паттерны обладают удивительными свойствами 3) эти паттерны обладают удивительной взаимосвязью, ты можешь "выводить" новые типы шахматистов из "спектра" (может быть для этого будет требоваться немного информации (как толчок), но всё же, ты будешь сразу угадывать важнейшие свойства нового игрока, встретив его, будешь угадывать "куда думать").

(Обобщение:)

"Цвет" не обязательно выводить из единственной позиции (это точно не так как цвет/образы позиций пришли на ум мне). То есть пространством ""объективной реальности"" может быть не одна позиция, а все позиции из партии.

"Пространством" вообще может быть что угодно, а вместо "всюду подобия" и "фрактальности" и "цветности" могут быть любые другие "симметрии". Не говоря уже о том что "симметрии" не обязаны обладать настолько идеальной и последовательной структурой, какая предполагается в шахматах!

Если "цвет" существует, значит он связан с "пространством" вариантов в мышлении и шахматах, из которого выбирает ходы человек (иначе его выборы не приводили бы к не меняющемуся "цвету"). И не только с пространством выборов именно в шахматах, вряд ли человек создаёт нечто настолько особое только для шахмат.

Поэтому я верю, что это общая теория о мышлении! Как минимум, (при верности гипотезы) это был бы первый в истории evidence по этому вопросу хоть для какой-то теории.

Сейчас теории о мышлении (по-моему) это описания конкретных пространств/симметрий без понимания, что важны общие механизмы. Думаю, в худшем случае это происходит и в физике. А в лучшем случае происходит что-то хорошее, но не осознаётся. Я в этой проблеме мог бы предложить особую версию "Бритвы Оккама" — когда "разуплотняешь" все свои понятия и смотришь, какое реальное знание или реальная мотивация остались (проверяешь, не время ли освободить какую-то идею от лишнего контекста)... (шахматная гипотеза иллюстрирует все эти процессы условного отказа от понятий)

Думаю, "идеологии" необходимы даже на уровне восприятия, и они войдут в науку. (это пример разуплотнения понятия "идеологии" и открытия нового потенциала!)

>> No.4978072  

>>4978009

Это финальная эмансипация!

>> No.4978148  

>>4978006
Лопаты — это офигенно. Не могу представить, как можно пользоваться маленьким экранчиком. И насчёт моды я бы не был так категоричен. Возможно, они только сейчас нормально развились или им только сейчас нашли применение. А с развитием облаков прикрутить нейросеть стало проще.

>> No.4978154  
Файл: .gif -(983 KB, 498x278, .gif)
983

>>4978009

>Спасибо! Буду знать теперь!

А, нет, это я не тебе написал! Я знаю, что ты знаешь!

>> No.4978155  

>>4978148
Лопаты убили планшеты точно так же как смартфоны убили в своё время КПК. Ты считаешь что это офигенно. Лично для тебя и таких как ты. Никто не спорит. Но давай взглянем на ситуацию трезво. Раньше были и лопаты и микрообмылки и звонилки всех возможных форм и наладонники и чего только не было на любой вкус и цвет. Можно было взять тот же планшет и позвонить с него. А можно было гальку-звонилку-слайдер и выйти с неё в интернет. Теперь только лопаты. Тебе нравится да, а все кому не нравится идут лесом. Только потому что в тех кому нравились лопаты оказалось больше денег. А конкретно у производителей лопат. Самое смешное, что именно самсунг до последнего делали обмылки, за что весе потребители его открыто-показательно гнобили. не хотим обмылки, хотим гнущиеся (не гибкие!) и ломающиеся, взрывающиеся и бьющиеся люфтящие скрипящие лопаты с острыми углами. Ты не ослышался, давно просят дистанционное самоуничтожение к телефонам прикрутить.

>> No.4978156  

>>4978155
Сейчас тоже есть звонилки.

>> No.4978159  

>>4978156
Есть. Вот прям совсем последнее время с ними всё не так ужасно как хотя бы год-два назад. Даже пара-тройка моделей раскладушек появилось. Одна из них даже лопата-раскладушка. Но у меня такое чувство, что все их один и тот же китайский нонейм клепает. А в магазины их завозят под разными шильдиками как сезонные коллекции китайской одежды на рынках. Ну и качество у них всех соответствующее. Пользоваться могут только идейные ненавистники смартфонов.

>> No.4978279  

Забыл (>>4978071) ссылку на лекцию Пенроуза как пример!

https://www.youtube.com/watch?v=hAWyex1GKRU

>> No.4978294  

>>4978155
Лопаты не убили планшеты - они убили ограниченное множество маленьких планшетов на андроиде с не самыми лучшими чипами. Лично я пользуюсь айпадом, и лопата айпад заменить не может, поскольку:

  1. Больше экран. 10-дюймовых лопат не появится в принципе.
  2. Лучше софт. Питон(в виде pythonist-ы), цивилизация(которая на андроид, насколько я знаю, не вышла), и клиенты гита как некоторые из примеров. Плюс айпадовый софт обычно написан именно под большой экран. Плюс последняя система, которая по умолчанию дает десктопные сайты. Плюс адблок на системный браузер.
  3. Лучше оптимизация. Как пример: мой iPad Pro с 4 ГБ оперативки рвет Surface того же года даже в топовой комплектации с 16 ГБ оперативки и i7.
>> No.4978296  

>>4978159
Нокия недавно выпустила пару звонилок, причем даже с интернетом и магазином приложений. Надеюсь, когда-нибудь на той же ОСи сделают QWERTY-телефон...

>> No.4978417  
Файл: cirnoq4.jpg -(56 KB, 477x600, cirnoq4.jpg)
56

>>4978071
Про то что нужно разуплотнять конкретные факты и смотреть на общую картину - сказал очень правильно! Но про то что какая-то (нематематическая) идеология может как-то слишком сильно встрепенуть математическое сообщество довольно сомнительно звучит, математика и физика уже довольно давно "идеологически независимые" деятельности. И математики с очень большим скепсисом относятся к текстам в которых не доказывается теорем. Но я могу попробовать подкинуть каких-то математических метафор-примеров на какие-то твои идеологические философские конструкции, это всегда довольно эффектно на собеседников действует. К тому же я похожей деятельностью занимаюсь сам для себя когда читаю французских постструктуралистов энивей. Надеюсь что у тебя что-то выйдет!

С каким-то стат. аппаратом вряд ли помочь смогу, так как геометр а не статистик, разве что с каким-то самым примитивным.

>> No.4978456  
Файл: 0B112 Putanica.png -(283 KB, 989x1024, 0B112 Putanica.png)
283

>>4978417

> Но про то что какая-то (нематематическая) идеология может как-то слишком сильно встрепенуть математическое сообщество довольно сомнительно звучит, математика и физика уже довольно давно "идеологически независимые" деятельности. И математики с очень большим скепсисом относятся к текстам в которых не доказывается теорем.

Нет, нет, это всё вообще имеет смысл только в случае доказательства шахматной гипотезы!

Я когда-либо снова напишу только если что-нибудь получится.

А ты можешь помочь понятно описать саму гипотезу? Три вещи надо подчеркнуть: 1) есть типы игроков (речь не совсем об угадывании конкретных людей, хотя в определённых рамках об этом) 2) паттерны типов (а ведь надо ещё и как-то к этому перейти!) довольно общие и абстрактные 3) паттерны разных типов взаимосвязаны (это спектр, а не камушки разной формы, которые могут быть какими угодно вообще).

Вроде всё элементарно (и даже не ново), но я путаюсь. Как сказать что игрока можно узнавать по позиции, например? "Игрок определённого типа достигает типичных для своего типа [позиций]..." уже ужасно! А ведь надо ещё на всякий случай быть способным сделать уточнение, что может и нельзя вот так вот брать и угадывать по позиции во всех случаях, но можно понять какие-то "статистические перекосы" (как это сказать?) в положении фигур по сравнению с другим типом...

> С каким-то стат. аппаратом вряд ли помочь смогу, так как геометр а не статистик, разве что с каким-то самым примитивным.

А вот с такой ситуацией?: есть море разноцветных шариков. Когда шарик играет с другим шариком в шахматы, они окрашивают доску в свои цвета, эти цвета немного смешиваются воедино. Цвета смешиваются не идеально (? обычно всё-таки можно узнать оба шарика, точнее сказать их черты не растворяются в друг друге абсолютно хотя это всё ещё зависит от перспективы, т.е. определения и способа распознания "цвета", наверное!..) и сами цвета порой не идеальны (не "чисты"). (При этом и сама позиция наверное может вносить какие-то "помехи".)

Надо в связи с этим передать такие моменты:
Как в такой ситуации узнавать цвет шарика?
Хорошо бы сравнивать наиболее (субъективно для угадывателя/анализатора?) отличающиеся цвета.
Надо сравнивать именно конкретную пару игроков, из-за того что типы универсальны.
Если берём упрощённый критерий сравнения, не представляющий собой чёрный ящик (-), то надо выбирать достаточно отличающуюся по этому критерию пару!
Для понимание цвета, может оказаться, требуются достаточно специфические условия (по крайней мере на каком-то уровне этого "мастерства"). Из-за всех этих нюансов с оттенками неидеальными. Или может можно сравнить это с восприятием цвета в реальном мире, где всякие нюансы типа освещения и т.д. могут играть роль — в реальных ситуациях это не приводит к большим проблемам, но при всяких тестах эти тараканы могут полезть (или при попытках взять случайного человека и быстро увидеть цвет его партий, изучив пару). Если бы все так воспринимали, особых проблем бы не было, но когда нужно именно доказательство выудить из потенциально худших примеров...

(-) Это тоже надо выразить: некий идеальный универсальный критерий различия цветов невыразим ("бесконечно" детализирован), но можно упрощать его, получая менее универсальные (сами по себе) критерии...

И можешь ли объяснить ссылку ниже?
https://www.quantamagazine.org/a-power-law-keeps-the-brains-perceptions-balanced-20191022
https://en.wikipedia.org/wiki/Coastline_paradox

Разноцветные шарики это "светлячки", и надо поймать хотя бы пару в банку:
https://www.youtube.com/watch?v=psuRGfAaju4
https://www.amalgama-lab.com/songs/o/owl_city/fireflies.html

Чтобы доказать, что всё это не сон! Своей любимой и себе даже доказал это (достаточно сильно) в тестах, которые она для меня делала (не шахматист). Но методика не отработана и никому не будет интересно её отрабатывать... и прямая проверка (статистикой) лучше, думаю, доказать объективные и доступные (не чёрно-ящиковые) факты...

P.S. Может в тему ещё всякие такие штуки, подобные запары у биологов:
https://ru.wikipedia.org/wiki/Законы_Менделя
Шарики за роликик!

>> No.4978457  
Файл: 1562892134978.png -(241 KB, 586x800, 1562892134978.png)
241

>>4978456
Да у тебя же ГМО на картинке!

>> No.4978458  

>>4978457

Блина!!! Не думал, что сциентистская пропаганда могла проникнуть так глубоко. Хотя никогда не доверял википедии...

Пойду помою свои гены с мылом! Прочищу (от) гаметы.

>> No.4978660  
Файл: cirnoq6.jpeg -(9 KB, 225x225, cirnoq6.jpeg)
9

>>4978456
Может всё-таки на языке задач кластеризации? Игры - это объекты, кластеры - это типы игроков, различные метрики - это паттерны разных типов игроков. Тогда гипотеза в том, что существует универсальное разбиение на кластеры множества игр которое будет "хорошо работать" для любого "хорошего критерия". "Хорошо работать" можно формализовать количественно запросто, а вот как формализовать "хороший критерий" не очень понятно. Кстати, это https://www.researchgate.net/publication/305808081_Classifying_Chess_Players_with_Fuzzy_Clustering_Analysis_in_Fuzzy_Data_Using_Eco_Codes ты конечно же уже находил?

>> No.4978670  
Файл: cirnoq7.jpeg -(7 KB, 198x254, cirnoq7.jpeg)
7

Кстати, нашёл недавно очень милый чат с нативным Латехом и без регистрации, хочу потестить, добавляйтесь если кто хочет более живого общения: https://hack.chat/?math

>> No.4978776  
Файл: 0B113 depositphotos_103149346-stock-phot(...).jpg -(90 KB, 1024x1024, 0B113 depositphotos_103149346-stock-phot(...).jpg)
90

>>4978660

> Может всё-таки на языке задач кластеризации?

У меня мандраж от того, что ты подразумеваешь будто я знаю этот язык или не слушаешь меня. Если ничего не получится, у меня съедят/сломают глаза, ноги-руки, сердце и мозг. Если бы я знал, я бы сказал! Спасибо за твою версию. (Мне страшного от того, что от этого зависит многое)

В твоём описании не очень понял, где вводится понятие "хорошего критерия" (что это?). Формализовать не надо, только объяснить! "Объектом" в пределе может быть одна единственная миттельшпильная позиция из партии.

1) И как сказать, что метрики (паттерны) взаимосвязаны? Как спектр (можно плавно/"одинаково" переходить от паттерна к паттерну). 2) Где ты описал всё что я рассказал о смешениях? (:)

Просто тут отличие от задачи классификации кошек и собак, например, потому что тут не бывает собаки отдельно от кошки. Любой объект (партия/позиция) это смесь кошки и собаки. Плюс ещё, возможно, смесь со спецификой самой позиции, не связанной с шахматистами (хотя это всё может зависить от свойств/силы распознавателя).

(Кстати, почему объекты не игроки?)

И из этого всякие нюансы и интересности могут вытекать! Например, ты можешь угадать кого-то неправильно (игрока А), но "извинить" свою ошибку, если взамен дашь предсказание о свойствах позиций игрока Б, который помешал угадать. Могут быть всякие сложные предсказания.

Ну и всё что я говорил про сравнение именно пар игроков разного типа И (пока/если угадыватель не идеален) сравнение пар особенно отличающихся игроков.

(Ещё раз,) Отличия от классификации кошек и собак и пингвинов такие:
1) Никогда не можем наблюдать чистую кошку или чистую собаку или чистого пингвина. (Хотя это может зависеть от определений или чего-то ещё, это не высеченная в камне аксиома)
2) Кошки и собаки и пингвины отличаются друг от друга одинаково/равномерно, образуют "спектр". Поняв кошку, ты можешь предсказать существование собаки и пингвина ИЛИ как минимум быстрее выучивать их. (Они в каком-то смысле подобны, биологически-голомолгичны или даже ещё сильнее связаны)
3) Пингвин везде самоподобен всеми своими свойствами, представляет собой что-то типа текстуры. (>>4978071)

> Кстати, это https://www.researchgate.net/publication/305808081_Classifying_Chess_Players_with_Fuzzy_Clustering_Analysis_in_Fuzzy_Data_Using_Eco_Codes ты конечно же уже находил?

Не искал и не знаю, как искать! Мне было достаточно общих наблюдений шахматного сообщества, то что я спросил на Реддите и недавно ещё статьи на википедии читал, где ничего такого не было. Мне было достаточно, что мир не потрясло какое-то супер-открытие. Увидев это мысленно выругался, но опасности нет:

> In general there are 500 different types of chess openings.

Это просто про дебюты.

> Some openings are aggressive and some are defensive.
> They are; aggressive player, defensive player or positional player.

Это про "бытовые" понятия. И ни к каким удивительным предсказаниям не ведёт, если они вообще предсказывают хоть что-то.

Дальше не читал (не понял бы всё равно).

>> No.4979515  

>>4979508
А ещё бывают забавности, когда слишком умный парсер пытается во много систем счисления https://forums.docker.com/t/docker-failing-to-correctly-process-valid-yaml-config/183/2
Или, например, шелл-скрипт, который перестаёт работать в августе, потому что 08 не является валидным числом (а 07 является).
(Сомневаюсь, что тут что-то хотя бы отдалённо похожее, просто бакую.)

>> No.4979585  
Файл: 640px-Hilbert's_Hotel.png -(46 KB, 640x302, 640px-Hilbert's_Hotel.png)
46

>>4979277

>Но обычно когда нечто состоит из кусков разной размерности, то совокупная размерность определяется как максимум кусков.

Ну а если максимума нет, но при этом в любой точке n-мерного пространства оно конечномерно? Считать его бесконечномерным?
И кстати, можно ли придумать такое двумерное пространство с неевклидовой геометрией, которое было б изоморфно трехмерному пространству с евклидовой геометрией? Хотя... Под изоморфизмом, как я понял, понимается взаимно однозначное соответствие одного другому (биекция) - тогда похоже что одномерное пространство изоморфно сколь-угодно-мерному пространству, можно ж в одномерную координату упаковать сколько угодно координат. Т.е. допустим координату (x=1.234, y=5.678) в 2D можно соотнести с координатой (1.5263748) т.е. первая десятичная цифра одномерной координаты будет первой цифрой X координаты, вторая десятичная цифра одномерной координаты будет первой цифрой Y координаты, потом третья цифра это вторая по X и так далее. В вики https://ru.wikipedia.org/wiki/Бесконечномерное_пространство - сказано "Бесконечномерное пространство не изоморфно никакому конечномерному" - но вообще-то и для бесконечномерного случая я могу разбросать циферки от бесконечности размерностей в одну размерность (например через простые числа - цифры первого измерения будут занимать позиции, кратные 2, циферки второго - позиции кратные 3, циферки второго - позиции кратные 5, короче Гостиница Гильберта) - тогда что это вообще значит?

>> No.4979589  

>>4979585
Ты описал просто биекцию, а изоморфизм — частный случай биекции, который должен ещё сохранять какую-либо структуру (в зависимости от того, что именно изоморфируем).

>> No.4979595  

>>4979589
Что именно значит "сохранение структуры" если мы изоморфируем одно n-мерное пространство другому n-мерному?

>> No.4979596  
Файл: cirno q10.jpeg -(6 KB, 284x177, cirno q10.jpeg)
6

>>4979585
Ты говоришь о теоретико-множественном изоморфизме, а в статье говорят о линейном изоморфизме, поэтому противоречия нету.

>Ну а если максимума нет, но при этом в любой точке n-мерного пространства оно конечномерно? Считать его бесконечномерным?

Ну да, можно сказать что оно локально конечномерное, чтобы подчеркнуть что оно конечномерное в небольшой окрестности каждой точки.

>И кстати, можно ли придумать такое двумерное пространство с неевклидовой геометрией, которое было б изоморфно трехмерному пространству с евклидовой геометрией?

Если понимать буквально, то нет.

>> No.4979748  

Как находят магические квадраты?
Там ведь n^2 неизвестных и 2n+2 уравнений.

>> No.4981056  
Файл: cirno15.jpeg -(15 KB, 210x240, cirno15.jpeg)
15

>>4979497
Вообще это интересно, как чисто формально можно было бы различать "более абстрактные" и "менее абстрактные" статистические гипотезы, хотя думаю ты и не о том спрашивал.

>> No.4981676  

>>4981056

А почему ты не подумал о проблеме с шариками? Это же теорию множеств можно вспомнить просто, наверное. RGB-модель цветов. Шифрование, может быть.

Или квантовую механику, где подобные "проблемы"/тесты со стастистикой были (со всякими "пирогами" и "парами обуви"). Или биологию (ссылку дал на Менделя), где надо было статистикой из комбинаторного смешения вывести существует или не существует паттерн.

>> No.4982361  

>>4981676
Да, про квантовую механику тоже подумал, так как там как раз такой mixed сеттинг но как конкретно всё что ты хочешь сказать впихнуть в сеттинг не могу сходу придумать.

>> No.4982362  
Файл: cirno16.jpeg -(10 KB, 300x168, cirno16.jpeg)
10

Сырна отвалилась

>> No.4982370  

>>4982361

Прости, просто я совсем глупенький (совсем с логикой не в ладах) — мне в этой ситуации неочевидно даже самое очевидное. Например, можно ли вообще (и при каких условиях) узнать цвет шарика, если смешивание идеальное.

Хотя ладно, пофигу на всё это! Зря тему тащил, прости.

>> No.4983255  
Файл: cirno18.jpg -(539 KB, 860x1214, cirno18.jpg)
539

Заходите в https://hack.chat/?math а то там пустовато.
>>4982370
Ну я не знаю, я думал что ответы на такие вопросы должны как-то следовать из твоей концепции, я лишь какую-то интересную формализацию хотел подобрать.

>> No.4983353  

>>4983255

Ну, я не математик! Не знаю, какую аналогию провести (почему не вытекает), знаю только то что вижу и что сам бы делал (своим мозгом, а не слабой моделью, которой надо устранять помехи).

Если бы мог легко оперировать в уме конструкциями типа "вычеты из кольца образуют тело, сумма которых образует 'прапорщика' и полностью ложится в топологию пересечений стрелки", то мог бы идти в теорию чисел или топологию или комбинаторику или абстрактную алгебру или теорию формальных грамматик или ещё куда...

Ничего не считаю. Заглянуть на 1.5 шага вперёд уже больно.

Нашёл:
https://ru.wikipedia.org/wiki/Центральная_предельная_теорема
https://medium.com/nuances-of-programming/заставляем-глубокие-нейронные-сети-рисовать-чтобы-понять-как-они-работают-12d051f5081f
Там некая теорема всплыла якобы из-за того, что вся картина становилась серой в каждом пикселе.

>> No.4984049  

бамп

>> No.4984076  

https://ru.wikipedia.org/wiki/Небесная_магия
Это ваша истинная цель?

>> No.4984623  
Файл: cirno20.jpeg -(11 KB, 191x263, cirno20.jpeg)
11

>>4984076
Забавно, не видел этого раньше!

>> No.4984675  

>>4984076
Мне больше вот это понравилось https://ru.wikipedia.org/wiki/Естественная_магия

>Тогдашнее определение естественной магии включало дисциплины, которые в настоящее время признаны отраслями естественных наук.
>> No.4984725  

>>4984623

Прости, так вышло что ты единственный человек во вселенной (кроме моей любимой), который мне отвечал с закономерностью/гарантией. Мой друг детства слабо реагировал на мои мольбы и недавно просто удалил аккаунт и всё (а другой связи с ним просто не существует, кстати), так что сам можешь оценить — по таким причинам ты и единственный... Я не доставал его так же, как тебя! В последний раз просто сам помог/ответил ему по невинному вопросу, а на следующий день акк уже был удалён

Мог ли бы/согласился ли бы как-то помочь? Точно не можешь сравнить идею с архитектурами сетей, там всё на языке векторов? А сравнить идею со "спиновым стеклом", если это имеет смысл?

Мой тред: >>4984546 (там есть даже как я выгляжу)

Я надеялся, что экстренная ситуация хотя бы однажды даст талон на одно серьёзное обсуждение... (и она экстренная по многим фронтам)

>> No.4985163  
Файл: 1__jDTWlZNUySwrRBgVNqoNw.png -(256 KB, 1320x760, 1__jDTWlZNUySwrRBgVNqoNw.png)
256

Математик-кун, а ты умеешь jupyter? Пользуешься?

>> No.4985229  

Чирно. А что если скорость света это "бесконечность" скорости?
А что если при ускорения, график время-скорость лежит не на плоскости, а на цилиндре с замкнутый осью скоростей?
Что произойдёт с телом в такой ситуации если превысить скорость света?

>> No.4985280  

>>4985229

>график время-скорость лежит не на плоскости, а на цилиндре с замкнутый осью скоростей.

Что.

>Что произойдёт с телом в такой ситуации если превысить скорость света

Вопрос не корректный, превысить скорость света невозможно.

>> No.4985282  

>>4985280

> Вопрос не корректный, превысить скорость света невозможно.

Возможно с отрицательной массой. Это во-первых.

Во-вторых, свет замедляется в разных средах и потому, теоретически, его может там что-то и обогнать.

В-третьих, абсолютная скорость это скорость причинности (speed of causality). Однако из-за того, что фотон обладает нулевой массой, то в полном вакууме скорость фотона бесконечно близко приближена к скорости причинности. Плюс говорить "скорость света" намного проще и привычнее.

>> No.4985328  

>>4985282
Так что будет?
А что будет если двигаться медленнее 0?

>> No.4985334  

>>4985328
Двигаться медленнее ноля это просто двигаться в другую сторону. Если же меньше нуля по модулю - то такое невозможно, поскольку 0 это самое маленькое значение из возможных по модулю. И если какое-то значение будет меньше - оно и будет новым нулём.

> Так что будет?

Если иметь отрицательную массу? Много чего может быть, вплоть до путешествия этой частицы назад во времени (т.е. секунды для неё будут течь в обратном направлении). Зависит от многих параметров.

>> No.4985340  

>>4985334
А если иметь положительную массу?

>> No.4985345  

Есть мнение, что эфир это и есть фотоны без движения, а значит с нулевой массой. А значит они не существуют.

>> No.4985346  

>>4985340
Тогда всё будет как обычно.

>> No.4985348  

>>4985345
Эфирные масла втирают кожу, значит существуют.

>> No.4985939  
Файл: cirno21.jpeg -(7 KB, 210x240, cirno21.jpeg)
7

>>4984725
Да расслабься, просто не всегда выходит треду время уделять, так-то я довольно серьезно к нашей дискуссии подхожу, просто мне кажется у меня недостаточно высокий уровень чтобы как-то хорошо твои принципы заформализовать, ну по-крайней мере мне кроме каких-то мутных ассоциаций, которые к тому же были названы (случайные величины, квантовая механика, задачи кластеризации) ничего в голову не пришло. Тред почитаю.
>>4985229
Ну, траектория на цилиндре это всё равно что траектория с периодическими условиями на скорость, что не очень интересно. Скорость света это и есть "бесконечная скорость" в том смысле, что при с \to \infty формулы СТО переходят в формулы ньютоновской механики, то есть это действительно аналог "бесконечной скорости" в СТО.
>>4985163
Нет, я слишком абстрактными штуками занимаюсь чтобы это полезно мне было.
>>4985345
Интересное мнение!

>> No.4986098  
Файл: сСhessobuch215 Fisher 1.png -(36 KB, 476x476, сСhessobuch215 Fisher 1.png)
36

>>4985939

Расслабил булки, чтобы было проще всплывать или ...

> ну по-крайней мере мне кроме каких-то мутных ассоциаций, которые к тому же были названы (случайные величины, квантовая механика, задачи кластеризации) ничего в голову не пришло. Тред почитаю.

Может не стоит пока читать, если что! Сделаю тему на Киберфоруме и там напишу короче и с примером.

Спасибо за серьёзное отношение! Может использовать мозговой штурм и не обязательно с целью провести именно чёткую аналогию? Просто собрать сводки какой-то общей инфы, или может я сам могу поделиться какой-то важной (в общем, как детективы сработать, где нет мгновенного прыжка к ответу). Шажок за шажочком...

Я вот узнал, что такое тензоры (хотя бы для своего уровня)! Пара стрелочек, которые могут означать вообще разные вещи.

Вот мои мысли о зацепках и том что дано: (спрашивай, если что, не помню что рассказал а что нет)

1) Любая мысль или осознаваемое событие это "(а)симметрия" в некотором "пространстве". Математические и физические формализмы это частный случай (того что вообще можно подумать)(Кэп).

На примере банальнейших вещей: диагональ делит прямоугольник пополам (это симметрия-пространство), угол движется по полукругу и всегда является прямым (это симметрия-пространство)...

%%"Пространство" вообще неотделимо от "симметрии" (пространство = симметрия) и любое действие или последствие действия это тоже (а)симметрия. Например, возведение в квадрат [действие] это тоже симметрия и то что оно делает неважным знак (плюс/минус) числа это тоже симметрия [последствие]%%

Суть терминологии в том, чтобы описывать самые разные вещи максимально похоже друг на друга для максимально честного и подробного сравнения.

2) (На тему доказательств, аргументации и прочего) "Обосновать" или "формализовать" какую-то симметрию — найти ещё одну симметрию, похожую на первую, в каком-то другом пространстве. Например, в другой области знания (или "проблеме"). Читал статью на Лурке о рождении понятия "энтропия" и его [понятия] связывании с идеей об "информации" (история такой формализации - одна из "улик")

Как понял, сначала была выражена какая-то симметрия движения тепла в кучах тел (Клаузис). Затем симметрия "движения" в пространстве возможностей (разбивающаяся кружка, состояния, степени свободы)(Больцман и Планк). Потом в статье "Демон Максвелла" и циклы Карно (тоже симметрии). Потом информация, "информация" символа это его симметрия в пространстве последовательности и "алфавита" (логарифм вероятности появления, основание логарифма — алфавит). Потом НАД этой симметрией вводится другая (описывающая поток информации или источник)... и эта симметрия "совпадает" с симметрией энтропии (скрещенивание областей/проблем завершается)

Потом фон Нейман и Ландауэр описали "симметрии" потери и приобретения тепла при записи и стирании информации.

Всякие симметрии могут быть и в теории течения потоков:
https://en.wikipedia.org/wiki/Turbulence#Kolmogorov's_theory_of_1941

Или в описании спиноров: (как и чего угодно)
https://en.wikipedia.org/wiki/Spinor
https://en.wikipedia.org/wiki/Plate_trick

3) (про 1-ый пункт) Моя идея про то же: про симметрии/пространства, которые называю "цветами", и симметрию/пространство этих симметрий/пространств внутри "спектра цветов".

"Цвет" ещё связан с представлением позиции в виде везде одинаковой "текстуры", и эта одинаковость тоже "симметрия". И способ получать такую одинаковость это тоже какая-то "симметрия":

Если представить, что информация о каждой области позиции (типа "Что во мне? что поблизости от меня?") описывается какими-то векторами/тензорами (или чем-то типа того; тензоры просто вдохновили), то способ перемешивать информацию между этими векторами/тензорами — это текстура/цвет/спектр.

4) Всякие сильные/слабые/электромагнитные взаимодействия это тоже симметрии. Например, слабое взаимодействие позволяет частицам обмениваться "штуками". Значит, тут должна быть хотя бы общая аналогия с обменивающимися чем-то областями доски (выше). "Штуки" это тоже симметрии.
https://ru.wikipedia.org/wiki/Слабое_взаимодействие#Свойства

5) Всякие теории это тоже про симметрии в каких-то пространствах.

6),

Значит, по-моему, чтобы нащупать что-то надо:

1) Выписать "пространства" новой идеи.
2) Выписать "пространства" из уже известных идей.
3) Подумать, где может быть пересечение. (Или мысленный эксперимент в духе "Что было бы/добавилось бы, будь эти вещи одним и тем же")

Ты можешь у меня спросить по 1-у, а я по 2-у.

Вот что "я" имею:

0) Пространство всех возможных шахматных позиций. А в нём пространство позиций, которых могут достичь люди. Гипотеза предполагает, что оно делится людьми на регионы и каждый живёт только в своём регионе. А встречаются люди только на неких "пересечениях" регионов. Человек движется по пространству так, чтобы сохранять "цвет", и это ещё больше сокращает количество позиций в которых он может оказаться (помимо исключения "неадекватных" и слишком сложных "компьютерных" поз). Возможны маршруты, не сохраняющие цвет, но по ним не ходят. НО в самом начале цвета нет и в глубочайших эндшпилях цвет неизбежно стирается. (Хотя это ещё может зависеть от способа определения цвета! И цвет можно использовать для попытки предсказать, какие дебюты использует шахматист и [может быть] в каких глубоких эндшпилях оказывается)

1) Отдельные области доски и комбинации фигур и пешек и пустот в них.

2) "Текстуры", описывающие симметрии для комбинаций и позиций. (Пространство "текстуры")

3) "Цвета", описывающие симметрии для всего выше. (маршрутов, регионов, текстур, комбинаций и позиций)(Пространство какого-то конкретного "цвета")

4) "Спектр", описывающий [ещё] симметрии для всего выше. (Пространство "цветов")

(Ещё замечу, что цвета, теоретически, могут на 100% быть просто отношениями, а не сущностями, то есть не быть закреплены ни за какими конкретными объектами реальности. Функциями, которые до поры до времени вообще неизвестно что значат.)

Навело ли на какие-то мысли описание симметриями и пространствами? Могли бы помочь уточнения про симметрии из какого-то из пунктов? Я вот подумал, что какая-то связь может быть даже с энтропией/информацией ("цвет" это тоже макро-состояние, но оно так просто не разрушается). Или идею фантастического рассказа о том, как эволюция создала для [какого-то рода] существ комплементарные инстинкты (где один боится, идёт другой, и наоборот).

Вот ты упоминал "критерии"(метрики?), например. А бывают ли критерии критериев? Какие есть исторические прецеденты (что можно узнать из истории)? Что я бы мог уточнить со своей стороны?

Думаю, возможен хотя бы такой "штурм":

Как математически описываются факты о самом обычном цвете?

Если цвет это тензор/вектор/ещё что-то, то что такое "спектр"?

Почему комплексные функции разноцветные/про что они?

Если цвет это [какая-то концепция из физики], то что такое спектр? Например, в физике есть концепция супер-симметрии (для частиц), которая вроде бы несёт похожую функцию: изобразить пространство, внутри которого каждая штучка соответствует чему-то реально существующему + сгруппировать все существующие типы чего-то в единое пространство.

...

Разве мы не должны знать хотя бы что-то о том, как выглядел бы формализм? +Идея фрактальности...

>> No.4986099  

>>4946080
Ого,как он смог решить? 1+1,даже я не знаю

>> No.4988599  
Файл: сСhessobuch230 Graph 2.png -(594 KB, 1885x2681, сСhessobuch230 Graph 2.png)
594

>>4985939, Прости, если что, какие-то вещи пишу в формате мозгового штурма, но какие-то серьёзны на 1000 процентов

У меня появилась идея! Когда мы изучаем "цвет" шахматиста X, то его противники — это наши измерительные приборы. А отсюда связь как минимум с этим:

> Часто тензоры используются для описания физических явлений, параметры которых могут отличаться для разных наблюдателей, которые могут использовать для измерений разные приборы с разными шкалами, разными единицами мер.

https://ru.wikipedia.org/wiki/Тензор#Пример

"Измерительными приборами" могут быть и всякие понятия и критерии, которыми можно пытаться выразить "цвет".

И "прибор" в виде соперника ещё как влияет на то, что измеряет. Если играют два цвета, получаются разные комбинации этих цветов (здесь может быть более прямая связь с квантовой механикой? типа "комбинации базисов": А, Б, А + Б, А - Б).
https://www.youtube.com/watch?v=SEcdJMhfFFQ

Вот мой сокращённый (и где-то обогащённый?) тред на Кибере, там много аналогий "на грани" математики:
http://www.cyberforum.ru/ai/thread2547374.html

> Наряду со скалярными и векторными величинами встречаются во многих вопросах физики и геометрии величины более сложного строения. Эти величины, называемые тензорными, описываются в каждой системе координат несколькими числами (компонентами тензора), причём закон преобразования этих чисел при переходе от одной системы координат к другой более сложен, чем для векторов (точные определения будут даны ниже).

https://dic.academic.ru/dic.nsf/bse/138986/Тензорное

Я мысленно разбиваю позицию на условные области и каждая область сообщает мне какую-то информацию. Например "В какую сторону отсюда больше фигур?" "Какого типа здесь фигуры?"... Порой эту информацию можно выразить стрелочками. Значит, это что-то похожее на тензор или тензорное поле? "Условность" разбиения означает, что информация перемешивается, показания одной стрелки влияют на показания другой (суть в том, чтобы показания были "одинаковые": это даёт представление позиции как случайного набора фигур в виде "текстуры", чего-то всюду одинакового).

Почему мы не можем сказать хоть что-то о будущем формализме?

Например, что это будет каким-то пространством, не важно, метрик или не метрик или чего. Нельзя мазок за мазком пытаться нарисовать, задавая какие-то вопросы по пути?

И что оно будет как-то связано с другими пространствами. Например, с пространством всех возможных позиций. И так дойдём до чего-нибудь...

Вот ещё какая идея для мозгового штурма:

Берём какую-то математическую концепцию или проблему и думаем, какие хорошие свойства или "решения" для неё могли быть дать "цвета"/"спектр". (решение может быть не совсем "настоящим": например, людские (и не только) шахматы это вряд ли игра эксоненциальной сложности, люди и нейросети и движки её "упростили", хоть и не совсем "по-настоящему")

Например, может быть тензор или тензорное поле в общем случае не обладает какими-то свойствами, а именно эти тензоры/именно это поле — обладает.

А может быть "цвета" обладают какими-то хорошими свойствами с т.з. хранения и сжатия информации? (Даже если они "не совсем" сжимают информацию: видение цвета не позволяет мне прямо воссоздавать позиции.) То что я описал про места это же тоже своего рода рекурсивное описание.
https://ru.wikipedia.org/wiki/Колмогоровская_сложность

Является ли "текстура" аналогом трансформации данных, типа Фурье или Хаара? Просто считаю, что всё мышление можно представить последовательностью/наслоением "текстур" (об этом в теме).
https://en.wikipedia.org/wiki/Haar_wavelet

> "Цвет" это бесконечный список/бесконечная сеть понятий (симметрий), "спектр" — тоже. (это из темы на Кибере ещё)

Просто вместо синусов складываются любые "симметрии" в каком-то пространстве мысли.

Возможно, могут быть аналогии с петлевой квантовой гравитацией или с диаграммами Фейнмана.

Или как-то с вероятностью сравнить! Например, гипотеза утверждает, что при определённом подходе к формулированию свойств объектов наверняка найдётся какое-то отличие между любыми двумя их свойствами или любыми двумя позициями:
https://ru.wikipedia.org/wiki/Асимптотически_достоверное_событие

Могут ли "цвета" или их обобщение помочь в какой-нибудь теории о хаотических процессах? Ведь шахматная партия тоже в каком-то смысле хаос. А тут вдруг оказываются, что существует такой строгий инвариант как "цвет".
https://en.wikipedia.org/wiki/Turbulence#Kolmogorov's_theory_of_1941

Кстати, раз любая человеческая шахматная позиция получается смешиванием двух "цветов" в какой-то пропорции, значит "цвета" в каком-то смысле и есть аналог тех синусов? "Цвет" это что-то типа функции, указывающий, к каким состояниям позиция будет стремиться и в каких дольше задерживаться.

А знаешь ли ты генетику, что дала бы аналогия с ней? "Понятно", как передаются отдельные качества, а как могли бы передаваться такие вещи как "спектры" качеств? Или спектр образуется и поддерживается сам собой?.. А если провести аналогию с компьютерной генетикой, генетическими алгоритмами?

Твоя помощь очень нужна!

>> No.4990390  

Ычан, а давай попробуем определить числа.

Вот есть у нас какой-то абстрактный тип данных, какие операции он должен поддерживать, чтобы считаться чиселками?

Какой формальный набор функций будет достаточным? Как быть с нулём? Отдельно его паттерн-матчить? Опеределить через аддитивную айдентити? Или как быть?

Непонятно как church numerals те же получились. Или von neumann ordinals например.

А если другой тип данных, со своими отношениями?

>> No.4990846  

>>4990390
Ох, намудрил.
https://dic.academic.ru/dic.nsf/enc_philosophy/3689/%D0%A7%D0%98%D0%A1%D0%9B%D0%9E
А вообще я бы дал такое определение: число - точная характеристика, указывающая размер группы объектов или порядок в группе объектов. Кажется, здесь не из-за чего огород городить.

>> No.4991271  
Файл: ECiZdOJVUAAclx2.jpg -(98 KB, 1126x1125, ECiZdOJVUAAclx2.jpg)
98

В чем смысл шутки в этой картинке?

>> No.4991272  

>>4991271
Никакой шутки. Все так и есть на самом деле.

>> No.4996317  

>>4985939

Новогодний бамп! Я заканчиваю иллюстрации по идее и кое-чего смог накопать!

>> No.4996318  

Я тоже хотел бы быть умненьким.

>> No.4997806  
Файл: cirnono.jpg -(30 KB, 459x459, cirnono.jpg)
30

Бамп.

>> No.4998288  
Файл: cirno0.png -(27 KB, 800x550, cirno0.png)
27

Так, тред выходит заброшенным, так как не выходит в него спамить так как не до конца со своей жизнью разобрался, но можете сюда что-то писать, буду время от времени заходить. Всех с новым годом! Ваш ОП.

>> No.4998297  

>>4991272
Спасибо, офигенное объяснение, так и подбивает писать больше интересных вопросов в тред (нет).

>> No.4998299  
Файл: cirno1.jpg -(16 KB, 480x360, cirno1.jpg)
16

>>4998297
Ну зато подбивает написать невероятно остроумные (нет) ёрничанья по поводу интересности объявления, что тоже можно считать успехом!

>> No.4998301  

>>4996318
Что сделал, если это желание прошло?
Мое никуда не делось, и неистребимо, как утренний стояк.

>> No.4998731  
Файл: A9 domek_przy_zrodle.jpg -(384 KB, 955x750, A9 domek_przy_zrodle.jpg)
384

>>4998288

Я сделаю видео-анимацию по теме цветов/спектра на примере картин Яцека Йерки (эта часть уже сделана!). Видео.

По-моему это поможет найти связи с математикой, какие-то (потенциальные) уже сам нарыл.

Вероятно, я сначала спрошу (про связи) даже не у тебя (может, так удобнее будет)!

Что думаешь об этом?

>>4998288

> (спойлер)

Всё настолько плохо? То есть даже не было?

"Не имевший рогов их не терял" или как там... (знаменитый софизм)

>> No.4998808  
Файл: cirno3.jpg -(111 KB, 1000x648, cirno3.jpg)
111

>>4998731
Люблю такие картины, анимацию сделай обязательно. Связь с математикой быть может, но максимум метафорическая/метанимическая, что тоже неплохо конечно!

Всё относительно плохо, но не так плохо как у тебя (прочитал твою историю), не знаю что тебе посоветовать - просто хочу чтобы у тебя всё хорошо было!

>> No.4998849  

Если вместо интегрирования по Коши использовать интеграл по Лебегу, можно будет зафурьить функцию Дирихле?

>> No.4998852  
Файл: cirno4.jpeg -(8 KB, 246x205, cirno4.jpeg)
8

>>4998849
Хм, но теория Лебега не отличает функции которые отличаются лишь на множестве меры 0, поэтому теория Лебега "будет видеть" что функция Дирихле это 0 и конечно возьмёт от неё Фурье.

>> No.4998864  

Так я думал, спасибо.

>> No.4998869  
Файл: A6 straznik_poziomek.jpg -(456 KB, 676x540, A6 straznik_poziomek.jpg)
456

>>4998808

> Связь с математикой быть может, но максимум метафорическая/метанимическая, что тоже неплохо конечно!

Подожди, я же ещё ничего не сделал и ничего не показал тебе!

Плюс мы же обсуждали с тобой немного >>4985939 — может всё получится, если я дам больше "подсказок"/информации?

> что тоже неплохо конечно!

Если такое будут говорить и программисты, и математики, это звиздец как плохо (историю ты уже знаешь).

>> No.4998873  

>>4998869

(То есть отсутствие связи это как минимум не очень хорошие новости!)

>> No.4998886  

Почему тред не в /sci/?

>> No.4998894  
Файл: cirno5.jpg -(36 KB, 500x500, cirno5.jpg)
36

>>4998869
Ну я попробую, но не обещаю что выйдет.
>>4998886
Потому что был бы ещё более мёртвым чем сейчас!

>> No.4999181  

>>4998894

(Я не мог бы просить, чтобы получилось!) (заанимирую досмерти, даже вручную)

Просто ещё у меня самого знание увеличилось!

Получилась своего рода периодическая таблица (знаю, что с настоящей периодической таблицей мало математики связано да же? не говорю о формулах для орбиталей и такого рода математики но тут думаю другое дело)

Думаю, всё получится! Как минимум объяснить максимально просто.

>> No.4999373  
Файл: 65946409_p0.png -(731 KB, 1200x1071, 65946409_p0.png)
731

Разубедите меня в том, что современная математика это изощрённые умствования, напрочь оторванные от остальных наук и распавшиеся на многочисленные ветви так, что представители одних ветвей не понимают теорий, над которыми работают представители других.
И все вместе они не понимают и не могут объяснить, почему их деятельность интересна или зачем она нужна.

>> No.4999381  

>>4999373
Это ты сейчас точь в точь философию описал. А математика используется и приносит пользу.

>> No.4999384  
Файл: 50633732_p0.png -(610 KB, 780x1020, 50633732_p0.png)
610

>>4999381
Не хотелось бы, чтобы разговор скатился в обсуждение одной лишь "прикладной пользы". Все мы понимаем, что "польза" не интересна и не нужна свободному мышлению. Это всего лишь любимый лозунг "практически-мыслящих" рынков и политиков, заинтересованных в штамповании новых программистов. Но вот интересность "чистой" математики от такой критики яснее не становится. Есть ли на Ычане люди, которые разбираются во всяких интер-универсальных теориях Тейхмюллера, в чём они видят интересность своей деятельности?

>> No.4999389  
Файл: spoy.gif -(1 KB, 57x24, spoy.gif)
1

>>4999384
Интересно ты заходишь, вначале просишь разубедить тебя в том, что математика оторвана от жизни, а когда говорят, что у нее есть применения, говоришь, что не это хочешь услышать.
Ты как-то определись, что ли.
Если же твой вопрос сводится к тому, чем именно математикам нравится математика, то честный ответ будет "просто нравится". Точно также, как с любым другим любимым занятием.

>> No.4999411  

>>4999389

>Разубедите меня в том, что современная математика это изощрённые умствования, напрочь оторванные от остальных наук

Совершенно ясно, что имеется в виду оторванность от других наук, например от физики. Современная физика это отнюдь не обыденная жизнь, не правда ли? От "жизни" на приличном расстоянии отстоят вообще все науки. Но естественные науки описывают и объясняют реальность, а математика сама по себе - нет.

>чем именно математикам нравится математика, то честный ответ будет "просто нравится"

Но как она может нравится? Вот в чём вопрос.
Например, физика интересна, потому что может дать не только количественные, но и качественные интригующие сведения об окружающем мире. Математика даёт умозрительные конструкции, в которых я не могу увидеть движущие силы и мотивацию, если они не вдохновлены наукой.

Интерес к математике, по твоим словам, это необъяснимая случайная данность - либо он "просто" есть, либо нет. Тогда некоторые люди(я) в принципе не смогут никогда понять, чем же хороша математика и почему все её так любят. И ты не можешь его объяснить, подтверждая сказанное в 1м посте.

>> No.4999416  

Любая современная наука, это прежде всего выбивание денег. На этом, в общем то, зачастую наука и заканчивается.

>> No.4999452  
Файл: 1534103335019.jpg -(65 KB, 604x403, 1534103335019.jpg)
65

>>4999411

>Современная физика это отнюдь не обыденная жизнь, не правда ли? От "жизни" на приличном расстоянии отстоят вообще все науки
>> No.4999460  

>>4999452
Бакости какие-то постите и не краснеете. Очевидно, что глубокое понимание теоретической физики в деятельности инженера не годится, не говоря уже об остальной армии людей, пользующихся лишь небольшим множеством плодов науки. Но восклицания о "приложимости" часто используются, чтобы задеть "чистых" теоретиков или философов, а я не хочу, чтобы мой вопрос ассоциировался с попытками язвить над той деятельностью учёных, которая видится слишком абстрактной.

Философы вообще очень полезные и нужные люди, так как само их существование способно вывести из себя целый класс людей, слепо поклоняющихся выхолощенным идеалам утилитаризма.

>> No.4999466  
Файл: iichan.hk.gif -(2 KB, 122x19, iichan.hk.gif)
2

Рекомендую эссе британского математика Годфри Харди на тему выше: http://www.math.ualberta.ca/~mss/misc/A%20Mathematician's%20Apology.pdf

>> No.4999552  
Файл: 1118full-puella-magi-madoka-magica-the-m(...).jpg -(67 KB, 1118x629, 1118full-puella-magi-madoka-magica-the-m(...).jpg)
67

>>4999460

Просто зашёл сказать "Респект".

Слеза утилитариста в лунном свете способна озарить всю планету Земля.

>> No.4999553  
Файл: 1488232196355.jpg -(25 KB, 320x400, 1488232196355.jpg)
25
>> No.4999686  
Файл: cirno6.jpg -(331 KB, 1920x1200, cirno6.jpg)
331

>>4999373
>>4999384
Так, ну во-первых "распавшиеся на многочисленные ветви так, что представители одних ветвей не понимают теорий, над которыми работают представители других" это совсем неправда: математики топ и претоп уровня знают всё важное что происходит в современной математике, а если не знают, то объяснить друг другу в общих чертах могут за пару часов (что, вообще говоря не свойственно, почти всем остальным наукам, вроде химии, медицины и биологии, так что вдвойне странно это предъявлять в качестве претензии именно математике). Конкретно я, конкретно в интеруниверсальной теории Мочидзуки не разбираюсь но, во-первых я конечно далеко не претоп уровень, а во-вторых не думаю что там что-то очень важное, по крайней мере среди компании Шольце есть консенсус что abc там доказана не была, а именно в этом была её главная ценность. Тем не менее я уверен, что процент статей которые я смогу разобрать за неделю-две на том же arxiv около 70.

Во-вторых я вообще не понимаю почему математика должна быть связана с другими науками, мне кажется никакая наука априори не должна быть связана ни с какой другой, она может совсем краешком по счастливой случайности использоваться, скажем, можно сказать что ядерная физика связана с историей, потому как там применяют радиоуглеродный анализ, или генетика связана с криминалистикой, так как там используют генетическую дактилоскопию, но в этом смысле математику тоже постоянно краешком по счастливой случайности используют, Тао скажем несколько лет назад нашёл способ конструировать более эффективные алгоритмы для томографов (compressed sensing).

Но даже тут математика выгодно отличается тем, потому что помимо того что она то тут то там используется краешком на неё есть прямой почти что рыночный запрос из фундаментальной физики. Фундаментальной физике мат. аппарата буквально не хватает (чтобы построить непертурбативные QFT/теории струн, скажем).

Различные направления исследований интересны по разным причинам, иногда интерес действительно философский или исторический (другие математики посчитали что это интересно), иногда интерес сугубо практический (вроде полезности для теор. физики), иногда какой-то микс предыдущих вариантов. Впрочем так и во всех остальных науках тоже, глубокое исследование какого-нибудь одноклеточного организма или какого-то экзатического нестабильного вещества (не упоминая такие уж абсолютно бесполезные вещи как эволюция каких-нибудь галактик и звёзд) интересно не потому, что из него может потенциально получиться лекарство или ядерная бомба, а потому что другие биологи и химики посчитали что это интересно.

Короче все твои претензии к математике относятся и к другим наукам тоже в куда более большей степени. Вот так вот.

>> No.4999689  
Файл: cirno7.png -(140 KB, 225x350, cirno7.png)
140

>>4999553
sqrt(x^2)=|x|

>> No.4999692  
Файл: cirno8.png -(156 KB, 225x350, cirno8.png)
156
>Но как она может нравится? Вот в чём вопрос.

Хм, ну как и любой другой пласт человеческой культуры наверное, кому-то вот арабскую музыку слушать просто нравится, мало ли кто как развлекается.

По-поводу интригующих фактов о реальном мире: для меня математические теоремы куда более реальные например, потому что если теорема доказана я в том что "это так" 100% уверен (ну по модулю прискорбных ситуаций когда в доказательствах позже находят ошибку), а вот в адекватности какого-то нового *QFT фреймворка, которых придумывают по штуке каждые пять лет, часто не уверены даже его создатели, разбираться в каждом никакого желания нет. Фундаментальная физика в принципе в кризисе сейчас, принципиально новых эффектов или предсказаний (последнее было предсказание Бозона Хиггса который предсказли ещё в шестидесятых) которые бы подтолкнули бы к созданию принципиально новой модели физики нету, со старыми моделями куча мелких неприятных проблем, которе любому человеку с, так скажем, "чувством вкуса" мозолят глаза. А в математике куча красивых и важных теорем каждый год, которые кардинально меняют представление о свойствах и структуре математических объектов.

>> No.4999745  
Файл: f68b213ceb1a7c49a1b7388867e1c4f7 -(895 KB, 1488x1500, f68b213ceb1a7c49a1b7388867e1c4f7)
895

>>4999692

>Фундаментальная физика в принципе в кризисе сейчас, принципиально новых эффектов или предсказаний (последнее было предсказание Бозона Хиггса который предсказли ещё в шестидесятых) которые бы подтолкнули бы к созданию принципиально новой модели физики нету,

Ну не совсем. И на Тэватроне, и на БАКе (параллельно с бозоном Хиггса) искали предсказанных суперсимметричных партнеров. Обломались - и в результате Стандартная Модель сохранила свою кристальную ясность и первоначальную простоту.
Это еще не считая излучения Хокинга. А Е8 Лиси? И это - только ТЭЧ!

> со старыми моделями куча мелких неприятных проблем, которе любому человеку с, так скажем, "чувством вкуса" мозолят глаза

Это какие же? Масса нейтрино или полное смешивание в поколениях лептонов (матрица Кобаяши)? Так странно было бы ожидать, что в лептонном секторе смешивание трогает два поколения (или не трогает вообще), когда в кварковом оно работает в полный рост.
А как насчет вполне себе чисто математического, кстати доказательства, что любая петлевая гравитация не может быть не то что перенормирована, но и и даже быть внутренне непротиворечивой?

>> No.4999754  
Файл: cirno9.jpeg -(13 KB, 208x243, cirno9.jpeg)
13

Тёмная материя не очень нравится, как чисто концептуально, так и экспериментально как я понимаю есть проблемы (missing satellites, cruspy halo, ещё Stacy McGaugh в своём блоге говорил что тёмная материя плохо предсказывала угловую скорость неярких галактик, отсутствие прямых экспериментальных подтверждений).

QFT и GUT не построены (с математической точки зрения, по крайней мере), определение интеграла Фейнмана в духе всяких конструктивистов от QFT негодное, и то что физики говорят что несмотря на то что они не построены они всё равно могут всё посчитать тоже весьма сомнительно.

Квантовая механика построена на мой вкус крайне плохо, потому что сказать что нечто является системой неограниченных операторов это как бы не сказать особо ничего, что не позволяет глубокое математическое исследование каких-то общих качественных свойств квантовомеханических систем (это не только моя философская претензия, если что, а ещё и всяких умных дядек из Оксфордов которые тоже задумываются о какой-то альтернативной более удачной переформулировке).

Ну я имел в виду что какого-то успешного эксперимента не было, так-то и существование аксиона предсказывали позже 60х.

Кинь ссылку на теорему, кстати, что-то не смог нагуглить, но обычно такие теоремы они как бы либо не совсем о том о чём хотелось бы, либо не теоремы.

Но вообще я как бы не физик, так что конечно за слова не сильно отвечаю, но и у нас тут вроде не конгресс, так что всё нормально думаю!

>> No.4999761  

>>4999754
Типичная ситуация: учёные что то посчитали по имеющимся формулам и у них не сошлось с объективной реальностью. Формулы конечно же верные, поэтому они добавили константу +X чтобы подогнать. Потом посчитали снова и у них снова не сошлось. Поэтому добавили константу -Y чтобы подогнать результат. Теперь любые отклонения можно устранять пересматривая только X и Y.

>> No.4999762  

Здесь сидят очень умные Срыны, может быть они объяснят совсем не умной, чем реликтовое излучение отличается от эфирного ветра? По факту измерения, а не по предполагаемой природе возникновения.

>> No.4999767  
Файл: cirno11.png -(123 KB, 200x320, cirno11.png)
123

>>4999761
Ну это конечно по духу так, но огрублять всё же не стоит, тут вопрос в балансе "количество подгоночных параметров" vs "количество устраняемых проблем", если бы двумя новыми подгоночными константами устранялись бы все проблемы было бы замечательно на самом деле. Проблема в том, что подгоночный параметр - это распределение материи (cильно бесконечномерный по сути) а решает это проблемы не все, но если физики говорят что лучше не умеют, то нет причин им не верить и тут уж ничего не поделаешь, обвинять кого-то в шарлатанстве точно не стоит.

>>4999762
Не знаю что такое эфирный ветер (что-то из дорелятивистских теорий?), но если это то же самое что реликтовое излучение (по твоему), то какая разница каким словом называть?

>> No.4999771  

>>4999767
Такая, что за упоминание эфира в приличном научном обществе живьём сжигают на костре. В то время как с течением лет всё больше непонятно за что и почему.

>> No.4999780  
Файл: cirno13.jpeg -(7 KB, 267x189, cirno13.jpeg)
7

>>4999771
Ну, не думаю что какие-то обидки на научное сообщество это легитимный аргумент для переименовывания уже существующего термина в идеологических целях. Но что научное сообщество очень малоподвижное и плохо переносит всякие совсем смелые идеи (хотя в серьезной физике альтернативные теории гравитации вообще говоря обсуждаются) это думаю что верно. С другой стороны обычно приверженцы теории эфира не профессионалы и не очень считаются с накопленным экспериментальным базисом (или, что хуже, отрицают его скатываясь в конспирологию).

>> No.4999781  

>>4999780
Ну, так сложилось, что теории признанные еретическими обычно становятся прибежищем фантазёров всех мастей. Из официальной физики их гонят мокрыми тряпками, а здесь фактически штаб попозиции, с готовым набором научных теорий политических лозунгов, которые можно развивать.

>> No.4999838  
Файл: v guy-journalist-with-a-mike-1024x1024.png -(110 KB, 1024x1024, v guy-journalist-with-a-mike-1024x1024.png)
110

>>4999754

> QFT и GUT не построены (с математической точки зрения, по крайней мере), определение интеграла Фейнмана в духе всяких конструктивистов от QFT негодное, и то что физики говорят что несмотря на то что они не построены они всё равно могут всё посчитать тоже весьма сомнительно.
> Квантовая механика построена на мой вкус крайне плохо, потому что сказать что нечто является системой неограниченных операторов это как бы не сказать особо ничего, что не позволяет глубокое математическое исследование каких-то общих качественных свойств квантовомеханических систем (это не только моя философская претензия, если что, а ещё и всяких умных дядек из Оксфордов которые тоже задумываются о какой-то альтернативной более удачной переформулировке).

Я понимаю """""нелюбовь""""" к физике, но это твоё сообщение открыло мне нечто принципиально новое, что-то что показалось уже по-настоящему пессимистичным и даже пугающим:

1. Квантовая механика дала много новых идей (это так?)(и создавалась во много этапов). Разве не ужасно признать, что эти идеи каким-то образом "вне математики"? По-моему в таком случае получается даже сидение на многих стульях одновременно: с одной стороны говоришь о математической банальности квантмеха (видимо, внутри математики квантмех это просто очень специфическое построение из мульонов возможных?), с другой стороны критикуешь его за плохо-сформулированность (то есть на самом деле ничего не знаешь об этой конструкции, т.к. не знаешь пути улучшения)...

Хотя, вижу что здесь нет противоречия, а скорее наоборот (плюс ответ на это как бы уже содержится в твоём посте, во второй цитате). Но это противоречие может быть с точки зрения/точки отсчёта других людей.

2. Разве "плохость" интеграла Фейнмана не должна сильно вдохновлять, а не вызывать бурчание? Лишний источник вдохновения и т.д. и т.п. ... Почему это воспринимается как чужая проблема?

3. Мы же (вроде бы) знаем из реальности, что квантмех сказал очень "много чего". Для человека вне математики фраза "мы можем изучить мало свойств" будет выглядеть странно... в смысле, что это значит для квантмеха, что мы знаем мало свойств? Или что недостаток формализма позволяет выводить/считать только конкретные свойства? В общем, ничего не понятно!... по сравнению с чем оценивается количество знания и делается ли какой-то reality check? Есть ли у математики хоть какая-то критическая саморефликсия и попытки определить своё место в "пространстве идей"?

В общем, хотелось бы узнать, что собой представляет "Знание о чём-то" (много знания/мало знания) с т.з. математики и как физические теории относятся к математике (вопрос банальный, но осознал что не понимаю его). Физические теории это случайные конструкции или "монстры франкенштейна" или что? Но, несмотря на случайность, каждое революционное упрощение физической теории воспринимается как неожиданность, которую нельзя было предсказать? Ты ещё писал о математической случайности/непонятности физических концептов как таковых (не понятно, почему определять массу надо именно так и т.д.)

Просто, задумавшись, понял что ничего вообще не понимаю.

Вот ты говоришь, что квантмех даёт мало инфы. А более фундаментальная и простая теория что, даст вообще отрицательное количество информации? Ещё лучше описывая мир? Или речь идёт лишь об обёртке, "формулировке"? (но это порождает свой ряд вопросов: если одна и та же информация в разных обёртах вызывает разный эффект, то это проблема по-моему не того кто выбирает обёртку)

И просто (тревожит): если сам квантмех даёт мало инфы, то на что надеяться простым дурачкам от народа (>>4998894, >>4999181)

Теория вероятности и топология и комбинаторика (7 мостов Кёнинсберга, прыжки коня) вроде начинались с реальности и всем становилось очень хорошо ещё задолго до строгих определений... наш мир так больше не работает, такое уже не канает?

Или рандомно? Может повезёт и проканает, может нифига?

Может математик хоть в каких-то общих чертах представлять, какую информацию он должен получить? Возможны ли абсурдные ситуации, когда оценка количества информации будет слишком различаться для разных областей знания/разных перспектив?

В меме "поразительная эффективность математики" больше нюансов, чем кажется на первый взгляд? Из твоего поста складывается такая дешифровка:

> поразительная'
> 'никого не поразили и ничего не сказали?
> эффективность''
> ''эффективность? умеем ли мы считать?
> математики'''
> '''того что физики считают математикой?
>> No.4999918  
Файл: cirno15.png -(788 KB, 1024x576, cirno15.png)
788

>>4999838

  1. Хм, не могу собраться как-то чтобы полностью твой пост прокомментировать, поэтому прокомментирую как-нибудь. Квантовая механика дала много, признавать что некоторые интуиции приходят не из математики не ужасно, потому что в математике много других важных интуиций, которые чисто внутриматематические.
  2. Плохость интеграла Фейнмана должна вдохновлять тех, кто претендует на то чтобы интегралы Фейнмана строго формализовать (то есть люди уровня Концевича) и отторгать тех, кто физику хочет "просто выучить". Ну потому что многим неприятно когда нестрого, мне например неприятно.
  3. Хм, вопросов много, я постараюсь быстро рассказать как это вижу я. Каждый математический формализм описывающий эволюцию какой-то системы должен иметь какой-то "формат данных" следуя которому ты эту систему внутри формализма кодируешь. При этом этот формат он может быть очень "неограничивающим" (аналогия - байткод), а может быть наоборот очень структурным и ограничивающим (аналогия - синтаксически правильный терм какого-то сложного языка). Проблема с "неограничивающими форматами" (байткодом) в том, что там помимо того что ты можешь закодировать "реальные системы" можешь ещё закодировать и кучу мусорных. Когда куча мусора это плохо, потому что, во-первых нельзя формулировать теоремы в духе "любая система обладает хорошим глубоким свойством Р", а во-вторых если ты случайно найдёшь какую-то интересную систему (чисто на бумаге) то скорее всего это будет мусор не имеющий отношения к реальности (что не позволяет делать предсказаний о вероятном существовании каких-то экзотических систем, вроде того как предсказали чёрные дыры и радиоволны). Вот по мне квантовая механика она ближе к "байткоду" (как говорил, то что для того чтобы квант.систему задать нужно выбрать систему неограниченных операторов на rigged hilbert space и какой-то state это как бы ерунда, с таким же успехом могли бы написать "нужно выбрать последовательность нулей и единиц", никаких глубоких теорем о неограниченных операторах нет), а ОТО ближе к "сложному языку" (то что пространство-время кодируется как псевдоримановое многообразие с невырожденной (3+1)-метрикой это уже очень "ограничивающее" условие сразу связывающее ОТО со всякой deep mathematics вроде диф.топологии, глубокие теоремы о (3+1)-псевдоримановых многообразиях есть).

Про поразительную эффективность могу добавить позитивных нот. Как я уже говорил, мы уже более-менее у лимита предела наших экспериментальных возможностей, но совсем не у предела наших теоретических возможностей. Зеркальная симметрия математически неосмысленна (даже в самой-самой простой версии - гомологической зеркальной симметрии больше непонятного чем понятного), некоммутативная геометрия тоже, всякие AGT и 3d-3d correspondences выглядят очень загадочно. Возможно осмыслив всё это более глубоко мы найдём хотя бы чисто теоретическое решение многих проблем (появится очевидно хороший кандидат на GUT или QFT), потому что если чему-то и учит история с бозоном Хиггса, так это тому, что самое простой и естественное с математической тз решение каких-то физических проблем иногда оказывается верным, но чтобы его сформулировать, неплохо бы эту "математическую тз" иметь.

>> No.4999919  
Файл: cirno16.jpg -(7 KB, 200x200, cirno16.jpg)
7

>>4999918
Я как всегда накосячил с падежами и пунктуацией, но исправлять, конечно же, ничего не буду!

>> No.5000381  

Крутой у вас тред. Я вот сегодня вспоминал, как делить столбиком.

>> No.5000402  

>>5000381
Если вспомнил - расскажи, а то я тоже не помню.

>> No.5000466  

>>4999918

> Проблема с "неограничивающими форматами" (байткодом) в том, что там помимо того что ты можешь закодировать "реальные системы" можешь ещё закодировать и кучу мусорных. Когда куча мусора это плохо, потому что, во-первых нельзя формулировать теоремы в духе "любая система обладает хорошим глубоким свойством Р", а во-вторых если ты случайно найдёшь какую-то интересную систему (чисто на бумаге) то скорее всего это будет мусор не имеющий отношения к реальности (что не позволяет делать предсказаний о вероятном существовании каких-то экзотических систем, вроде того как предсказали чёрные дыры и радиоволны).

Блин, очень интересно, и совсем не раскрыто в разных книгах (наверное?). Об одном этом можно было бы целую книгу написать.

Математическая строгость это одна из интуиций (вывел экзотику строго = вероятно существует) и просто способ сказать хотя бы, точно ли твоя идея решает какую-то проблему? (Хотя иногда можно угадать, поставив именно на нестрогое решение или нестрогую идею)

Можно ли/имеет ли смысл ещё пояснить (метафору с кодом/байткодом) на визуальных образах? Получается, образ квантмеханического супа (или что у них) не так интересен (/богат) для математики, как образ искривлённого пространства?

Или сам образ интересен, но не получается подобрать ему математического аналога? Или "образ" имеют лишь ограничения (как в [программистском] языке)...

Бывают ограничивающие и не ограничивающие инструменты, может быть можно сравнить с языком или рисованием... короче, понял тебя, ну буду плодить вопросы!

Сам хотел бы, чтобы суть была в идеях/понятиях, а не в языке, и критерием было бы не ограничение (какой-то гипотетический императив), а что-то другое! (просто мысли)

Сегодня узнал, что плохо не только мне, но и занятому товарищу (оказывается, он занят и озабочен не только учёбой!), это ещё одна причина по которой сворачиваю вопросы и бегу доделывать штуку.

Спасибо, ты ответил очень понятно и исчерпывающее, если увидишь глупый вопрос у меня или что-то такое не думай что объяснял плохо.

>> No.5000501  
Файл: iichan.hk.gif -(1 KB, 75x20, iichan.hk.gif)
1

>>4999918

>если чему-то и учит история с бозоном Хиггса, так это тому, что самое простой и естественное с математической тз решение каких-то физических проблем иногда оказывается верным, но чтобы его сформулировать, неплохо бы эту "математическую тз" иметь.

А история с QM сколько она жила без S-интеграла и даже QFT сколько прошло времени до появления ренорм-группы учит тому, то да, неплохо - но глубоко необязательно.
>>5000466

>Математическая строгость это одна из интуиций (вывел экзотику строго = вероятно существует)

Ах, если бы все так хорошо было! Та же E8, десу.

>Хотя иногда можно угадать, поставив именно на нестрогое решение или нестрогую идею

Да, ХХ век на таких "методах" и выехал, по сути. Но вот дальше они почему-то не работают. Боюсь, к ToE или даже QG такой подход неприменим.
капчя предлагает быть смелым и не боятся?О_о

>> No.5000725  
Файл: cirno18.jpg -(822 KB, 1280x1024, cirno18.jpg)
822
>Или сам образ интересен, но не получается подобрать ему математического аналога?

Скорее так. Мне кажется просто пока удачной формализации не придумали. На это указывает ещё кстати и то, что там есть десятки "уточняющих подтеорий" скажем есть теории квантовых траекторий, уточняющих как именно происходит переход между разными квантовыми состояниями. В классической квантовой механике такой переход более-менее дискретен, сначала система была в одно состоянии а потом на неё чем-то подействовали и она моментальна оказалась в другом.

В то время как ОТО дана вся целиком и сразу (ну по крайней мере пока мы исследуем только гравитацию и не примешиваем туда никакие другие силы и квантовые заморочки).

>> No.5000933  

>>5000725

Всё ещё вспоминаю твою метафору (байткод и язык). По-моему очень хорошая! Кажется, как будто на все вопросы ответила. (на 100 вопросов)

> Скорее так. Мне кажется просто пока удачной формализации не придумали.

А если с тз математики интересный (если!), то почему интересный, что квантовая механика "обещает" или на что намекает математике?

> На это указывает ещё кстати и то, что там есть десятки "уточняющих подтеорий" скажем есть теории квантовых траекторий, уточняющих как именно происходит переход между разными квантовыми состояниями.

Хотел об этом спросить, но не спросил! Эти подтеории это часть современной теории или нет?

А было ли когда-то так, что всё работало как в сказке? Например, физики сообщают свои знания/догадки, а математики с помощью уже своих знаний паттернов и дедукции помогают отгадать остаток паззла? Что-то очень отдалённо похожее сам слышал лишь о создании ОТО (знаю лишь обрывки)(сейчас чуть-чуть ещё читнул).

Придумал какие-то короткие вопросы, но забыл почти все... вот такой есть: (баковый для треда)

Гиперкуб это просто 4-х мерный куб, а изучал ли кто-то(/имеет ли это вообще смысл) свойства не самого гиперкуба, а именно его вращающейся проекции?

Просто вот допустим мы встречаем такое поведение какого-то объекта (странные изменения размеров), но не знаем что это гиперкуб или хотим забыть или хотим сделать похожее (но может не гиперкуб).

Вдруг окажется, что это не гиперкуб, а что-то гиперболическое...
https://en.wikipedia.org/wiki/Circle_Limit_III

Скорее так. Мне кажется просто пока удачной формализации не придумали.

>>5000501

> А история с QM сколько она жила без S-интеграла и даже QFT сколько прошло времени до появления ренорм-группы учит тому, то да, неплохо - но глубоко необязательно.

А почему так поздно придумали? (вопрос всем) Если квантмех сейчас это байткод, то (страшно спросить) чем он был раньше? Почему его не могли свести к байткоду сразу?

> Та же E8, десу.

Слышал, это про это?:
https://ru.wikipedia.org/wiki/Исключительно_простая_теория_всего

>> No.5001211  
Файл: cirnog0.jpeg -(10 KB, 267x189, cirnog0.jpeg)
10
>А если с тз математики интересный (если!), то почему интересный, что квантовая механика "обещает" или на что намекает математике?

В классической механике у тебя есть "фазовое пространство", то есть пространство всех состояний системы, если твоя механическая система имеет конечное число степеней свободы (= задаётся конечным набором чисел, как в случае, скажем, маятника который задаётся вообще одним числом) то математика работает очень хорошо, можно сказать что "фазовое пространство" это многообразие, а многообразия математики любят (это некоторый фундаментальный математический класс объектов, который считается естественным настолько, что любой результат о них считается самоценным). Соответственно в квантовой механике хотелось бы "квантовое фазовое пространство" кодировать чем-то что называлось бы "квантовое многообразие" или "некоммутативное многообразие" (математики слово "квантовый" не любят а слово "некоммутативный" любят, хотя значат они примерно одно и то же). Соответственно квантовая механика намекает на существование чего-то, что можно было бы назвать "некоммутативной геометрией", это более-менее моя специализация. К сожалению некоммутативная геометрия сейчас похожа на нечто вроде набора примеров (которые хотелось бы называть "некоммутативным многообразием"), никакого хорошего определения нету.

>Хотел об этом спросить, но не спросил! Эти подтеории это часть современной теории или нет?

Ну, они часть современной теории в том смысле что ими пользуются чтобы писать всякие статьи в Nature, но не часть в том смысле, что они гораздо менее строгие чем классическая квантовая механика. Поэтому такие математики как я они классическую ещё туда-сюда знают, а когда начинаются всяки открытые квантовые системы с диссипативностью то сразу плывут. Впрочем, открытые классические (не квантовые) механические системы это тоже очень сложно и плохо ложится во всякие строгие фреймворки симплектической/пуассоновой геометрии, так что тут проблема не в квантовости, а в том что диссипативные системы сложные и "по-простому" с ними работать не выходит.

>А было ли когда-то так, что всё работало как в сказке? Например, физики сообщают свои знания/догадки, а математики с помощью уже своих знаний паттернов и дедукции помогают отгадать остаток паззла? Что-то очень отдалённо похожее сам слышал лишь о создании ОТО (знаю лишь обрывки)(сейчас чуть-чуть ещё читнул).

Ну, бывало по-разному, СТО и ОТО действительно похожим образом придумали, я в истории физики не очень силён, только какие-то байки знаю. Вроде кварки в стандартной модели соответствуют простым алгебрам Ли маленьких порядков или чему-то такому.

>Просто вот допустим мы встречаем такое поведение какого-то объекта (странные изменения размеров), но не знаем что это гиперкуб или хотим забыть или хотим сделать похожее (но может не гиперкуб).

Да, конечно, это более менее то что называется convex geometry, восстановить выпуклое тело по набору проекций - это одна из типичных задач, которая в том числе и прикладной смысл имеет, понятно какой (томография, МРТ и такое всё). Обычно правда задача ставится в духе "есть конечное число проекций и направлений вдоль которых эти проекции мы берём, восстановить тело", а в твоём случае проекций бесконечно и направлений мы не знаем, буквально твою формулировку я нигде не видел, но допускаю что такое тоже есть (поищу если не лень будет)

>> No.5001257  

>>5001211

> можно сказать что "фазовое пространство" это многообразие, а многообразия математики любят (это некоторый фундаментальный математический класс объектов, который считается естественным настолько, что любой результат о них считается самоценным). Соответственно в квантовой механике хотелось бы "квантовое фазовое пространство" кодировать чем-то что называлось бы "квантовое многообразие" или "некоммутативное многообразие" (математики слово "квантовый" не любят а слово "некоммутативный" любят, хотя значат они примерно одно и то же). Соответственно квантовая механика намекает на существование чего-то, что можно было бы назвать "некоммутативной геометрией", это более-менее моя специализация. К сожалению некоммутативная геометрия сейчас похожа на нечто вроде набора примеров (которые хотелось бы называть "некоммутативным многообразием"), никакого хорошего определения нету.

Ну ты хотя бы сам понимаешь, насколько это круто?

Почему про такое не (?) пишут книги?

Почему тред не про это?

До боли понятно/интересно...

А математики "подключились" к QM/увидели в ней потенциал настолько быстро насколько возможно? Какова "математическая история" QM?
https://en.wikipedia.org/wiki/Mathematical_formulation_of_quantum_mechanics

Но тут, наверное, можно сначала самому почитать! Стыдно спрашивать, т.к. некоторые вопросы начинают ходить по кругу и ряд ответов оценить не смогу.

Под "классическая" квантовой механикой не имеется в виду какая-то из старых версий? (вдруг путаница у меня)

>> No.5001312  
Файл: cirno19.jpeg -(9 KB, 187x270, cirno19.jpeg)
9

>>5001257
Рад что тебя это так впечатлило! Про такое пока даже нормальных учебников написано, не говоря уже о популярных текстах, можешь посмотреть лекцию Каледина https://www.youtube.com/watch?v=QbDjELUGfmw она как бы от математика для математиков, но первые 20 минут там болтовня, поэтому может проникнешься духом.

Увидели конечно, очень большой кусок того, что Атья называл "core mathematics" связана так или иначе с фундаментальной физикой. Под "классической квантовой механикой" я имею в виду всё то, что не попадает под "Later developments" в статье которую ты кинул, то есть некоторый совсем базис позволяющий работать с совсем простыми квантовыми системами.

>> No.5001546  

>>5000381
А кто-нибудь знает как извлекать корни на бумаге?
Мну как-то рассказывали, но я не смог вникнуть.И демонстрация провалилась.

>> No.5001582  

>>5001312

Проникся духом первых слов 10-20 (20 много наверное уже)!

> Увидели конечно, очень большой кусок того, что Атья называл "core mathematics" связана так или иначе с фундаментальной физикой.

Имел в виду, что осталась такая непонятка: ты описал отношение математики к QM, но у QM и элементарных частиц была долгая история с "заторами". Те прошлые версии QM были тоже байткодом, или ещё не были даже им (как тогда к этому относились математики?), или... или почему нельзя было предсказать переход к тому байткодному языку, к которому перешли сейчас?

Хотя надуманный вопрос вероятно, сейчас понял это.

>> No.5001638  
Файл: cirno21.jpeg -(8 KB, 199x253, cirno21.jpeg)
8

>>5001582
Вопрос нормальный, но мне кажется никакой особо богатой истории у неё нет, фон Нейман почти сразу сформулировал тот "байткодовый" формализм которым пользуются и по сей день. Далее строили только различные уточняющие теории (не очень строгие с математической тз) над ним.

>> No.5001837  

>>4946088
https://youtu.be/5UWv6WSbMwQ

>> No.5001904  

>>5000402
Методом подбора.

>> No.5002512  

>>5001638

(Понял!... особенно ещё спустя время читая твой пост снова.)

Напишешь, если на горизонте нависнут дела и надо будет поторопиться с видео? (если это поможет вообще)

>> No.5002688  

Мне вчера сказали, что непрерывная математика умерла, ну или по крайней мере вышла из моды. Это правда?

>> No.5002690  
Файл: graph.png -(24 KB, 432x432, graph.png)
24

А как найти количество кратчайших путей короля от точки 0,0 до x,y где x, y больше либо равно нулю ? Можно ли формулу вывести? Это похоже на пути Дика, но не оно.

>> No.5002707  
Файл: cirno23.jpg -(444 KB, 1920x1200, cirno23.jpg)
444

>>5002512
Да у меня по-хорошему на горизонте всегда дела, тред не настолько активный чтобы я не успевал читать, так что торопиться не стоит, мне кажется если чего-то запредельно плохого не случится, то в течении недели я в любом случае отвечу.
>>5002688
Не, ни в каком смысле не правда. Я бы вообще делил не на "непрерывную" и "дискретную", а на "жёсткую" (hard) и "мягкую" (soft). В жёсткой конкретные оценки, неравенства, асимптотики, числа, а в мягкой вопросы существования/несуществования, препятствия, инварианты, классификации и т.д. В моём мировоззрении это довольно далёкие друг от друга области, и мало людей существует которые пишут статьи и в первой ветке и во второй, я вот не из таких к сожалению.
>>5002690
Выглядит просто, но подумаю попозже.

>> No.5002767  
Файл: Untitled.jpg -(11 KB, 418x179, Untitled.jpg)
11

>>5001546
Как-то так? Вроде бы метод Ньютона.

>> No.5002896  

>>5002688

>Мне вчера сказали, что непрерывная математика умерла

Нет, она просто так пахнет.
>>5002767
Похоже. Спасибо.

>> No.5002980  
Файл: Cheyb.png -(6 KB, 212x213, Cheyb.png)
6

>>5002690

манхэттенское расстояние

метрика прямоугольного города

метрика L1 или норма

метрика городского квартала

метрика такси

метрика Манхэттена

прямоугольная метрика

метрика прямого угла

Расстояние Чебышёва

https://ru.wikipedia.org/wiki/Расстояние_городских_кварталов
https://en.wikipedia.org/wiki/Taxicab_geometry
https://ru.wikipedia.org/wiki/Расстояние_Чебышёва
https://en.wikipedia.org/wiki/Chebyshev_distance

Не знаю, помогло ли это вообще!

Лично моё мнение в том, что сумма кол-ва путей по сути равна расстоянию (только в чём-то в два раза больше из-за отражения).

А картинку ты кажется оттуда и взял!
https://en.wikipedia.org/wiki/King%27s_graph

>> No.5002986  

>>5002707

Спасибо, понятно!

>> No.5002997  

>>5002980

> Лично моё мнение в том, что сумма кол-ва путей по сути равна расстоянию (только в чём-то в два раза больше из-за отражения).

Конечно же нет. Если смотреть количество кратчайших путей если оно строго по диагонали, т.е. от {0,0} до {1,1}, {2,2}, {3,3} и так далее, то там количество кратчайших путей всегда 1.

>> No.5003077  
Файл: original-3072992-1.jpg -(44 KB, 350x255, original-3072992-1.jpg)
44

>>5002997

Так совмести одно с другим просто/усовершенствуй/намотай на ус! Я высказал лишь своё мнение.

У тебя >>5002690 расстояние - 5.

Диагонали - 3.

4 способа уйти с первой диагонали. 4 способа уйти со второй диагонали. 0 способов уйти с последней диагонали (петлять не выгодно). key insight?

... Ну, и ещё сколько-то способов скомбинировать эти переходы "вниз" (наверное, какая-то простая комбинаторика).

== Ты получаешь формулу для части путей. Потом сшиваешь с формулами для оставшихся случаев (И опровергаешь кого хочешь.)

>> No.5003112  
Файл: graph.png -(25 KB, 432x432, graph.png)
25

>>5002690
Поправил картинку

>> No.5003137  
Файл: v Help raz.png -(954 KB, 2880x2880, v Help raz.png)
954

>>5003112

Как ты смеешь меня игнорировать? (Шучу!)

Вот первые эксперименты.

Тут пояснение идеи "количество путей = расстояние (+ что-то)" (своего рода классификация путей + паттерны внутри неё)

И метод более быстрого подсчёта (не считая формулы, которую ещё не открыли мы), к которому пришёл по итогу.

По моему не-математическому взгляду может быть очень интересной задача!

>> No.5003452  
Файл: The-19-grand-Motzkin-paths-of-length-4-T(...).png -(15 KB, 397x498, The-19-grand-Motzkin-paths-of-length-4-T(...).png)
15

Motzkin paths какие-то нагуглил, в общем сложная какая-то математика там
https://arxiv.org/pdf/1506.06824.pdf

>> No.5003458  
Файл: delannoypaths4.png -(7 KB, 295x290, delannoypaths4.png)
7

https://www.robertdickau.com/lattices.html

>> No.5003471  
Файл: v Help 2 raz dva.png -(366 KB, 1728x1728, v Help 2 raz dva.png)
366

>>5003452

На твоей картинке (>>5003112) расстояние до точки — 5, количество путей — 15.

Будь точка в два раза ближе по этой диагонали, расстояние было бы 3, количество путей — 6. (то что пока узнал!)

>>5003458

Пришёл к тому же здесь внизу:
http://iichan.hk/b/src/1579433393132.png

>> No.5003538  

Вот ответ, если что https://en.wikipedia.org/wiki/Trinomial_triangle

>> No.5003816  
Файл: v Help 3 CsyzK.png -(18 KB, 885x246, v Help 3 CsyzK.png)
18

>>5003538

А вот ответ покрупнее, если что! 6 и 15 здесь видны.

>> No.5009553  

>>4946080

Можно ли сказать, что создание физической теории (и не только) — это придумывание сущности, типа "пучка информации", который каким-то новым способом кодирует информацию?

Например, у Пенроуза это твисторы, в которых как-то странно точки становятся линиями и т.д.

В квантовой механике это всякие пространства состояний или матрицы рассеивания или волны вероятности или диаграммы Фейнмана.

В попытках квантовой гравитации это графы, в которых рёбра становятся объёмами и прочими вещами и т.д.

В общей теории относительны - куски искривлённого пространства.

Понимаю, что для тебя это ничего не говорит и не добавляет информации, но для меня важно/было бы интересно!

В математике этим могут быть вероятности или понятие информации (кодируют какие-то свойства штук тоже).

>> No.5009559  

>>5009553
Физическая теория обычно работает наоборот. Есть сущность и вы придумываете её описание языком математики, либо же пытаетесь объяснить её языком математики. Собственно на уточнениях и поисках точного описания и базируется создание физических законов.

>> No.5009599  
Файл: cirno59.jpg -(78 KB, 474x842, cirno59.jpg)
78

>>5009553
Ну примерно можно, хорошие физические теории это всегда придумывание новых фундаментальных математических сущностей. Более-менее.

>> No.5009810  

>>5009559

> Собственно на уточнениях и поисках точного описания и базируется создание физических законов.

Описание и назвал сущностью тогда!
>>5009559, >>5009599

Спасибо, что вы есть!

>> No.5015544  
Файл: Hydra_Eng.png -(90 KB, 897x954, Hydra_Eng.png)
90

В какой (мета)теории можно доказать, что для любой возможной непротиворечивой теории можно синтезировать непротиворечивую надтеорию, в которой изначальная теория описывается и в которой можно доказать неполноту (доказать недоказуемость некоторых утверждений из) исходной теории?

>> No.5015756  
Файл: cicirno.jpg -(40 KB, 432x512, cicirno.jpg)
40

>>5015544
Ну это странный вопрос какой-то. В любой, если метатеория позволяет как-то говорить об объектных теориях, то она позволяет и теорему Гёделя сформулировать (ну я подразумеваю что ты имел в виду "в любой гёделевской непротиворечивой теории", а не "в любой возможной", потому что иначе это просто неверно). Арифметики Пеано хватит, и даже какого-нибудь её слабого подмножества вроде I \Sigma_1.

>> No.5015799  
Файл: 3-Table1-1.png -(24 KB, 642x268, 3-Table1-1.png)
24

>>5015756

>В любой, если метатеория позволяет как-то говорить об объектных теориях, то она позволяет и теорему Гёделя сформулировать (ну я подразумеваю что ты имел в виду "в любой гёделевской непротиворечивой теории", а не "в любой возможной", потому что иначе это просто неверно). Арифметики Пеано хватит, и даже какого-нибудь её слабого подмножества вроде I \Sigma_1.

А как в рамках арифметики Пеано мы докажем, что вот есть некая теория с таким-то набором аксиом, и она тождественна теории которую мы описали в рамках арифметики Пеано? Быть может, нужна некая особая алгебра аксиом или нечто подобное?

>> No.5016622  
Файл: cicicirno.png -(160 KB, 393x500, cicicirno.png)
160

>>5015799
Ну это совсем странный вопрос. Как в рамках арифметики Пеано доказать, что число 1, которое чистая идея из пещеры Платона, тождественно тому число 1 которое описывается в арифметике Пеано? Вообще говоря никак, математика работает с маетматическими формальными описаниями, а не непосредственно с метафизическими сущностями.

>> No.5017026  

>>5016622
Ну вот допустим мы описали арифметику Пеано(PA) в PA. Как доказать в самой PA, что любое доказуемое утверждение в PA, доказуемо и в PA описанное внутри PA, любое недоказуемое утверждение в PA недоказуемо в PA внутри PA, и что нет никакого контрпримера, чтоб что-то вот там доказывалось, а там не доказывалось?

>> No.5017052  
Файл: dididirno.jpeg -(19 KB, 225x225, dididirno.jpeg)
19

>>5017026
Никак, PA не умеет "обращаться" к чему-то, что не описывается внутри PA. Ты можешь это доказать только с "внешней" точки зрения, если сама PA описана внутри, скажем, ZFC (или любой другой метатеории), и внутри РА описана своя маленькая РА, то ты можешь доказать "эквивалентность", "внутренней" и "внешней" РА c точки зрения этой внешней ZFC.

>> No.5017263  
Файл: slide-1.jpg -(118 KB, 1024x768, slide-1.jpg)
118

>>5017052
А есть ли понятие, родственное Тьюринг-полноте, но только про теории? Ну что если теория достаточно мощная, чтобы в ее рамках можно было описывать теорию X и доказывать в рамках этой теории-в-теории все теоремы из этой теории, то такая теория обладает некой Тьюринг-полнотой или какой-то другой полнотой. Ну т.е. по аналогии с Тьюринг-полнотой, что если на чем-то можно проэмулировать Машину Тьюринга с бесконечной лентой, то это что-то тоже полно по Тьюрингу.

>> No.5017272  
Файл: _.jpg -(58 KB, 500x708, _.jpg)
58

Как научиться математике, если ты глупый и ленивый?

>> No.5017307  
Файл: 1580650146431.jpg -(99 KB, 600x785, 1580650146431.jpg)
99

>>5017272

>> No.5017312  

>>5017272
Для чего, например?

>> No.5017337  
Файл: cikirno.jpg -(274 KB, 707x999, cikirno.jpg)
274

>>5017263
Гёделевская теория: непротиворечивая, эффективная теория в языке арифметики, в которой выразимы все вычислимые предикаты.
>>5017272
Учи глупо и лениво тогда!

>> No.5017682  

Как соотносятся понятия "истинность", "доказуемость", "вычислимость", где про это почитать?

>> No.5017688  

>>5017682
Вычислимость как-то выпадает из этого ряда - функция вычислима, если есть машина Тьюринга, которая ее вычисляет. Остальные два пункта - это про метатеории, но не знаю, где именно искать.

>> No.5017711  

>>5017688
Будто бы есть доказуемое, но не вычислимое.

>> No.5017753  
Файл: cirirno.jpeg -(10 KB, 180x280, cirirno.jpeg)
10

>>5017682
В учебниках логики. "Вычислимость" действительно определена для функций на каких-нибудь конструктивных множествах (натуральных чисел, конечных строк над конечным алфавитом, конечных графов и тд), "доказуемость" для утверждений в дедуктивных системах, "истинность" для утверждений в языках с фиксированной интерпретацией. Связей много. Если для какой-то дедуктивной системы зафиксирован "класс допустимых моделей" и верно что, из доказуемости следует истинность в любой модели (и сущестование хотя бы одной) то говорят что выполнена "теорема о корректности", если из истинности в любой модели следует доказуемость то говорят что выполнена "теорема о (семантической) полноте". Для теорий первого порядка выполнены и теорема о корректности и теорема о полноте. Теорию назыают вычислимой если множество её аксиом вычислимо. Теория не может быть одновременно вычислмой, непротиворечивой, синтаксически полной (для любого утверждения A, доказывать либо А, либо не А) и "достаточно сильной" (например, интерпретировать внутри себя какой-нибудь большой кусок арифметики типа I \Sigma_1) это теорема Гёделя о (синтаксической) неполноте (сильных арифметических теорий первого порядка).

Вопрос просто слишком ёмкий, весь стандартный университетский курс мат.логики более-менее о различных взаимоотношениях тех понятий которые ты перечислил.

>> No.5017756  
Файл: cikurno.png -(619 KB, 500x626, cikurno.png)
619

>>5017753
Совсем конкретная рекомендация: книжка Шеня "Языки и исчисления" мне кажется супер-хорошая, очень тонко выдержана грань таким образом, что учебник не скатывается ни в махания руками, ни в компьютеризированное занудство, что мне кажется весьма ценным для первого учебника по логике. По крайней мере лучшего я не видел.

>> No.5017856  

>>5016622

> Вообще говоря никак, математика работает с маетматическими формальными описаниями, а не непосредственно с метафизическими сущностями.

А раз такая пьянка, то...

Что означает """опровержение""" (обращаю внимание на три кавычки!) теоремы Нётер в случае теории относительности?

https://youtu.be/04ERSb06dOg?t=267

4:27 - 5:56

Были ли какие-то интересные "драмы" на тему """опровержения""" Зеноном математического моделирования, области в которых это обретало какое-то интересное значение?

https://ru.wikipedia.org/wiki/Апории_Зенона#Современная_трактовка
https://en.wikipedia.org/wiki/Supertask

видео уже в пути! к появлению

>> No.5017970  
Файл: cilirno.jpg -(12 KB, 200x202, cilirno.jpg)
12

>>5017856
В классической механике есть энергия, которая инварианта относительно сдвигов по времени, и импульс, инвариантный относительно сдвигов пространства. В СТО есть тензор энергии-импульса, инвариантный относительно преобразований Лоренца. Означает, что ситуация иногда бывает сложнее, чем кажется на первый взгляд и эта сложность может быть замечена только при экстремально больших/маленьких значений некоторых параметров.

Про "апории Зенона" не могу что-то интересное сказать. Помню была байка, якобы Коши после того как придумал определение суммы ряда как предела частичных сумм пошёл проверять все свои вычисления на предмет соответствия этому определнию. Вообще я бы этих мысленных экспериментов с бесконечностями можно много придумать (супертаски которые ты кинул), в математике придумали как оперировать бесконечностями логически непротиворечивым образом. В математической формализации "апории зенона" никакой проблемы нету, потому что сумма бесконечного числа положительных чисел (согласно внутренней логике теории действительных чисел) может быть конечным числом. Но это всё банальности конечно.

Вообще есть целые области математического знания которые работают с какими-то очень абстрактными и почти что метафизическими штуками (большие кардиналы, нестандартные модели арифметики, вычисления с оракулами, дескриптивная теория множеств) и по духу очень похожие на разрешения всяких "апорий", схожих с "апорией Зенона".

>> No.5018323  
Файл: 1011px-Cake_quarters.png -(112 KB, 1011x768, 1011px-Cake_quarters.png)
112

>>5017970

Спасибо большое! Снова ответ, заставляющий считать себя недостойным ответа, словно цитаты из какой-то вкусной книги (отсутствующей на полках книжных магазинов) посыпались.

... Один вопрос ещё мучает: (не сильно!)

Дроби иногда объясняют разрезанием пирога.

А что если пироги разного размера или разной плотности? Понимаю, что это можно учесть отдельно.

Существуют ли понятия, похожие на дроби, но не совсем дроби?

>> No.5018410  
Файл: cipirno.jpg -(59 KB, 496x767, cipirno.jpg)
59

>>5018323
Есть поле частных от любой области целостности и есть локализация категорий по некоторому множеству стрелок удовлетворяющему условиям Оре, но там обобщения понятия "дроби" не совсем в ту сторону, в которую хочешь ты.

>> No.5018731  

>>4946088
От их страстного соития возникнет неопределяемая неопределенность

>> No.5018876  
Файл: differentiation_and_integration.png -(144 KB, 632x643, differentiation_and_integration.png)
144

Почему интегрирование настолько сложнее дифференцирования?

>> No.5018912  
Файл: cilirno.jpg -(54 KB, 512x320, cilirno.jpg)
54

>>5018876
Потому что дифференцирование это локальная операция и чтобы посчитать производную нужно учесть информацию в сколь-угодно малой окрестности точки, а (определенное) интегрирование глоабльная операция и должна учитывать всю информацию о функции в каждой точке, это сложнее.

>> No.5018938  

>>5018876
А еще можно вспомнить, что извлечение корня/логарифма сложнее, чем возведение в степень, и даже деление сложнее умножения.
Обратная задача почти всегда сложнее прямой. Иногда — сильно сложнее. На этом, кстати, шифрование во многом основано.

>> No.5018946  
Файл: cipirno.png -(801 KB, 1000x1270, cipirno.png)
801

>>5018938
Только какую задачу считать "обратной" а какую "прямой" во многом вопрос конвенций. Особенно если учесть то, что численно посчитать (с хорошей точностью) сложнее как раз производную, а (определенный) интеграл проще.

>> No.5019886  
Файл: d1nd4ytg1cch4fxaad97wh41nt.png -(411 KB, 600x600, d1nd4ytg1cch4fxaad97wh41nt.png)
411

Вот например если рассмотреть круг - у круга есть периметр (длина граничной окружности) и площадь. Если рассмотреть шар - у него есть площать и объем. А вот у 4-мерного шара должен быть объем и еще некая характеристика X, которая относится к объему так же, как объем относится к площади, и как площадь относится к периметру. Как эта характеристика называется?

>> No.5019887  
Файл: cinirno.jpg -(53 KB, 384x512, cinirno.jpg)
53

>>5019886
Специальных названий нет, в бытовой речи (на докладе, например) говорили бы "объём 4-шара" и "площадь 3-сферы", можно говорить 4-объём 4-шара и 3-объём 3-сферы, если хочется быть более точным. Можно приставки "гипер" добавлять (хотя они немного по-школьному звучат на мой вкус).

>> No.5019890  
>5019887

А в рамках какой теории можно показать, что вот для круга имеет смысл говорить о периметре и площади, у шара - о площади и объеме, у 4-шара о объеме и 4-объеме и так далее? Почему именно по две таких характеристики, всегда ли их будет по две если переходить к еще более многомерным шарам?

>> No.5019914  

>>5019890
Вообще говоря, их всегда будет N. Просто та величина, что последняя, имеет вполне ясный смысл. У остальных с этим плоховато: возьмем наиболее простой объект, вкладывающийся в четырехмерное пространство лучше, чем в трехмерное - тор. Объем 4-полнотория - вполне очевиден, и его 3-поверхность - тоже можно представить хотя бы по аналогии с трехмерным случаем. А вот какой смысл имеют площади характерных окружностей 4-тор это x1=rsina x2=rcosa x3=Rsinb x4=Rcosb - уже надо подумать... Конечно, их произведение - это 4-объем, но это ведь не единственный их смысл.

>> No.5019916  

>>5019914

>последняя

Предпоследняя d/dr имелась ввиду. Объем всегда осмысленен.

>> No.5019922  
Файл: cinirno.jpg -(49 KB, 710x444, cinirno.jpg)
49

>>5019890
Немного неправильный вопрос, потому что объём обычного 3-шара и площадь 2-сферы (которая граница 3-шара) это как бы не то чтобы какие-то особенно связанные друг с другом характеристики и чем-то выделенные, можно ещё рассмотреть, скажем, длину диаметра или длину большого круга (геодезической на сфере) или ещё каких-нибудь кривых/поверхностей внутри 3-шара. Если шар заменить на другие фигуры, какие-нибудь эллипсоиды с внутренностью, например, то там будут другие "особые кривые" (геодезические вдоль главных осей).

Про то сколько "характеристик" достаточно знать о фигуре чтобы знать всё о её внутренней геометрии. Вся метрическая информация о фигуре: объём, площадь границы, длины любых кривых на фигуре, k-объёмы любых k-мерных подфигур фигуры и т.д. инкапсулируется в объекте который называется "метрическим тензором". Это, если совсем грубо и не совсем правильно, следующий набор данных: для каждой точки данной n-мерной фигуры (n-шара, например) ты можешь при помощи некоторой процедуры построить n(n+1)/2 чисел и на любой вопрос о метрической геометрии пространства (объём всей фигуры, объём её границы и т.д.) ты можешь ответить используя вот этого вот огромного набора чисел.

В абстраткной (бескоординатной) математике всё наоборот, конечно, ты определяешь понятие "n-мерная фигура" как набор точек с вот этим вот "метрическим тензором": то есть с n(n+1)/2 числами для каждой точки фигуры.

Это конечно переупрощённая картина и не совсем корректная, но, надеюсь, хотя бы понятная.

>> No.5019962  

>>5019922

>объём обычного 3-шара и площадь 2-сферы (которая граница 3-шара) это как бы не то чтобы какие-то особенно связанные друг с другом характеристики

Да ты что! Связь проста, наглядна и ты сам ее описал: граница. То самое d/dr или ∂, если уж быть совсем точным.
Если продолжать рассуждать, у шара, помимо объема (+3) и поверхности (+2) есть еще две очевидные характеристики - обычная (-1) и риманова (-2) кривизна. Надеюсь, понятно, что за цифры имелись ввиду.

>> No.5019963  
Файл: civirno.jpeg -(8 KB, 223x226, civirno.jpeg)
8

>>5019962
Спасибо, буду знать!

>> No.5020306  

>>5018946

>какую задачу считать "обратной" а какую "прямой" во многом вопрос конвенций.

Мне кажется, при "прямой" — производится своего рода "свертка". Информация теряется и/или переводится в более труднодоступную/обобщенную форму. А обратная задача — ее из этой формы извлечь. Как-то так.

>> No.5021417  
Файл: D3-brane_et_D2-brane.PNG -(317 KB, 1185x839, D3-brane_et_D2-brane.PNG)
317

Готовься к "захвату" твоего треда видео-темой до 7-го

>>5018946 >>5020306

Вы же вроде все только что сами говорили об объективных отличиях одних задач от других? Немного странный/непонятный момент-поворот диалога!

1 Нашёл по видео Numberphile:
https://ru.wikipedia.org/wiki/Модель_песчаной_кучи

Сколько ещё таких мемов хрянится в секрете?

На песок насыпали песок

Клеточный автомат

Алгебру и группу

Фрактал

Настоящий бутерброд из концепций и всё наглядно!

2 Узнал ещё один-супер важный факт, о котором почему-то так просто не услышать --

Оказывается, придумали физический объект который является воплощением очень абстрактного математического:

https://en.wikipedia.org/wiki/String_theory#Branes

> Branes are frequently studied from a purely mathematical point of view, and they are described as objects of certain categories, such as the derived category of coherent sheaves on a complex algebraic variety, or the Fukaya category of a symplectic manifold.[31] The connection between the physical notion of a brane and the mathematical notion of a category has led to important mathematical insights in the fields of algebraic and symplectic geometry [32] and representation theory.[33]

Хотя это не очень понятно!

Чисто математически = интересуются не совсем тем, чем физики, или как?

>> No.5021747  
Файл: cimirno.jpg -(18 KB, 255x343, cimirno.jpg)
18

>>5021417
Да я сам об этой песчаной куче не знал, забавная штука.

А вот зеркальная симметрия это буквально моя тема и могу даже что-то по ней сказать. История была ровно наоборот, придумали некоторую математическую формализацию очень абстрактной физико-философской концепции. 2-браны это как бы "философско-физическая" абстракция - это обобщение понятия "точечная чистица" на более высокие размерности, в частности, отображение 2-браны в какое-то пространство время X, можно рассматривать как "тормозной путь" некоторой струны (некоторой философской сущности которая должна быть одномерным обобщенем понятия "точечная частица"). Одной из физических формализаций этой концеции было понятие фундаментальных 2-бран 3-сигма моделей и моделей Ландау-Гинзбурга. По модулю многих тонкостей физиками было замечено следующее: при некоторых режимах параметров Калаби-Яу сигма модели имеют описание в терминах моделей Ландау-Гинзбурга, и наоборот на теории Ландау-Гинзбурга (при некоторых определенных обстоятельствах) можно смотреть как на сигма-моделях. В такой ситуации выходило что калаби-яу сигма-модель М отображалась в калаби-яу модель ландау-гинзбурга М'. Это заинтересовало уже комплексных и алгебраических геометров (что это значит для геометрии зеркальных партнёров М и М'? как можно построить M' имея на руках только М?). Концевич придумал как формализовать "гомологическую тень" того, что нам бы хотелось называть "зеркальной симметрией" (как Аинф эквивалентность некоторых Аинф категорий) и назвал это "гомологической зеркальной симметрией", в частности из формулировки Концевича следовало что такая зеркальная симметрия должна как бы "менять местами" симплектическую и комплексную структуру кэлерова многообразия (потому что левая сторона этой А_инф эквивалентности зависит только от комплексной структуры а правая только от комплексной). Но это, конечно, не то чтобы хотелось называть "зеркальной симметрией", потому что гомологическая алгебра "теряет слишком много информации", но общая философия такая, что есть некоторая глубокая двойственность между комплексной и симплектической геометрией, и она понята не до конца, кое-какими "проявлениями" этой двойственности я и занимаюсь, я хочу показать что зеркальная симметрия меняет некоторые числа местами, что априори совсем не следует из гомологической формулировки Концевича. Рассказ конечно очень сумбурный вышел, но надеюсь хоть что-то удалось уловить.

>> No.5021860  

>>5021747

А, не хотел сказать, что порядок важен (что связь хронологическая)! не думал, что браны откуда-то "стащили".

> Рассказ конечно очень сумбурный вышел, но надеюсь хоть что-то удалось уловить.

Думаю тебе удалось передать много интересного!

Но что в этом вопросе стоит на кону у математиков и у физиков? Их интересует одно и то же? Улучшенное понимание физики? Или неизвестно?

Ты упоминал как-то "орбиты" (что занимаешься ими), это была та же проблема?

>> No.5021864  

>>5021860

(про математическую часть стало как раз понятно! и заодно немного про физическую)

>> No.5021985  
Файл: civirno.jpg -(270 KB, 1920x1080, civirno.jpg)
270

>>5021860
Их интересует немного разное, но продвижения физиков влияют на продвижение математиков и наоборот. Физиков интересует построение конформных теорий поля / непертурбативных теорий струн и какое-то адекватное описание всех симметрий в них и пространств стабильных состояний (есть очень глубокие интуиции которые говорят нам, что такие пространства должны быть конечномерными и иметь кучу симметрий, но пока никто не придумал как формализовать), математиков интересует глубокая связь между комплексной и симплектической геометрией. Но и та и другая программа пока довольно далеки для завершения. Про орбиты - не помню уже что я говорил если честно, пол-года назад я решал алгебро-геометрическую задачу с зеркальной симметрией не связанную непосредственно, так что наверное нет.

>> No.5022885  

>>5021985

Спасибо! Не успел "до 7-го", видео попозже!..

Нашёл ещё диалоги Гордона, но там не так интересно как здесь.

Там нет гомологических теней идей в виде гомологических симметрий!

>> No.5022927  
Файл: kipirno.jpg -(43 KB, 900x596, kipirno.jpg)
43

>>5022885
Имено про зеркальную симметрию или теорию струн? Скинь в любом случае.

>> No.5023583  
Файл: Minkovsky_fantas1.svg.png -(32 KB, 587x600, Minkovsky_fantas1.svg.png)
32

Кстати по поводу времени - вот есть темпоральная логка, а бывает ли темпоральная логика с общей/специальной теорией относительности?

>> No.5023657  
PANIC!!!
>> No.5027048  
Файл: yhu.gif -(1707 KB, 480x270, yhu.gif)
1707

>>5027046
Жду!

>> No.5027064  
Файл: сiqirno.jpg -(48 KB, 500x500, сiqirno.jpg)
48

>>5027046
А, ну да, я чуть облажался, потому что, как правильно ты заметил в стёртом сообщении n=6 1 2 4 1 2 4 тоже цикл, можно тогда сформулировать "при любом натуральном n>=1, f^(n)(x) = x не имеет решений в натуральных x>4", скажем.

>> No.5027066  

>>5027046

> решений кроме n=3 и x=1,2,4

Ну для n=6 тоже будет решение, притом такое же самое, цикл будет 1,4,2,1,4,2. Это если у нас одна операция - x/2, вторая 3x+1.

Если одна операция это x/2, вторая (3x+1)/2 то тогда цикл длиной 4 будет 1,2,1,2. Цикл длиной 6 будет 1,2,1,2,1,2

Кстати из этого получается должно следовать, что для всех циклов кратных чему-то и если таким-то образом идет чередованием операций то должно быть только это решение, и оно единственное?

>> No.5027089  

>>5027066
т.е. если цикл такой, что идет {нечет, чет} например для случая 1 2 для правила f(x) = if(x is even) {x/2} else {(3x + 1)/2} то тогда это же будет для {нечет чет нечет чет}, {нечет чет нечет чет нечет чет} и так далее

>> No.5027185  
Файл: cixirno.png -(7 KB, 224x224, cixirno.png)
7

>>5027019
>>5027025
Ну, в тех постах ставится много вопросов которые можно обсуждать довольно долго. Я буду высказываться по некоторым из них как человек, полукомпетентный в теории струн (я занимаюсь темами которые имеют косвенное отношение к теории струн, но всё же имеют).

Во-первых про то что "М-теория так же обоснована как и то что яблоко будет падать вниз если его бросить", это вообще немного некорректное утверждение, они находятся на разных уровнях абстракции. Утверждение про яблоки - это некоторое внутретеоретическое предсказание конкретной модели конкретной теории (Ньютоновской гравитации при G равном тому чему надо), а теория струн - это некоторый конструктор теорий, чтобы она вообще начала делать предсказания нужно фиксировать некоторую модель, более-менее никто не знает какую модель нужно фиксировать чтобы получить теорию, которая бы хорошо соответствовала нашему миру. Более того, если фиксировать модели которые более-менее соответсвовали бы тому что называется сейчас "стандартной моделью (в физике частиц)" (например G2-MSSM), то теория струн начинает предсказывать новые эффекты только на энергиях, которые в ближайшем будущем не будут достигнуты ни на каких ускорителях. При такой ситуации сложно говорить что она "так же обоснована", потому что, ещё раз, непонятно как выбрать модель, а даже если её как-то и выбрать, то непонятно как её верифицировать экспериментально.

Во-вторых, при помощи математики делалось много неправильных предсказаний, в ситуациях, когда для рассчёта использовался неправильный фреймворк (а неправильный он потому, что делал неправильные предсказания). Есть много анекдотичных случаев, например теоретическое вычисление космологической постоянной не совпадало с экспериментальным на 120 порядков, потом были предложены всякие решения этого "парадокса" заключающиеся в некоторой модификации стандартного фреймворка, в котором делались вычисления, но пока что нету какого-то консенсусного, насколько я знаю. Математика может только очень точно описать некоторый вычислительный аппарат, но не может сказать насколько он адекватен решаемым задачам (это, конечно, некоторое огрубление, но оно сознательно). И именно поэтому "математически доказать", вообще ничего нельзя, наука немного не так работает, мы сначала строим некоторый фреймворк описывающий некоторый ансамбль интересующих нас эффектов, потом убеждаемся, что он хорошо описывает уже имеющиеся эксперименты, а потом делаем индуктивное предположение о том, что неизученные эффекты он описывает тоже хорошо (которое может оказаться неверным, и в истории физики было много таких примеров).

В-третьих, предположим действительно что теорию струн построили, модель подобрали, все проблемы в ней порешали, построили коллайдеры размером с галактику и проверили какие-то новые предсказания на очень высоких масштабах энергий и они сошлись с теорией, в том числе и какие-то эффекты связанные с потерей энергии из-за вылета в другие измерения при каких-то экстремальных условиях. Значило бы это что параллельные вселенные "действительно существуют"? Ну это как бы вопрос феноменологический, должны ли мы принимать за чистую монету некоторую феноменологию теории струн или мы должны к ней просто относится как к вычислительному фреймворку который не говорит "как оно на самом деле", а просто удачным образом описывает наблюдаемые эффекты? Ну тут уж зависит от мировоззрения, и хорошего ответа нету, обе позиции легитимны, мне кажется.

Далее, какие основания есть. Действительно, с точки зрения внутренней логики теории струн самый естественный вариант это мир на 10-мерной бране вложенный в 11-мерный ансамбль, потому что 11-супергравитация с 1-суперсимметрией устроена теоретически проще, чем маломерные, потому что там много чего сокращаются и появляются хорошие решения, которые в пределе похожи на стандартную модель. Это вроде как самое перспективное направление исследований, которое уже дало много инсайтов, в том числе и в математике. Но всё может резко изменится если придумают что-то получше. В любом случае, теория струн пока самый лучший кандидат на построение адекватной QFT, и, как сказал кто-то из классиков "теория струн знает больше математики, чем мы сейчас", поэтому изучать её явно небесполезно. Но конкретно вот эта история про мультивёрсы она лишь маленький кусочек теории струн и не факт что вообще войдёт в финальную версию окончательной QFT/GUT описывающей наш мир.

Я так думаю.

>> No.5027186  
Файл: strangecirno.jpg -(128 KB, 772x1024, strangecirno.jpg)
128

>>5027066
Да, всё так.

>> No.5027200  

>>5027186
А как узнать, известно ли (описано ли) это уже? Ну т.е. не открыл ли я нераноком что-то новое, что доселе неизвестно тем, кто эту гипотезу Коллатца доказать пытается?

>> No.5027208  
Файл: сisirno.jpg -(36 KB, 400x400, сisirno.jpg)
36

>>5027200
Нужно использовать принцип академической щедрости и считать, что если ты доказал что-то на имиджбордах за 5 минут самыми тривиальными и простыми рассуждениями, то это известно.

А если серьезно, то более-менее никак, нужно читать статьи и общаться со специалистами, чтобы быть вкурсе что известно, а что нет.

>> No.5027305  

>>5027208

>если ты доказал что-то на имиджбордах за 5 минут самыми тривиальными и простыми рассуждениями, то это известно

На Форчонге вполне доказали что-то.

>> No.5027316  
Файл: cirno781.jpg -(179 KB, 1000x1000, cirno781.jpg)
179

>>5027305
Да, известная история, ну, во-первых, там была явная комбинаторная конструкция которая улучшала нижнюю оценку (это вообще единственный способ для аматоров делать вклад в математику, исключения из этого правила очень редки), а не абстрактное сущностное рассуждение. А во-вторых, там всё же какая-то комбинаторная гипотеза пятого эшалона, на которую всем было бы плевать если бы не эта громкая история про доказательство на фочане, а не гипотеза Коллатца, над которой думают математики уровня Терренса Тао.

>> No.5027326  
Файл: yhu.gif -(1707 KB, 480x270, yhu.gif)
1707

>>5027185
А ещё говорят, что сырны баки! Спасибо!

>> No.5027498  

>>5027019ъ
Из картинки:

>Доказательство есть. Оно называется М-теория.

Эмм, разве теория может доказывать что-либо относительно физики реального мира? Автору этих строк не мешало б почитать что-нибудь о методологии науки. Вот взять например закон сохранения энергии. Какая-нибудь теория его реально доказывает? Нет, не доказывает. Вывод, что вечного двигателя скорее всего нельзя построить, сделан на основе кучи неудачных попыток его построить, и потом вписан в теорию как соответствующий закон. В квантовой механике он кстати может на некоторое время нарушаться, но не суть.

Да, теории могут обладать некоторой предсказательной силой. Самый известный пример - периодическая таблица Менделеева - по ней можно было предсказать что вот будут такие-то химические элементы с какими-то такими атомными массами и хим. свойствами, и эти предсказания действительно сбывались. Но это не значит что таблица Менделеева что-то там доказывает, она просто описывает. Притом описание может быть неточным, или точным в определенных пределах, например классическая евклидова геометрия вполне справляется с обычными бытовыми задачами, а когда речь идет про какие-то черные дыры, она уже не работает.

>> No.5027751  
Файл: lych_threads.png -(50 KB, 736x350, lych_threads.png)
50

А можно ли сделать фрактал из проблемы 196, по аналогии как с гипотезы Коллатца фрактал построили https://youtu.be/GJDz4kQqTV4 ?

>> No.5027863  
Файл: ciarno.jpg -(110 KB, 1024x1027, ciarno.jpg)
110

>>5027751
Вряд ли, функция Лихреля в том виде в котором она на вики записана никаким естественным образом на C не продолжается.

>> No.5027876  

>>5027863
Ну можно продолжить неестественно! Вот например как можно записать функцию, которая бы давала число значащих цифр? Скажем, для всех чисел от 0 до 9.9999... вернула 1, для чисел от 10 до 99.9999... вернула 2, для чисел от 100 до 999.9999... вернула три. Это что-то десятичного логарифма, только не совсем

А как сделать функцию разворота числа, принимающую длину в десятичны цифрах и число, возвращающую результат разворота?

>> No.5027976  
Файл: cicirno.jpg -(110 KB, 970x1370, cicirno.jpg)
110

>>5027876
Кол-во цифр в целой части числа x>=1 это floor(log10(x)) + 1 =: k, для ревёрса числа никакой хорошей алгебраической записи нету, только тот вид что на википедии:
f(n) = n + \sum_{i=0}^{k-1} di 10^{k-i-1}
где di = (n mod 10^{i+1} - n mod 10^i)/10^i
ну это как бы плохо записанный естественный алгоритм того, как бы ты реверсал цифровую запись числа на каком-нибудь языке программирования, почти уверен что бесполезная для твоих целей формула.

>> No.5028229  
Файл: 188.png -(241 KB, 800x600, 188.png)
241

>>5027185

>на разных уровнях абстракции

Ясно. У тебя математика имеет "разные уровни абстракции". Тебя не смущает то, что любая модель в математике одинаково абстрактна, какой бы приблежённой она ни была к той реальности, которую мы привыкли видеть? Что простейшая прямая, что тессеракт, что многообразие Калаби — Яу — все эти вещи отвлечены от физического мира, который можно пощупать (by the way, некоторые части физического мира нельзя пощупать: например, пульсары или кварки). Если ты возьмёшь математическую сферу, то ты не сможешь её потрогать, потому что тебе будет доступна только физическая (глобус, мячик, тестикулы). С математической можно взаимодействовать только посредством ручки или мела. Сравнение корректно, потому что нет никаких "уровней абстракции".

>а теория струн - это некоторый конструктор теорий

Не надо вот этих игр в слова. Это набор математических фактов. Фа-ктов.

>чтобы она вообще начала делать предсказания нужно фиксировать некоторую модель

Чем и занимаются физики, математики в MIT и других вузах.

>начинает предсказывать новые эффекты только на энергиях, которые в ближайшем будущем не будут достигнуты ни на каких ускорителях. При такой ситуации сложно говорить что она "так же обоснована"

Ты явно считаешь, что вычисления выеденного яйца не стоят, а достаточная обоснованность — это только такая обоснованность, которую можно получить через эксперимент. Как бы так сказать, не навредив твоей психике? Есть ещё косвенные доказательства и математическое обоснование. Математика тычет нас лицом в то, что это правда > это правда.

>при помощи математики делалось много неправильных предсказаний, в ситуациях, когда для рассчёта использовался неправильный фреймворк

Если ты выбираешь "неправильный фреймворк", то получаешь "неправильные предсказания". Это проблема не математики, а людей, которые выбирают неверные фреймворки. То, что в границах "неправильного фреймворка" работает неправильная математика, говорит лишь о том, что она прекрасно выполняет назначенную ей роль, потому что, ну... она работает, делая неправильные предсказания. Это подобно тому, как из ложной предпосылки ты приходишь к ложному выводу.

>И именно поэтому "математически доказать", вообще ничего нельзя

Пуанкаре в гробу перевернулся, узнав, что Перельман наебал его, когда доказал его гипотезу. Вот хитрец!

>построили коллайдеры размером с галактику

Бака. Чем меньше исследуемый объект, тем больше должен быть коллайдер. Квантовая струна бесконечно мала, а это значит, что ускорителю нужно объять всю видимую Вселенную, а не одну галактику. Самый маленький объект исследуется при помощи самого большого объекта. Красиво.

>предположим действительно что теорию струн построили, модель подобрали, все проблемы в ней порешали [...] и проверили какие-то новые предсказания [...] и они сошлись с теорией
>Значило бы это что параллельные вселенные "действительно существуют"?

Да, потому что если "все проблемы" в теории струн "порешали", то это значит, что была и решена проблема с параллельными вселенными, а проблема эта была в том, что они существовали только на бумаге (всякая научная проблема в конечном итоге сводится к тому, чтобы поставить эксперимент). Как только "все проблемы порешались", то они стали экспериментально доказанными. Параллельные вселенные перешли из фазы "только теория" в "а теперь и эмпирика" > паралелльные вселенные действительно существуют.

>должны ли мы принимать за чистую монету некоторую феноменологию теории струн

Ты не знаешь, что такое феноменология. Феноменология — такие описание и фиксация объекта, которые не претендует на исчерпывающие описание и раскрытие всех возможных причинно-следственных связей выбранного феномена. Если, как ты уже сказал, "все проблемы порешались", что значит "исчерпывающее описание", то теория струн не может быть феноменологической.

>>5027498

>разве теория может доказывать что-либо относительно физики реального мира?

Ещё как может. Специальная теория относительности доказывает, что законы природы одинаковы во всех системах координат, движущихся прямолинейно и равномерно друг относительно друга; что скорость света в вакууме одинакова во всех системах координат, движущихся прямолинейно и равномерно друг относительно друга. Ну и сама теория — это обобщенная система знаний, которая даёт объяснение тем или иным сторонам и свойствам изучаемого предмета или явления. Обрати внимания на "даёт объяснение", что, по сути, и есть доказательство, поскольку нельзя ничего не объяснить, одновременно не доказав это. Ты когда просишь человека объяснить тебе что-то, то он, если не является клиническим идиотом, приводит некоторый ряд аргументов, а аргументы — это и есть доказательство. Объяснил (доказал) красное смещение? Выявил расширение Вселенной. Объяснил (доказал) полупроводниковую электронику? Выявил верность квантовой механики и создал то, с чего ты сейчас сидишь в этом треде и читаешь мой пост.

>Вот взять например закон сохранения энергии. Какая-нибудь теория его реально доказывает?

Закон сохранения энергии ФУНДАМЕНТАЛЕН. Ты слышал про метафизику и теорему Гёделя о неполноте? Метафизика — это физика физики; это даже не физика, это философия, потому что мы не можем узнать, почему наши константы имеют именно такие значения, какие имеют. Для этого нам потребуется покинуть нашу Вселенную и рассмотреть её под лупой сверху, снизу, сбоку — какую угодно позицию выбирай. Закон сохранения энергии из этой оперы. А теорема о неполноте говорит о том, что любая теория строится на неких постулатах, которые сама эта теория обосновать не может. Конечность скорости света никак не доказывается из теории относительности, к примеру. Закон сохранения энергии тоже никак не выводится из других теорий. В рамках теории более высокого уровня (метатеории) этот постулат объясняется, но метатеория сама строится на каких-то постулатах, которые из неё не выводятся, то есть нужна метаметатеория и так до лежачей восьмерки.

>> No.5028248  
Файл: ciwirno.jpg -(116 KB, 1052x1099, ciwirno.jpg)
116

>>5028229
Очень интересное мнение, со всем согласен, я полную ерунду написал, выходит. Только зачем удалять и переотправлять? Редиректы же сбиваются, и некрасиво получается.

>> No.5028295  
Файл: 190.png -(670 KB, 539x1482, 190.png)
670

>>5028248

>Очень интересное мнение, со всем согласен, я полную ерунду написал, выходит.

Это не сарказм? Если да, то был бы рад узнать, что я где-то неправ, дабы приблизить свою точку зрения к истине.

>Только зачем удалять и переотправлять?

Редактировать нельзя — приходится удалять.

>> No.5028296  
Файл: сixirno.jpg -(61 KB, 550x767, сixirno.jpg)
61

>>5028284
Без негатива, но я готов спорить о математике с другими математиками и спорить о каких-то аспектах теории струн с людьми, которые знают теорию струн, да и то, не в тоне: "Ты дурак и написал полную чушь". В этом треде я могу только что-то рассказать, высказать мнение или объяснить, если меня попросят, и поспорить о чём-то в дружелюбном тоне, если буду видеть что человек в этом чём-то минимально разбирается и если минимально разбираться в этом буду я сам.

>> No.5028306  
Файл: 251.png -(671 KB, 696x696, 251.png)
671

>>5028296

>не в тоне: "Ты дурак и написал полную чушь"

Я назвал тебя один раз бакой, но я разложил твой пост по полочкам и ответил на те его части, которые мне показались неверными, с доводами. Не согласен — отвечай, мне интересно будет услышать твоё мнение.

>> No.5028311  

>>5028229

>Ещё как может. Специальная теория относительности доказывает, что законы природы одинаковы во всех системах координат, движущихся прямолинейно и равномерно друг относительно друга; что скорость света в вакууме одинакова во всех системах координат, движущихся прямолинейно и равномерно друг относительно друга.

Нет, физическая теория не доказывает. Физическая теория описывает и предсказывает (иногда правильно, иногда неправильно).

Допустим, мы провели некий физический эксперимент, который опровергает выводы некоторой физической теории. Что дальше будешь делать с этой теорией и ее "доказательствами"? Например, масса нейтрино. Стандартная модель предсказывает, что нейтрино должно не иметь массы, но опыт показывает что масса таки есть, поэтому стандартную модель расширили, чтобы она лучше описывала реальную физику. Сами эти модели ничего доказать не могут.

>> No.5028315  

Зря про овтеты тому чуваку с мультивселенными думали всерьёз, но он ещё и сюда пришёл/настолько же ядрёный.

>>5028295

> Это не сарказм? Если да, то был бы рад узнать, что я где-то неправ, дабы приблизить свою точку зрения к истине.

Ты не прав везде, где разошёлся с чужим мнением, везде где подумал что твоё мнение = всё, чужое мнение = ничто. Чтобы исправить надо использовать эмпатию, узнавать чужие причины, или честно говорить, что работаешь только в рамках конкретной идеологии и не на какую истину без этого не претендуешь.

Идя же по другому пути ты можешь и ещё на 1.000 цитат разбивать чужие слова и ещё более длительными разбалтываниями всех "разбивать".

Ты часто просто пишешь какой-то агрессивный оффтоп (агрессивно подаёшь оффтоп), тебе надо терять в весе:

>>5028229

> Закон сохранения энергии ФУНДАМЕНТАЛЕН. Ты слышал про метафизику и теорему Гёделя о неполноте?

Тебе показалось, что >>5027498 хочет как-то обидеть закон сохранения? Иначе это оффтоп.

> Обрати внимания на "даёт объяснение", что, по сути, и есть доказательство, поскольку нельзя ничего не объяснить, одновременно не доказав это.

По сути это болтология. (И противоречие слеюущему абзацу.)

И, в любом случае, этот ответ имеет смысл только вне контекста.

> Ты не знаешь, что такое феноменология. Феноменология — такие описание и фиксация объекта, которые не претендует на исчерпывающие описание и раскрытие всех возможных причинно-следственных связей выбранного феномена.

Не важно, знает он или нет, он выражал какую-то мысль. И он не пытался оскорбить феноменологию или феноменологов (может, в этом недопонимание), он писал о специфической группе людей со специфическим отношением к феноменологии. Эта группа людей уже как раз и может "претендовать".

> Да, потому что если "все проблемы" в теории струн "порешали", то это значит, что была и решена проблема с параллельными вселенными (1)
> Как только "все проблемы порешались", то они стали экспериментально доказанными. (2)

По-моему он имел в виду другое (не 1 и не 2), ты свой смысл в слова запихнул и его же оспорил.

> Бака. Чем меньше исследуемый объект, тем больше должен быть коллайдер. Квантовая струна бесконечно мала, а это значит, что ускорителю нужно объять всю видимую Вселенную, а не одну галактику. Самый маленький объект исследуется при помощи самого большого объекта. Красиво.

Агрессивный оффтоп.

> Пуанкаре в гробу перевернулся, узнав, что Перельман наебал его, когда доказал его гипотезу. Вот хитрец!

Речь шла не про математику, тут совсем из контекста вырвал.

>Ты явно считаешь, что вычисления выеденного яйца не стоят, а достаточная обоснованность
> Как бы так сказать, не навредив твоей психике? Есть ещё косвенные доказательства и математическое обоснование. Математика тычет нас лицом в то, что это правда > это правда.

А ты уверен, что он говорил о достачной обоснованности? У него там сравнение было.

У тебя опять та же схема: ты выдумываешь для себя триггер и начинаешь кого-то оправдывать. Вот тут тебе показалось, что кто-то оскорбляет другие методы доказательства...

> Не надо вот этих игр в слова. Это набор математических фактов. Фа-ктов.

По-моему и тут тоже, тебя чисто обидело что теорию струн как-то назвали. Он делал сравнение, а ты на это ничего не ответил.

> Ясно. У тебя математика имеет "разные уровни абстракции". Тебя не смущает то, что любая модель в математике одинаково абстрактна, какой бы приблежённой она ни была к той реальности, которую мы привыкли видеть?

По-моему он не говорил о математике там. И по-моему ты опять пишешь оффтоп/пытаешься найти униженных и оскорблённых. Выставить всё так будто он хотел какое-то равенство абстракций нарушить, сделать об этом глобальное заявление....

>>5028306

Ты заставляешь второй раз повторять, что диалога с тобой не хочется, я чего-то не понимаю твоей мотивации. Ты думаешь тебе нужно помочь ему разобраться в своих желаниях? Ведь он итак говорил бы с тобой уже, если бы хотел...

А, ещё ты что-то говорил о психике.

По-моему таких как ты надо раскладывать по полочкам, самым низким.

Беги, беги самыми тёмными лесами и зарывайся в самые глубокие ямы и молись что у тебя когда-то будет шанс вернуться назад как равному тем кого ты хотел "раскладывать".

>> No.5028328  

>>5028326

>То, что доказательства всегда временные, не отменяет того, что это доказательства.

Это доказательства в рамках модели. Модель может быть неправильной. Мы можем найти новые экспериментальные факты, противоречащие ей, на основе этих новых фактов придумываем новые модели, которые не противоречат новым экспериментальным фактам и что-то новое могут предсказывать. Все подобные теории про настоящую реальную физику ничего строго доказать не могут, они только предсказывают и описывают, притом не всегда правильно.

Так что утверждение, что М-теория именно доказывает существования параллельных миров в реальной физической действительности - оно ошибочно. Ч.Т.Д.

>> No.5028933  

Если в wolframalpha ввести функцию e^x то он пишет про нее что она periodic in x with period 2 i π :
https://www.wolframalpha.com/input/?i=e%5Ex

Что это вообще значит? Что значит периодичность с комплексным периодом? А гиперкомплексный период бывает?

>> No.5028935  
Файл: ciqirno.jpg -(96 KB, 640x640, ciqirno.jpg)
96

>>5028933
Если функция определена на комплексных, то она может быть периодична относительно сдвигов на любое комплексное число, или даже на два сразу, ну потому что комплексная плоскость она двумерна над R. В частности утверждение о периодичности экспоненты следует из т.н. формулы Муавра. Тут даже дело не в комплексности, ты можешь определить функцию на плоскости (не воспринимая её как комплексные числа) и потребовать от неё переодичности относительно сдвигов на определённый вектор.

Функции над разными гиперкомплексными числами тоже могут быть периодичны, в частности кватернионная экспонента периодична относительно сдвигов на все 3 мнимые единицы. Нно гиперкомплексные числа это некоторая алгебраическая экзотика, которая представляет интерес только для очень небольшого числа специалистов.

>> No.5029656  

>>5027498

>Какая-нибудь теория его реально доказывает?

Йеп. Гугли "теорема Нётер".

>> No.5029693  
Файл: -3-638.jpg -(69 KB, 638x479, -3-638.jpg)
69

>>5029656
Ну так это доказательство на основе неких других аксиом, которые доказать уже никак нельзя (т.е. мы не можем точно знать, правильно ли мы угадали "аксиомы реального мира", чтоб на основе них что-то потом доказывать). Ну и к тому же закон сохранения энергии может на некоторое время нарушаться, почитай про туннельный эффект, ну и вот видео https://www.youtube.com/watch?v=CcT_PlX46KM например

>> No.5029810  
Файл: cinoethirno.jpeg -(10 KB, 255x197, cinoethirno.jpeg)
10

>>5029656
Теорема Нётер вообще никакие законы сохранения не доказывает, она говорит (в одной из формулировок), что если есть закон сохранения в данной гамильтоновой системе (которыми обычно кодируют системы в классической механике, но не всегда), и есть симметрия которая его сохраняет, то эта симметрия будет сохранятся и на протяжении всей эволюции системы.

>> No.5029974  
Файл: Mion_school.jpg -(56 KB, 337x1000, Mion_school.jpg)
56

Просел по морали, подбодрите.
Мне нравится учить матёшу, но вижу вокруг себя много шарящих ребят и понимаю, что до их уровня тяжело, даже, наверное, невозможно, дорасти. Делаю из этого вывод, что учить её нужно для себя, но пробиваются глупые мысли о том, что из каждого занятия нужно извлекать пользу, а если такого нет, то отбросить.
Алсо, хочу связать жизнь с математикой, но очень глупый: медленно соображаю. Люди на семинарах соображают быстрее. Думаю войти куда-нибудь в прикладную: биоинформатика, AI или ещё куда-то. Однако понятия не имею куда копать, как составить представление об этом. На моём направлении больше программирования и оттуда в основном выпускается в макакинг, а я хочу что-то интеллектуальное.
Сколько времени нужно уделять математики в день/неделю? В принципе, если могу увлечься, то могу без перерыва сидеть, но в отсуствии чётких целей и мотивации блуждаю, как овечка в тумане.
Есть ещё один мой серьёзный недостаток. Мне 20 лет, и, на данный момент, знаю только матан 1 семестра, ангем и немного матлога, совсем чуть-чуть. Из вольного прослушивания курсов в НМУ слышал ещё поверхностно об общей топологии и алгебры, но мне туда немного стыдно ходить, там все такие умные и вежливые, а я такой глупый и не работаю по 5/6 часов.

>> No.5029996  
Файл: cihirno.jpg -(56 KB, 512x512, cihirno.jpg)
56

>>5029974
Ох, не знаю честно что посоветовать, я всегда себя не очень уютно в роли лайфкоуча чувствую. Могу только сказать, что не нужно быть гением чтобы заниматься наукой профессионально, есть много ниш, где деятельность довольно вычислительная, нужно выучить набор некоторых трюков и применять их по кд, и получать статьи которые даже будут в неплохие журналы попадать. А соображать быстрее тебя кто-то будет всегда, расстраиваться из-за этого не нужно. Программирование хорошее в ШАДе, чтобы туда поступить только матан и линал и нужно знать, но правда довольно глубоко, задачи на экзамен там не сложные, но требуют понимания. Времени нужно уделять чем больше, тем лучше, если конечно хочешь попробовать влиться в академическую деятельность или вообще чему-то научиться. В качестве мотивации можно взять себе сдать какой-нибудь курс НМУ. В конце концов, базовые курсы, типа анализа или линейной алгебры, тебе помогут даже если ты решишь уйти во что-то прикладное. Если что-то непонятно, в конце концов можно спросить здесь, тред тут по факту полумёртвый почти всегда. Добра тебе.

>> No.5030092  

>>5029810
А что означает симметрия применительно к физическим теориям? Что-то про изотропность пространства, или смысл шире?

>> No.5030124  
Файл: citirno.jpg -(83 KB, 728x442, citirno.jpg)
83

>>5030092
Ну, гамильтонову механику естественнее всего формулировать на языке симплектической геометрии, там фазовое пространство - это симплектическое многообразие М, гамильтониан - фиксированная функция Н на нём, а "симметрия" это действие некой группы Ли G на многообразии симплектоморфизмами. Теорема Нётер о том. что если гамильтониан инвариантен относительно действий G, то отображение моментов J : M -> g* инвариантно относительно потока порождённого векторным полем {H,?}.

Изотропность - это не совсем то.

>> No.5030131  
Файл: citirno2.jpg -(110 KB, 728x870, citirno2.jpg)
110

>>5030092
Если говорить проще, то не к "физическим теориям", а к "определенному классу диф. уравнений", там смысл "симметрии" в том, что ты можешь делать некоторые замены переменных так, что уравнения не меняются. Геометрия стоящая за определенные симметрии в определенных диф.уравнениях может быть проинтерпретирована как изотропия пространства, (в котором живут "объекты" рассматриваемой динамической системы, например), а другие симметрии в других диф.уравнениях могут быть проинтерпретированы как-то иначе. Думаю объяснил непонятно, ну и ладно.

>> No.5030262  

Почему нельзя делать преобразование sqrt(x) = sqrt((-1)·(-x)) = sqrt(-1)·sqrt(-x) = i·sqrt(-x) = i·sqrt((-1)·x) = i·i·sqrt(x) = -1·sqrt(x) ?

Хотя вроде бы понятно почему, но какое конкретно правило это делать запрещает?

>> No.5030265  

>>5030262
Нужно видимо делать ограничение, что sqrt(a·b)=sqrt(a)·sqrt(b) можно делать только если a, b не являются оба отрицательными, но откуда это вытекает?

>> No.5030278  

>>5030262
Но корень из x это +-sqrt(x), всё верно.

>> No.5030282  
Файл: MSP19951hicc967f2fc8b4600002d48fi5877e5h(...).gif -(3 KB, 200x196, MSP19951hicc967f2fc8b4600002d48fi5877e5h(...).gif)
3

>>5030278
Но тогда sqrt(x)=-sqrt(x) а это уже ерунда какая-то получается. В курсе школьной алгебры квадратный корень не определяли как многозначную функцию
https://www.wolframalpha.com/input/?i=sqrt%28a*b%29%3Dsqrt%28a%29*sqrt%28b%29 говорит что sqrt(a·b)=sqrt(a)·sqrt(b) равенство верно только если хотя бы один из сомножителей a b положительный

Нужно тогда вводить специальную функцию квадратного корня, которая возвращает не одно значение, а два.

>> No.5030288  

>>5030282

>В курсе школьной алгебры квадратный корень не определяли

На отрицательных числах.

>Нужно тогда вводить специальную функцию квадратного корня, которая возвращает не одно значение, а два.

e^((2n+1)*pi*i)=-1, корень - возведение в степень 0.5, готово.

>> No.5030300  
Файл: cijirno.jpeg -(6 KB, 225x225, cijirno.jpeg)
6

>>5030262
Математика в другую сторону работает: по-умолчанию запрещено, если нету правила, которое это разрешает. Правило sqrt(ab)=sqrt(a)sqrt(b) есть такое в случае a>=0 b>=0.

Но тут уже сказали причину, у ненулевого комплексного числа всегда два корня, и нету никакого способа непрерывно выбрать один из них, как в положительно-вещественном случае, когда можно было выбрать просто положительный. Поэтому как такового (как однозначно определённой функции) функции sqrt на комплексных вообще нет, это абьюз нотации.

Чтобы с этим справится можно рассматривать sqrt либо как многозначную функцию, либо как функцию на двулистном накрытии над C разветвлённом в нуле (т.н. "поверхность Римана" квадратного корня), то есть всегда выбирать ветвь, прежде чем брать какие-то корни. Обычно когда пишут корни (как, скажем, в формуле Кардано) то предполагают, что они особым образом между собой согласованы и из контекста понятно каким.

>> No.5030357  

ОП, если не секрет, а чем ты занимаешься, какие планы, цели?
Судя по ответам, знаком с компланом и теормехом. Ими?

>> No.5030361  
Файл: cirno940.jpg -(85 KB, 728x582, cirno940.jpg)
85

>>5030357
Занимаюсь алгебраической геометрией и в частности теорией Ходжа, в разных её проявлениях, но по верхам знаю много чего ещё, стараюсь быть всесторонне развитым, так сказать. Слишком много о себе говорить не хочу, так что в некотором роде секрет.

>> No.5030558  
Файл: GammaAbsSmallPlot.svg.png -(581 KB, 1280x1042, GammaAbsSmallPlot.svg.png)
581

>>5027863

>Вряд ли, функция Лихреля в том виде в котором она на вики записана никаким естественным образом на C не продолжается.

А как сумели продолжить на C факториал, что получилась гамма-функция? Как надо рассуждать, исходя из определения факториала, чтоб придумать эту гамма-функцию?

>> No.5030566  
Файл: gamma.jpg -(145 KB, 684x262, gamma.jpg)
145

>>5030558

Бака-вопрос: в чём эстетика и фишка комплексных чисел?

Вроде говорят, что просто сделали 2D, но уже в следующий момент начинается какая-то разноцветность 4-х-мерная и какие-то заводские трубы уходящие бесконечно вверх, это норма?

>> No.5030587  
Файл: cisirno.jpg -(73 KB, 728x504, cisirno.jpg)
73

>>5030558
С факториалом в некотором смысле повезло, можно доказать что есть единственная аналитическая на R^+ формула, которая положительная, лог-выпуклая и продолжает (n+1)!, ну а дальше её найти - это дело интуиции. Не знаю как исторически до неё догадались, но она сама по себе довольно естественная, так что это было несложно.

Вообще есть способ продолжить любую последовательность чисел a_n до всюду голоморфной функции (это вроде называется интерполяционной формулой Прингсхейма) взяв что-то типа f(z) = \sum_n a_n sin pi z/(pi cos (pi n) (z-n)), ну и может умножить общий член на что-то хорошее, чтобы ничего не поломать и гарантировать сходимость в каждой точке, проблема в том что такие функции не выделенные чем-то и не очень естественные (как минимум потому что в их построении есть произвол: нужно выбрать на что хорошее поумножать чтобы гарантировать сходимость) поэтому вряд ли дадут какую-то полезную информацию о последовательности (в случае с факториалом не так - там функция выделена набором аксиом, которые я написал).

>>5030566
Ну они с алгебраической точки зрения ведут себя лучше чем вещественные, любой многочлен имеет корень, и такое всё, поэтому при прочих равных лучше использовать их и, скорее всего, все конструкции будут выглядеть проще и красивее, это подтверждается многолетним опытом.

Про трубы и цвета, всё очень просто: так как комплексные числа это плоскость, то график функции естественным образом лежит в C x C, то есть в четырёхмерном пространстве, графики в четырёхмерном мы рисовать не умеем, поэтому придумали следующий способ визуализации. Любое комплексное число однозначно определено своим модулем и фазой, поэтому рисуют график |f(z)| который уже лежит в C x R (а, значит, трёхмерен), а фазу подкрашивают цветом: фаза, это точка на окружности, раскрасив окружность в разные цвета мы можем точку (z,|f(z)|) которая лежит в трёхмерном C x R подкрасить цветом, соответствующим Arg (f(z)), и получаем цветной трёхмерный график. Это в некотором смысле очень естественно: эти "трубы" показывают что функция вблизи этих труб уходит в бесконечность, то есть мы понимаем что там особенность, а по тому насколько резко меняются цвета вблизи этих труб мы можем судить о характере особенности (это полюс или существенная сингулярность).

Как-то так.

>> No.5030706  

>>5030587

>ну а дальше её найти - это дело интуиции

Что ты понимаешь под интуицией? Ну вот как конкретно находить?

>> No.5030730  
Файл: civirno.jpeg -(10 KB, 220x229, civirno.jpeg)
10

>>5030706
Ну сложно ответить. Нужно использовать что (d/dx)^n x^n = n!. А именно интегрируя \int x^n f(x) dx по частям можно заметить \int x^n f(x) dx = x^(n+1)/(n+1) f(x) - \int x^(n+1)/(n+1) f'(z) dz, читая справа налево получаем
(n+1) \int x^n f(x) dx + x^(n+1)/(n+1) f(x) = - \int x^(n+1) f'(x) dx
сдвигая индексы получаем
n \int x^(n-1) f(x) dx + x^n f(x) /n = - \int x^n f'(x) dx
теперь если бы у нас была такая функция, что f'(x) = - f(x) и lim(x->inf) x^n f(x) = 0 мы бы победили, потому что тогда мы бы получили тождество
n \int x^(n-1) f(x) dx = \int x^n f(x) dx
и применяя его n раз мы бы получили
\int x^n f(x) dx = n! \int f(x) dx а это как раз то что нужно, такая функция есть и это e^(-x), ну как-то так.

>> No.5030754  

Что такое функтор?

>> No.5030759  
Файл: cidirno.jpeg -(9 KB, 225x225, cidirno.jpeg)
9

>>5030754
Ну там определение само за себя говорящее. Любая естественная конструкция, которая строит по одному математическому объекту другой чаще всего функтор. Скажем, построить кольцо матриц размера n по кольцу - это функтор. На самом деле довольно сложно понять зачем они нужны и почему это определение полезное, а не "пустое", не имея некоторого бэкграунда в алгебре, но у функторах есть некоторые общие теоремы (скажем, теорема о сопряжённом функторе), которые позволяют тебе задёшево доказывать много технических утверждений. Ситуация примерно как с группами: сложно понять почему определение "группы" не пустое не имея некоторого бэкграунда, но есть некоторые общие теоремы о группах, скажем, теорема Лагранжа, позволяющие тебе задёшево доказывать много полезных, хоть и простых конкретных утверждений (скажем, из неё задёшево следует что если n взаимнопросто с m, то n^\phi(m) = 1 (mod m) ). И вместе с тем группы являются удобным языком для формулировки идеи "симметрии" во многих контекстах. Точно так же функторы являются удобным языком для формулировки идеи "естественной конструкции" во многих контекстах.

>> No.5031265  
Файл: __the_math_sweater__gauss___by_serketsta(...).png -(103 KB, 765x1044, __the_math_sweater__gauss___by_serketsta(...).png)
103

>>5030754
Структура - это когда на объекте (множестве) заданы какие-то операции. Например, кольцо - объект со сложением и умножением, связанными раскрытием скобок; векторное пространство - объект со сложением и операцией умножения на скаляр.

Между объектами со структурой бывают функции, согласованные со структурой. Между группами - гомоморфизмы групп. Между кольцами - гомоморфизмы колец. Между векторными пространствами - линейные функции. И т.п.

Категория - набор объектов с какой-то единообразной структурой, между которыми есть сколько-то функций, согласованных с этой структурой (для краткости эти функции называют морфизмами). Например, три кольца и десяток кольцевых гомоморфизмов (маленькая категория), или все существующие конечномерные векторные пространства над R и всевозможные линейные функции между ними (о-очень большая категория).

Категорию можно воображать ориентированным графом, вершинами которого являются объекты, а стрелками - морфизмы. Вершин может быть конечное или бесконечное количество (или даже запредельно-бесконечное). Между двумя вершинами в принципе может быть много стрелок, даже бесконечно много. Или их может вообще не быть.

В категории есть операция композиции морфизмов. Она заменяет путь из A в Z вида A→B→C→...→Y→Z единственным морфизмом A→Z. На эту операцию композиции по определению категории наложены такие требования, что в категориях возможны вычисления, очень напоминающие алгебраические. Об операции композиции часто можно думать как об обычном последовательном применении нескольких функций. В принципе, категории очень напоминают моноиды.

Функтор - это морфизм категорий. То есть это функция из одной категории в другую, которая переводит объекты в объекты, морфизмы в морфизмы и при этом согласована с операцией композиции.

Имеется категория, объектами которой являются категории, а морфизмами - функторы. Она называется Cats.

>> No.5031313  
Файл: cicicirno.jpg -(147 KB, 1400x1050, cicicirno.jpg)
147

>>5031265

>Имеется категория, объектами которой являются категории, а морфизмами - функторы. Она называется Cats.

Малые категории, а так-то всё так.

>> No.5031373  

>>5031313
Просто не очень люблю рассказывать о таких нюансах. К тому же до конца не знаю, как именно соотносятся аксиома Тарского и большие кардиналы.

>> No.5031397  
Файл: cirno15000.jpg -(1765 KB, 2577x3437, cirno15000.jpg)
1765

>>5031373
Это да, возится с этим неприятно, но если по-честному то нужно. Аксиома Тарского это и есть аксиома о больших кардиналах. Говоря более строго, теории ZFC+"для любого x есть унивёрсум гротенидика U т.ч. x \in U" и ZFC+"для любого кардинала \kappa есть cильно недостижимый кардинал строго больший \kappa" proof-theoretical эквивалентны (т.е. доказывают одни и те же утверждения).

>> No.5031449  
Файл: unnamed.png -(14 KB, 445x295, unnamed.png)
14

А есть ли что-то типа интегрирования, только чтоб не суммировать, а перемножать?

>> No.5031454  

>>5031449

https://en.wikipedia.org/wiki/Product_integral
https://math.stackexchange.com/questions/1140507/is-there-a-geometric-interpretation-of-the-product-integral

Присоединяеюсь к тебе/к этому господину!

>> No.5031496  

Кстати, а что если сделать как интеграл, но непрерывное среднее арифметическое? Ну например если надо неопределенное непрерывное среднее арифметическое sin(x)^2 сделать, то это будет:

\lim_{a \to \infty}\left (\frac{\int_{-a}^{a}sin(x)^{2}dx}{2a}  \right ) = 0.5

Такое уже придумали? Как это называется?

>> No.5031514  
Файл: cizizirno.png -(780 KB, 1000x1412, cizizirno.png)
780

>>5031454
Так по ссылкам же всё написано, мне добавить особо нечего, exp \int ln f dx довольно естественное определение для того что ты хочешь, мне кажется.

>>5031496
Да "средним" и называется. Если f это характеристическая функция какого-то множества, то называют "плотностью" или "асимптотической плотностью" ещё.

>> No.5031591  
Файл: sym_attractors.jpg -(1077 KB, 2002x1338, sym_attractors.jpg)
1077

https://examples.pyviz.org/attractors/attractors.html

>> No.5031644  
Файл: Sonozaki.Shion.600.273174.jpg -(64 KB, 428x600, Sonozaki.Shion.600.273174.jpg)
64

Подскажите книг по диофантом. Хочу руку набить.

>> No.5031648  
Файл: cihirno.jpeg -(7 KB, 198x254, cihirno.jpeg)
7

>>5031644
Вот это вопросы. Не знаю хороших книжек для олимпиадников если честно, особо их не читал никогда, может кто-то другой посоветует.

С тз "высокой науки" какого-то одного подхода к диофантовым уравнениям нет, есть много разных техник как решать те или классы диофантовых уравнений, проще всего начать наверное с теории Галуа (я учился по Milne "Fields and Galois theory", мне кажется хорошая), в частности она исследует существование/отсутствие рациональных решений у уравнений типа "многочлен с рациональными коэффициентами от x = 0". Ну и вообще, это стартовая точка для изучения теории чисел (после линейной алгебры и конечных групп, конечно, хотя можно и параллельно), если будешь учить более сложные штуки - теорию ивасавы / теорию минковского и тд, то там знание теории Галуа предполагается по-умолчанию.

>> No.5031685  

Вот есть такая последовательность - https://oeis.org/A051037 - называется 5-гладкие числа. Вот https://wandbox.org/permlink/ueNvdMzwbZQzqnny такую программу я написал, она подряд выводит такие числа. Функция modndiv() имеет какое-то математическое название, может "остаток от логарифма" или типа того? Смысл ее в том, что она исключает такое-то число из факторизации, ну скажем если у нас есть число вида 5^10 x 7^11 x 13^2 и мы делаем modndiv(5^10 x 7^11 x 13^2, 7) то получим 5^10 x 13^2 т.е. там мы делим на эту 7 пока оно делится без остатка.

>> No.5031744  

Мой вопрос:
https://en.wikipedia.org/wiki/Gibbs_phenomenon

Много ли в математике таких штук, которые называются аж "феноменами", подобно физике?

>>5031591

Сам собрал картинку? Тогда это как фанарт к треду.
>>5031514

> Так по ссылкам же всё написано

Ура, я нашёл верные ссылки и помог кому-то! Мог бы помочь то есть...

> Да "средним" и называется

Злорадствую неудаче чужого вопроса!

Противоположные эмоции, если задуматься... уполз в нору.

>> No.5031767  

>>5031744

> Сам собрал картинку? Тогда это как фанарт к треду.

Лишь скопировал оттуда, увы.

>> No.5031828  
Файл: cigirno.jpg -(60 KB, 630x630, cigirno.jpg)
60

>>5031685
Не помню такой.
>>5031744
https://ncatlab.org/nlab/show/Stokes+phenomenon
https://ncatlab.org/nlab/show/Laurent+phenomenon
https://en.wikipedia.org/wiki/Runge%27s_phenomenon
Это всё что вспомнил, вообще - не много.

>> No.5032207  

>>5030759
А сопряженные функторы?

>> No.5032657  
Файл: cirnoicecream.jpeg -(206 KB, 811x1147, cirnoicecream.jpeg)
206

>>5032207
Ну снова же, это сложно почувствовать не имея бэкграунда и пачки примеров. "Быть сопряжённым"и это вторая хорошая вещь с двумя функторами которая может произойти, после "быть взаимообраными". Это обобщение (или, как ещё говорят "категорификация") идеи "двойственности Галуа". В математике часто бывают ситуации когда по одному объекту строится другой таким образом, что маленькие подобъекты первого объекта ставятся в соответствие большим подобъектам второго объекта. Самый простой пример с векторными пространствами, имея конечномерное векторное пространство V, можно построить двойственное пространство V* такое, что k-мерным подпрастранствам V естественным образом соответствуют n-k-мерные подпространства V*. Сила сопряженных функторов в том, что они встречаются чаще, чем можно было бы подумать, и если функтор имеет сопряженный то он ведёт себя очень хорошо (коммутирует с пределами или копределеами), что позволяет доказывать и ёмко формулировать кучу простых технических утверждений, которые с перспективы сопряжённых функторов становятся очевидными (скажем, что тензорное произведение точно справа, а хом точен слева в категории R-модулей). Ну как-то так, какой-то классик говорил следующее: "Основная задача категорной алгебры - делать простым и очевидным то, что на самом деле является простым и очевидным", думаю, что слова мудрые.

>> No.5032831  

>>5031828

https://arxiv.org/pdf/1912.06462.pdf
https://www.reddit.com/r/explainlikeimfive/comments/1700nr/eli5_padic_numbers/

Можешь ли ты объяснить их теорию/идею доступно простым людям?

Просто это Sabine Hossenfelder (блогер по физике) и Tim Palmer, какой-то сумасшедший погодчик (так описано в одном посте о встрече: человек связанный с погодой вдруг захотел обсуждать супер-детерменизм)
http://backreaction.blogspot.com/2019/12/the-path-we-didnt-take.html

Что там за фрактал, что одни состояния лежат на этом фрактале, а другие не лежат?

Вроде понял (условно), что такое p-adic числа, но как/зачем они там?

Спасибо!

>>5031767

Но там прямо такой картинки нет? просто обратил внимание так как сам недавно делал много-много коллажей, коллаж надо не полениться и собрать.

>> No.5034082  

Верно ли, что если из доказуемости утверждения X следует недоказуемость утверждения X, то утверждение X недоказуемо?

>> No.5034086  

X->~X = 1
~X v ~X = 1
~X = 1 <=> X = 0
Ну я не уверен, что мы можем так делать.

>> No.5034355  
Файл: cirnoday.jpeg -(10 KB, 290x174, cirnoday.jpeg)
10

>>5032831
Сори, слишком долго статью на архиве ту читать, пока нет времени, может как-нибудь вернусь.
>>5034082
Да, ниже в принципе написали почему, из A -> не А, выводится не А.

>> No.5034590  

>>5034082
Зависит от определений. Если "недоказуемость" - невозможность написать в реальном мире строку символов, пользуясь некими конкретными правилами, а "следует" означает логическое следование классической логики, то да.

Иначе рассмотрим утверждение "ZFC непротиворечива", а под доказуемостью будем понимать доказуемость средствами ZFC. Если это утверждение доказуемо, то ZFC непротиворечива, поэтому по теореме Гёделя это утверждение недоказуемо. Но оно может быть доказуемым, если ZFC противоречива.

>> No.5034703  

>>5034355

Прости, это совсем бредовый вопрос был! (переводить для деревенских последние новости супердетерменизма) Но если серьёзно, есть ещё обзорные статьи (записи) какие-то:

http://backreaction.blogspot.com/2020/02/guest-post-undecidability.html
http://backreaction.blogspot.com/2020/02/guest-post-undecidability_10.html

Хотел спросить как ты из-за альтруизма и из-за корысти (готовлю всякие штуки, которые идут с видео, внутри которого анимация, про которую когда-то сказал!)

>> No.5034794  
Файл: cirno22000.jpeg -(10 KB, 230x219, cirno22000.jpeg)
10

>>5034590
Да, не, не зависит, даже в твоём примере. Если ZFC противоречива то там просто не выполняется "из доказуемости Con(ZFC) следует недоказуемость Con(ZFC)", ну потому что Con(ZFC) там доказуемо, если ZFC непротиворечива, "из доказуемости Con(ZFC) следует недоказуемость Con(ZFC)" там выполняется, потому что Con(ZFC) там недоказуема и вывод о том, что Con(ZFC) недоказуемо всё равно корректен. Теоремы Гёделя и всякая метаматематика в данном случае не нужна, (A -> not A) -> not A это пропозиционная тавтология её истинность не зависит от того, насколько сложное устройство имеет подлежащее утверждение A и в какой логической системе мы это рассуждение проводим (по крайней мере до тех пор, пока в этой логической системе работает пропозиционная логика).

>>5034703
Ага, посмотрю, дела потихоньку идут, есть локальные проблемы, но они вроде как решаемы.

>> No.5034970  

Вот взять например про теорему Гудстейна. Ну допустим вот есть функция Goodstein(x) которая определена для множества натуральных чисел без нуля. И вот, получается что нельзя доказать в PA тотальность такой функции т.е. что для всех натуральных она вычислима? Ну ок допустим мы придумаем расширение PA и добавим в нее аксиому, что Goodstein(x) не тотальна (т.е. что есть такое натуральное x больше нуля, при котором функция гудстейна невычислима) - но тогда наверно надо найти и сконструировать такое x, доказать что значение функции Goodstein(x) в этой точке не определено, иначе это какая-то ерунда получается!

>> No.5034973  

>>5034970
И кстати, если функцию Goodstein(x) определять как число тех вот степенных преобразований (или как правльно это назвать???), после которых число приходит к 0, то можно ли эту функцию расширить на комплексные числа?

>> No.5034979  
Файл: cirnochibi.png -(325 KB, 1300x1300, cirnochibi.png)
325

>>5034970
Твоё рассуждение бы работало, если бы PA описывало бы только "настоящие" натуральные числа с естественными операциями сложения и умножения, но у PA есть очень много моделей, бесконечно много, добавив "Goodstein не тотальна" ты просто, на уровне моделей, выкинешь стандартную модель ("настоящие натуральные числа") и ещё какие-то (все в которых Goodstein была тотальной), но остануться много нестандартных моделей, в каждой из которых будет "нестандартное число", большее всех стандартных натуральных чисел, на которой Goodstein будет зацикливаться.
>>5034973
Я уже писал раннее в >>5030587, продолжить можно любую функцию f : N -> N то всюду голоморфной функции C -> C, проблема в том, что таких продолжений много, и чаще всего ни одно из них ничем особо не выделенно, поэтому сомнительно что они дадут какую-то полезную информацию об исходной функции, определённой на натуральных. Сколь-нибудь разумного и естественного продолжения, насколько я знаю, нет.

>> No.5034980  

>>5034979

>Твоё рассуждение бы работало, если бы PA описывало бы только "настоящие" натуральные числа с естественными операциями сложения и умножения, но у PA есть очень много моделей, бесконечно много

А как называется PA, которая описывает только "настоящие" натуральне числа? Какая там аксиоматика? Опровержима ли там теорема Гудстейна? Зачем вообще PA с ненастоящими натуральными числами?

>> No.5034982  
Файл: strangecirno.jpg -(6 KB, 200x200, strangecirno.jpg)
6

>>5034980
Называется True arithmetic, Th(N). Потому что, согласно теореме Гёделя, такая теория не может быть перечислимой, в том смысле что множество её аксиом не может быть сгенерировано никаким алгоритмом (оно даже не арифметично, согласно теореме Тарского о выразимости истины), то есть это такая, очень "абстрактная теория" существующая в мире идей. Такие теории тоже исследуют, но на равне с этим хотелось бы иметь какую-то вычислимую/перечислимую альтернативу, в которой можно делать математику (ну или хотя бы её большой кусок), потому что это в некотором смысле то как работают математики: они из вычислимого множества аксиом по каким-то заранее заданным правилам вывода генерируют теоремы. Арифметика Пеано представляется разумной для этих целей. Теорема Гудстейна истинна (или, что то же самое, доказуема в True arithmetic).

>> No.5034985  

Давайте посмотрим определение натуральных чисел. Во-первых, натурльное число это 0 (иногда бывает что начинаем с 1 но не суть), и во-вторых что есть у нас функция inc(x) и если в нее подставить натуральное число, то она вернет натуральное число. Таким образом, можно перечислить все натуральне числа: 0 inc(0) inc(inc(0)), inc(inc(inc(0))) и так далее. Если у нас есть нечто, что в таком виде не может быть представлено, на каком основании мы считаем это натуральным?

>> No.5034987  
Файл: cirnosnow.png -(1607 KB, 1736x2400, cirnosnow.png)
1607

>>5034985
Всё так и есть, проблема в том, что предикат "представимо в виде inc(inc(...inc(0)...))" не выразим в языке арифметики первого порядка и в любых его разумных расширениях, это снова же теорема Гёделя. Натуральные числа называют натуральные числами, нестандартные точки нестандартных моделей PA так и называют (ещё иногда называют "нестандартными натуральными числами в данной модели"), никто эти вещи не путает.

>> No.5034998  

>>5034987

>Всё так и есть, проблема в том, что предикат "представимо в виде inc(inc(...inc(0)...))" не выразим в языке арифметики первого порядка и в любых его разумных расширениях, это снова же теорема Гёделя.

А как такой предикат называется?

Допустим, есть у нас некая модифицированная PA с таким предикатом, и есть аксиома что N это те, которые представимы как inc(inc(...inc(0)...)) и там мы формулируем теорему Гудстейна. Доказуемо ли там это?

Допустим, недоказуемо и допустим мы вводим новую аксиому : теорема Гудстейна неверна, и существует такое inc(inc(...inc(0)...)) что если его подставить в goodstein(x) функцию, которая возвращает число шагов чтоб прийти к 0 - то там будет зависание, т.е. goodstein(x) функция нетотальна. Тогда нам ведь надо предъявить, что вот есть такое-то inc(inc(...inc(0)...)) что вот если его туда подставить, то тогда оно зависает, так ведь? Или я что-то не так понял?

>> No.5035008  
Файл: cirnopaint.png -(463 KB, 1280x1024, cirnopaint.png)
463

>>5034998
Не очень ясная конструкция, если мы вводим новый предикатный значок \phi(n) и сужаем класс допустимых интерпретаций до тех, в которых \phi(n) интерпретируется как "представимо в виде inc(...inc(0)...)", то мы теряем полноту нашей подлежащей логики - перестаёт быть верным то, что предложение доказуемо тогда и только тогда, когда истино во всех моделях, и не приобретаем никаких новых средств для доказательств - потому что мы не добавили никаких новых аксиом на \phi (как я уже говорил, аксиому "\phi(n) <-> n представимо inc(...(0)...)" на языке арифметики первого порядка написать нельзя, потому что теорема Гёделя). Поэтому в новой системе доказуемо будет всё то, что было доказуемо в старой, в частности теорема Гудстейна будет не доказуемой. Если очень хочется доказать тотальность функции Гудстейна, то достаточно к РА добавить индукцию до \eps_0.

>> No.5035024  

>>5035008

>(как я уже говорил, аксиому "\phi(n) <-> n представимо inc(...(0)...)" на языке арифметики первого порядка написать нельзя, потому что теорема Гёделя)

А где представимо?

>Если очень хочется доказать тотальность функции Гудстейна, то достаточно к РА добавить индукцию до \eps_0.

Откуда мы знаем, что такая индукция это правильно?

>> No.5035034  
Файл: cirnokiss2.jpeg -(11 KB, 254x198, cirnokiss2.jpeg)
11

>>5035024
В арифметике второго порядка c полной семантикой подлежащее множество любой модели - это стандартные натуральные числа. Поэтому там этот предикат будет выразим просто как тождественно истинный. Но, там, снова же, не все арифметические истины доказуемы (хотя конкретно теорема Гудстейна доказуема) и перестаёт работать теорема о полноте, то есть если есть вычислимая функция f тотальность которой не доказуема, совсем не факт что найдётся натуральное число, на котором вычисление функции будет зацикливаться. Из-за этих недостатков арифметику второго порядка с полной семантикой никто особо серьезно не рассматривает. В любом случае, ограничения связанные с теоремой Гёделя не бюрократические, а фундаментальные, и просто сменой и тасовкой слов их решить нельзя, с ними просто надо учиться как-то жить.

>Откуда мы знаем, что такая индукция это правильно?

Тут есть два ответа (оба неудовлетворительные, к сожалению). Во-первых, это доказуемо в других системах расширяющих арифметику, в адекватность которых мы верим (арифметика второго порядка с семантикой Хенкеля, теория множеств ZFC), во-вторых, индукция до какого-то рекурсивного ординала - это такое утверждение в которое мы "хотим верить", потому что оно кажется нам очень убедительным в психологическом плане, все математики в каждое из них верят (кроме всяких экстремистов типа финитистов/ультрафинитистов, но те во много чего не верят и помимо индукции до рекусривных ординалов).

>> No.5035036  

>>5035008
А если б было такое конкретное натуральное число (ну типа 872907648972356), для которого функция Гудстейна не завершается, то тогда мы бы могли средствами PA доказать, что такая функция вот для этого конкретного числа не завершается?

>> No.5035041  
Файл: cirnokiss.jpg -(9 KB, 236x173, cirnokiss.jpg)
9

>>5035036
Ну, довольно сложно отвечать на вопрос "что было бы если бы это было бы не так, как в природе есть на самом деле", как я уже говорил, мы знаем что теорема Гудстейна истина, то есть на любом стандартном натуральном числе не зацикливается. Априори, если мы не знаем ничего о "внутреннем устройстве" функции Гудстейна - не факт, арифметика Пеано не на все вопросы типа "завершается ли программа М" умеет отвечать (потому что тогда мы бы проблему останова решать умели).

>> No.5035048  

>>5035041

>Ну, довольно сложно отвечать на вопрос "что было бы если бы это было бы не так, как в природе есть на самом деле", как я уже говорил, мы знаем что теорема Гудстейна истина, то есть на любом стандартном натуральном числе не зацикливается.

Я не понимаю почему мы это знаем. Мы ж для этого должны поверить в какую-то особую индукцию, а что если такая индукция неправильна, и на самом-то деле, если написать компьютерную программу, которая будет пытаться полным перебором найти такое число, для которого функция гудстейна будет зациклена, то она это число найдет?

>> No.5035052  
Файл: cirnocihiron.gif -(590 KB, 640x640, cirnocihiron.gif)
590

>>5035048
А что если обычная индукция неправильна, и мы когда нибудь найдём пару чисел a,b такую, что a+b не равно b+a? Окажется тогда, что мы в ещё более удивительном мире живём, чем думали в 20ом веке, и тут нужно будет радоваться, а не огорчаться. Я так думаю!

>> No.5035093  
Файл: cirnonya.jpg -(12 KB, 192x192, cirnonya.jpg)
12

>>5035036
А, ну кстати, совсем вылетело из головы, арифметика Пеано может доказать для любого конкретного стандартного числа, что Goodstein(n) не зацикливается, то есть она может доказать что Goodstein(1) не зацикливается, Goodstein(2) не зацикливаетс, ..., Goodstein(10^100) не зацикливается и тд Индукция до \eps_0 нужна только за тем, чтобы доказать \forall x (f(x) не зацикливается), то есть чтобы "универсализировать" все эти утверждения в одну кучу, поэтому, отвечая на твой вопрос: такого быть не может, арифметика Пеано (без всякой индукции до eps_0) доказывает что "Goodstein(872907648972356) не зацикливается"

>> No.5035095  

>>5035093

>арифметика Пеано может доказать для любого конкретного стандартного числа, что Goodstein(n) не зацикливается, то есть она может доказать что Goodstein(1) не зацикливается, Goodstein(2) не зацикливаетс, ..., Goodstein(10^100) не зацикливается и тд

Каким образом? Вычислением? Ну так это и для теоремы Ферма верно. Теорема утверждает: для для любого натурального числа n>2 уравнение a^n+b^n=c^n не имеет решений в целых ненулевых числах a,b,c. Каким-то образом (как диагональный метод Кантора) мы можем начать перебирать для всех n и для всех a,b,c натуральные числа подряд, и всегда будет оказываться что такое равенство не выполняется, но... а как мы поймем, что оно всегда не выполняется? Мы ж не можем за конечное время перебрать бесконечное число вариантов.

>> No.5035097  
Файл: cirno kiss 5.jpg -(508 KB, 1200x900, cirno kiss 5.jpg)
508

>>5035095
Не, там суть в том, что чтобы показать для какого-то конкретного числа n_0, что последовательность Гудстейна завершается, достаточно индукции до ординала который строго меньше чем \eps_0, а именно, до того ординала, который получится если в 2-ичном разложении числа n_0 (и 2-ичном разложении всех показателей и т.д.) заменить 2 на \omega, а индукцию до любого ординала меньше чем \eps_0 арифметика Пеано умеет делать "из коробки".

>> No.5035106  

>>5034794
Если ZFC противоречива, то в ней из любого утверждения следует любое утверждение, в частности из Con(ZFC) следует не-Con(ZFC). В рассматриваемом примере Con(ZFC) - не тотальное метаутверждение об объективной реальности, а просто строка символов, являющаяся ППФ в ZFC.

>> No.5035109  
Файл: cirnowedding.jpg -(53 KB, 640x850, cirnowedding.jpg)
53

>>5035106
Мне кажется ты немного нечестно смешиваешь метатеорию и объектную теорию. Говорилось об утверждении "(ZFC |- Con(ZFC) -> \not ZFC |- Con(ZFC)) -> \not ZFC |- Con(ZFC)" , (если из того что ZFC доказывает консис следует что ZFC не доказывает консис, то ZFC не доказывает консис) то есть об утверждении метатеоретическом (по отношению к ZFC), в качестве метатеории достаточно взять какую-то очень слабую арифметику, типа PRA, и тогда, по модулю всего этого, твоё замечание редуцируется до следующего: если в качестве метатеории (для рассуждений о доказуемости чего-нибудь в чём-нибудь) взять противоречивую теорию, то там можно вывести что-угодно. Это конечно правда, но называть это "зависит от определений” это как-то чересчур по-моему, с таким подходом всё что угодно зависит от определений, в том числе и вопросы в духе “верно ли что 0=0”, скажем.

>> No.5035439  

>>5035109

>всё что угодно зависит от определений

Именно так. Скажем, 0=0 верно в арифметике Пеано. Но если рассмотреть язык программирования с возможностью приведения типов и под 0 понимать все штуки, которые приводятся к 0, то легко могут возникнуть ситуации 0 != 0.

Перед использованием терминов всегда нужно вводить определения.

>> No.5035440  
Файл: cirnocixirno.jpg -(30 KB, 300x359, cirnocixirno.jpg)
30

>>5035439
Буду иметь в виду!

>> No.5035676  

>>5035440

Топ самых моральных и самых аморальных идей и результатов в математике?

Вот мой: (с 1-ым смирился, со 2-ым нет, 3-е и 4-е хорошо)

Бесконечность обезьян + Вавилонская библиотека (библиотека > обезьяны). Эти штуки исчерпывают всё, что может сделать человек.

И библиотеку не могли бы остановить божественные силы (или силы (не)удачи), но она слишком большая и я забил...

https://en.wikipedia.org/wiki/P_versus_NP_problem#P_=_NP
P = NP (идея)
Я был шокирован когда узнал что за этим стоит не только падение шифров (и то что об этом не всегда говорят в первую очередь), ведь такое превращает P = NP в одну из самых мерзостных и отвратительных штук...

Оказывается, это обессмысливает не то интеллект не то все мысли математиков (знаю, что есть пара нюансов!)

То есть это как библиотека Борхеса, только "карманная"...

И к сожалению слишком мало написано о моральной стороне вопроса, как верящие в P = NP оправдывают или рефлексируют свою веру (знаю, кто-то считает что док-во будет бессмысленным на практике, если будет) - как относятся к собственной вере и её перспективам (мне такое интересно!)

Детерменизм. (частично это математический результат?)

"Побеждён" самой реальностью. Будь он не побеждён можно было бы верить в божественные силы всё равно (ведь демонов Лапласа нет), а с квантами совсем "раздолье" фантазии началось (хотя физики скажут что раздолья нет и кто-то будет ещё бороться за супер-детерменизм)

Теорема Гёделя (Силы Света)

Позволяет немного ослабить тиски строгих математических законов, это как победа над детерменизмом.

>> No.5035698  
Файл: slide-14.jpg -(114 KB, 1024x767, slide-14.jpg)
114

>>5035052

>А что если обычная индукция неправильна, и мы когда нибудь найдём пару чисел a,b такую, что a+b не равно b+a? Окажется тогда, что мы в ещё более удивительном мире живём, чем думали в 20ом веке

Я думаю что если из такого-то набора формальных правил следует что-то неожиданное, то это еще никак не характеризует тот мир, в котором мы живем. Может быть что сами формальные правила неправильны.
И вообще, в физике нет никаких чисел самих по себе, числа непременно что-то характеризуют, например они могут характеризовать расстояния, и если допустим окажется, что наша вселенная это огромный куб с периодическими граничными условиями, то сложение расстояний будет действовать не так, как в арифметике.

>> No.5035701  
Файл: cirnosmall.jpg -(9 KB, 236x236, cirnosmall.jpg)
9

>>5035676
Мне нравятся результаты где есть элемент "неожиданности", мне они кажутся довольно "моральными", потому что указывают на место, где есть нетривиальная математика. Теоремы Гёделя, программа Ленглендса, монструоз муншайн, зеркальная симметрия, теорема Атьи-Зингера об индексе, словарь Мазура-Моришиты, теорема Гротендика-Римана-Роха, "аффинизация" производных категорий с оснащением, существование экзотических гладких структур, периодичность Ботта, интерпретация стабильных гом.групп как экстраординарных теорий гомологий, теория Громова-Виттена и тд

Из "неприятных моментов" могу вспомнить только всякую "модельную зависимость", скажем, контроль унивёрсумов при построении теории категорий или гомотопической теории типов, невозможность "модельно-независимым" образом определить гомотопический тип, невозможность "модельно-независимым" образом определить многообразие, и такое вот всё. То есть любая работа в координатах она как-то вызывает эстетическое отторжение. Ещё не нравятся аргументы где происходит "магическое сокращение" каких-то членов, потому что их никак кроме "попробовать выписать выкладку - и произойдёт магическое сокращение" не запомнить.

Вопрос хороший задал, а ответил я на него не очень, мне кажется, ну как есть пусть будет.

>> No.5035707  
Файл: cirnokk.jpeg -(6 KB, 300x168, cirnokk.jpeg)
6

>>5035698
Не знаю, это какая-то феноменология, не очень ко мне близкая. Для меня алгоритм вычисляющий какую-то вычислимую функцию на натуральных числах - это нечто супер-конкретное, намного более конкретное чем, скажем, понятие "температуры термодинамической системы". И индукция для меня "достовернее" всех записанных физических законов, потому что индукция самоочевидна, а физические законы - это попытка описать какие-то повторяющиеся видимые явления в каком-то формализме (и зачастую описания оказываются плохими при каких-то экстремальных параметрах описываемой системы).

>> No.5035720  

>>5035707

>Для меня алгоритм вычисляющий какую-то вычислимую функцию на натуральных числах - это нечто супер-конкретное, намного более конкретное чем, скажем, понятие "температуры термодинамической системы".

По-поводу алгоритмов - если у нас есть алгоритм, вычисляющий функцию Аккермана от чего-то очень-очень большого, ну скажем A(1000000000^1000000000, 1000000000^1000000000) то такой алгоритм определенно когда-нибудь завершится, только вопрос в том, будет ли возможно в реальном мире произвести такое вычисление (хватит ли ресурсов чтобы это посчитать), и где вообще хранить результат? Хоть это число чисто математически вполне вычислимо, на практике мы его скорее всего никак никогда не посчитаем, да и не очень надо.

>И индукция для меня "достовернее" всех записанных физических законов, потому что индукция самоочевидна

Не надо путать достоверность в плане "выводимости чего-то из таких-то аксиом/лемм/теорем" и достоверность в экспериментальной физике/химии.

Нет, индукция не самоочевидна. Индукция достоверна только в тех формальных системах, где она или является аксиомой, или выводима из каких-то там метатеорий.

И в общем случае в физической реальности она не выполняется. Скажем, если мы возьмем два мешочка с камешками, в одном будет 4 камешек, в другом 6, мы их пересыпаем из этих мешочков в третий, ну вот тут можно написать 4 + 6 = 10 если мы хотип описать это через арифметику. Но в общем случае это не работает. Если мы возьмем очень много камушков и будем их пересыпать, то вполне возможно что у нас там черная дыра образуется от такой большой массы этих самых камушков, и никаких камушков, никакой индукции по сложению уже не будет!

>> No.5035722  
Файл: cirnomarisa.jpeg -(14 KB, 189x267, cirnomarisa.jpeg)
14

>>5035720
Я и не говорил о достоверности "в плане "выводимости чего-то из таких-то аксиом/лемм/теорем"", мне кажется это довольно сложно вычитать из моего предыдущего сообщения, даже если делать это намеренно. Для меня индукция "более достоверна" в том плане, что я в неё больше верю, то есть для меня убедительнее тезис о том, что применяя индукцию я не приду к противоречию (не обязательно в формальной системе, математика появилась раньше чем попытки её формализовать, и индукцию использовали до появления всяких формальных систем, и после них тоже, не обращая на них внимания) чем о том, что скорость света постоянна в любой инерциальной системе отсчёта. Чтобы работать с числом A(100^100^100) и считать его легитимным объектом не нужно выписывать его десятичную запись полностью, точно так же как химикам не нужно выписывать пространственное положение и импульс каждой молекулы, чтобы делать заключения о взаимодействии хим.реагентов. Для меня пример с камушками демонстрирует, что камушки плохая вычислительная модель для арифметики, когда камушков много (не наоборот). Вообще всё это максимально неинтересно конечно, потому что это суть трансляция каких-то внутренних полурелигиозных переживаний в слова, а я такой человек, что мне ничьи переживания не интересны, кроме своих (извините).

>> No.5035724  

>>5035722

>Для меня индукция "более достоверна" в том плане, что я в неё больше верю, то есть для меня убедительнее тезис о том, что применяя индукцию я не приду к противоречию (не обязательно в формальной системе, математика появилась раньше чем попытки её формализовать, и индукцию использовали до появления всяких формальных систем, и после них тоже, не обращая на них внимания) чем о том, что скорость света постоянна в любой инерциальной системе отсчёта.

По-моему индукция это не то, о чем имеет смысл говорить "верю - не верю". Если она есть в какой-то теории, формальной системе или чем-то там еще теоретическом - значит она есть в этих рамках. Если нет - значит нет. Зачем тут вообще говорить о какой-то там вере?

Вот про "скорость света постоянна в любой инерциальной системе отсчёта" - да, если мы это утверждаем про физическую реальность - это вера. А если мы говорим о некоторой формализованной физической теории, где это утверждение является безусловной истиной - о какой вообще вере в этих рамках может идти речь, если это так дано по определению? Вот в саму физическую теори можно верить или не верить, можно верить в ее границы применимости для предсказания реальных физических процессов и так далее.

>> No.5035725  
Файл: ciqarno.jpeg -(8 KB, 186x271, ciqarno.jpeg)
8

>>5035724
Для меня индукция - это нечто что имеет смысл за рамками формальных систем: наоборот, формальные системы описываются, опираясь на наше понимание концепции конечных строк над конечным алфавитом и индукции (индуктивного описания синтаксиса формальной системы).

>> No.5035738  
Файл: 1579081090527.jpg -(64 KB, 502x568, 1579081090527.jpg)
64

>>5035725
Тут есть два обычных замечания.

Во-первых, естественной нотацией для формальных теорий являются деревья, а вовсе не строки; со строками приходится вводить всякие ограничения на вид используемых строк вроде скобок/польской записи и доказывать какое-то количество теорем об однозначности разбора. У деревьев нужные свойства есть автоматически.

Во-вторых, для формальных теорий нужны не просто строки, а строки с правилами комбинирования (подстановка, конкатенация, декомпозиция), и получающееся нечто оказывается очень сложным объектом. Например, бывает так, что даже про тексты конечной длины нельзя понять, могут ли они быть получены в данной формальной теории по правилам теории или же являются трансцендентным. Не понимаю, почему арифметика с субстратом из строк кому-то кажется более простым объектом, чем арифметика сама по себе.

>> No.5035742  
Файл: cirnolove.jpg -(333 KB, 600x685, cirnolove.jpg)
333

>>5035738
Всё так!

>> No.5036029  
Файл: 2020-04-02 02.54.32.jpg -(40 KB, 450x529, 2020-04-02 02.54.32.jpg)
40

Бамп мемами!

>> No.5036031  

>>5036029
Как раз у алгебраических геометров друзей много. Очень социальные ребята.

>> No.5036033  
Файл: cirnoxi.jpg -(46 KB, 300x615, cirnoxi.jpg)
46

>>5036031
Из этого бы следовало что я не алгебраический геометр, что очень хорошо, конечно, потому что тэг не самый приятный!

>> No.5036055  

>>5035701

> Вопрос хороший задал, а ответил я на него не очень, мне кажется, ну как есть пусть будет.

Нет, для меня всё хорошо!

Во втором абзаце есть совсем удивительно понятные места, первый можно разбирать на цитаты и загугливать.

>> No.5036131  
Файл: 15543654639341.jpg -(61 KB, 773x1094, 15543654639341.jpg)
61

1) Хочу посчитать знакопеременную сумму в n-й строке треугольника Паскаля комбинаторно. Ставлю вопрос сколько нечётно-элементных множеств можно собрать из n элементов. Получается, что (0|n)+(1|n)+...+(n|n)=2*( (1|n) + (3|n) + ...). Почему?

2) Сколько сюръекций из n элементного множества на k-элементное? Разобьём первое множество N на блоки по k элементов. Ответом на вопрос будет количество этих различных блоков. Если же поставим вопрос сколько сюръекций в неразличимое множество, то там блоки эквивалентные, если отличаются перестановкой индексов, то есть будут числа Стирлинга второго рода {n|k}. Но почему в первом случае вариантов больше в n! ?

>> No.5036316  
Файл: cirnokki.jpg -(70 KB, 728x1028, cirnokki.jpg)
70

>>5036131
1) Проще всего раскрыть по биному (1-1)^n
2) Нужно разбить {1,...,n} на k (упорядоченных) непустых подмножеств, а не на блоки по k элементов. В первом случае вариантов больше в k!, потому что для каждому неупорядоченному разбиению на k множеств соответствует k! неупорядоченных (просто напишем какой-то уникальный номер 1..k над каждым множеством, кол-во способов это сделать k!).

>> No.5037090  

>>5035725

Нашёл про программу Ленглендса:

https://habr.com/ru/company/piter/blog/402273

Мне интересен был ваш диалог про миры!

И идея что физика вообще не имеет отношения к нашему миру, что мир это именно математика. (понятна мотивация такого решения)

думаю, может быть такое (не обязательное!) примирение: и математика и физика даёт нам новую информацию о ""мире"", рождая новые мемы и новые возможности показывая.

both ways a equally correct

Может быть можно сравнить физическое знание с математическим если не требовать от него полного описания реальности = типа суть не в достижении "истины", а в том насколько большую "ложь" можно вместить в рамки нашего мира. Типа какой-то особо извращённый/специфический жанр математики (и может зависимый от времени), с т.з. которого популярные теории и возможности это достижения.

>> No.5037434  

>>5036316

У меня вопрос про симметрии: (Интересно и вообще пробел в том, что слышал!)

> Оказывается, группа может обладать представлениями разной размерности. Вот почему элементарные частицы можно сгруппировать в семейства из восьми или десяти частиц: у группы SU(3) есть как 8-мерные, так и 10-мерные представления. Восемь частиц в каждом октете из определенных Гелл-Маном и Неэманом (как тот, что изображен на рис. 1.4) взаимно однозначно соответствуют восьми осям координат 8-мерного пространства, являющегося представлением группы SU(3). Аналогично и с декуплетами частиц. (В то же время частицы невозможно объединить в семейства из, скажем, семи или одиннадцати штук — математики доказали, что у группы SU(3) нет 7- и 11-мерных представлений.)
> Поначалу это считалось всего лишь удобным способом группировки частиц, обладающих схожими свойствами. Однако ГеллМан пошел дальше: он объявил о существовании веских причин, обосновывающих такую схему классификации

Как связаны размерности и семейства/количества частиц?

Эти группы симметрий вообще, насколько они объяснены, каков их статус, какова фундаментальность этой идеи? (требуют ли они, по субъективной оценке, какого-то ещё фундаментального принципа или закона?)

Помню, что есть теории, где эти группы - часть ещё большей группы!

И как со всем этим связан спин?

>>5037090 Или может иерархия миров и мы на разных уровнях узнаём о нашем мире что-то новое. И бывает какие-то "якори" нового тянутся с уровня на уровень (например, новая математика даёт новую физику)

>> No.5037436  
Файл: 1460918300915.jpg -(246 KB, 1005x983, 1460918300915.jpg)
246

ЫЧАН, МНЕ СРОЧНО!
Необходимо запитать некий прибор, потребляющий 12V 2.7A, а под рукой только адаптер на 1,5A. Если я рискну, и попробую всё же подключить его, то сможет ли этот блок запитать его? И сколько времени у меня будет в запасе, прежде чем он взорвётся?

>> No.5037437  

Нет, не сможешь. Очень грубо выражаясь, амперы это как раз и есть то, что потребляют электроприборы. Надеюсь хотя бы вольтаж у тебя совпадает.

>> No.5037438  

>>5037437
Вольтаж-то совпадает, это амперов не хватает.

>> No.5037439  

>>5037438
Ну может в урезанном функционале и сможет работать. Всё зависит от прибора. Некоторые очень простые или напротив очень сложные смогут.

>> No.5037440  

>>5037439
Монитор.
Ладно, сейчас литанию прочитаю и попытаюсь пробудить духов машины.

>> No.5037441  

>>5037436
Смотря что за прибор, какой пусковой ток. Обычно на этикетке всегда завышают. Мб запустишь. Но лучше не рискуй, спалишь бп. А если бп дряной, то и прибору не поздоровится.

>> No.5037445  
Файл: 1556785703103.jpg -(86 KB, 811x447, 1556785703103.jpg)
86

>>5037441
>>5037439
В общем:
Мои молитвы были услышаны, блок питания превозмогал, но таки смог запитать монитор. Всё с ним хорошо, он всё таки рабочий.
Теперь осталось самое тяжёлое — придумать, как подключить этот монитор в добавок к уже имеющемуся, и где надыбать к нему дешёвые кабели.

>> No.5037446  

>>5037445
Ты только на постоянку использовать не вздумай. Посмотри в том же онлайтрейде, там много универсальных бп. И кабели есть. Ну или на озоне.

>> No.5037450  
Файл: 1551521172746.png -(425 KB, 700x769, 1551521172746.png)
425

>>5037446

> Посмотри в том же онлайтрейде, там много универсальных бп. И кабели есть

На барахолках буду искать. Иначе получается дурацкая ситуация — все эти дополнительные части обходятся значительно дороже, чем сами мониторы.

>> No.5037452  

>>5037445
Яркость сразу на минимальную поставь. А то о стабильной работе можно будет забыть.

>> No.5037454  
Файл: 1527667049131839944.jpg -(501 KB, 1076x1078, 1527667049131839944.jpg)
501

>>5037436
>>5037445
>>5037450
До сих пор помню как мой адаптер времён осады Терры медленно и верно подгорал, пока монитору совсем конец не настал. Причём воняло гарью постоянно, а я как-то не обращал внимания. А это адаптер горел. Дух машины, видимо, его покинул, или скоррапчен чем-то. Искал на барахолке замену, но чот стало лень и купил новенький монитор.

>> No.5037455  

>>5037450
Ну или так. Кабеля понятно, но бп это расходник, с возрастом первый дохнет. Проверяй перед покупкой. Мб от ноута какого найдешь за бесценок.

>> No.5037662  

Хотел научиться перемножать подстановки разной длины.
Наткнулся на видео https://youtu.be/4obT76a2l6c , где рассказывается, что этот объект связан с косами. Мол, если графически нарисовать связи между числами откуда и куда они идут, а потом нанести параллельные линии в очень малом масштабе, то они будут пересекать их в одной точке. Какая связь с перестановками не понял.

Кстати, как подстановки разной длины перемножать так и не понял. Например, если у меня есть (143) и (14) или как вообще такое посчитать (34251)(718)(275)(536).
Есть какой-то алгоритм?
Алсо подкиньте каких-нибудь задачек на подумать, связанных с перестановками.

>> No.5037950  

>>5035725

>наоборот, формальные системы описываются, опираясь на наше понимание концепции конечных строк над конечным алфавитом и индукции (индуктивного описания синтаксиса формальной системы).

Не понимаю. Каким образом и где конкретно в формальных системах присутствует индукция, кроме как в аксиоматике?

Вообще, надо тут разобраться. Что есть индукция? Вот возьмем допустим uninterpreted function. Индукция это если нечто справедливо для f(n) (где n - совершенно неважно что) и если верно, что f(next(n)) == f(n), то тогда f(n) == f(next(n)) == f(next(next(n))) == ...

Тут нужен квантор всеобщности, или что-то такое?

>> No.5037980  

я даже не знаю что такое перпендикуляр

>> No.5037982  
Файл: cixxirno.jpeg -(8 KB, 262x192, cixxirno.jpeg)
8

>>5037434
Вопрос хороший, отвечу я на него плохо потому что не физик. Идея идёт из интуиции, которая идёт из обсуждаемой выше теоремы Нётер, но только из квантовой её версии. По нашим представлениям фундаментальные частицы - это такие квантовые системы которые должны иметь параметры, скажем, массу или спин, которые не меняются при переходе к другим координатам. "Переход к другим координатам" формализуется как действие группы симметрий G (называемой группой пуанкаре) на пространстве состояний H. Идея Вингера состояла в следующем - а давайте попробуем найти все квантовые системы, у которых есть две величины М и S, масса и спин, не меняющихся при действиях группы Пуанкаре. Вигнер заметил, что на линейно-алгебраическом языке эти требования означают что такие представления должны быть неприводимыми. Как-то так. Соответственно фундаментальные предположения состоят в том, что линейно-алгебраический язык фон Неймана для описания квантовых систем вообще адекватный. Ну это естественное предположение, потому что другого языка у нас нет (ну, почти).

Про миры не понял, но физику как особую традицию математики (со своими понятиями о важном/неважно и строгом/нестрогом) я примерно и воспринимаю, по крайней мере теоретическую её часть, хотя сами физики были бы против такой трактовки.

>>5037662
Это нотации для циклов, имеется в виду что все перестановки одной и той же длины, но перемножаются циклы, можно, например, перевести нотацию циклов в обычную нотацию и посчитать. (275) значит что в перестановке 2 переходит в 7, 7 в 5, а 5 в 2.
Попробуй тут решить https://ium.mccme.ru/postscript/f19/algebra1-Seminar_7_Normal.pdf листки 7 8 9 все задачи связанные с перестановками.
>>5037950
Ну когда ты определяешь что такое терм, что такое формула, что такое корректная подстановка, что такое корректный вывод по правилам вывода, вот это всё, ты везде используешь индуктивные определения.

Индукция - это общая философская идея, так же как и "натуральные числа", в разных формальных системах она будет иметь разную имплементацию. Тот синтаксис который ты записал (если его совсем чуть-чуть грамотнее записать, но идею оставить ту же) называется "арифметика Персбургера". Там индукция бескванторная, в I\Pi_n, I\Sigma_n, PA арифметиках индукция с кванторами, в арифметиках высших порядков и теориях множеств индукция в некотором смысле ещё более сильная.

Вообще ты в некотором смысле тоже прав, мы просто из разных мест говорим, есть "индукция" как философская идея математического унивёрсума, а есть "индукция" как имплементация этой идеи в той или иной формальной системе. Почитай кстати вот эту заметку Успенского https://scisne.net/a-1600 мне кажется она несколько прояснит ситуацию. Там не про индукцию, а про натуральные числа, но сущностно это одно и то же.

>> No.5038108  

Реквестирую матрицу-тян. Нигде не могу найти.

>> No.5038511  
Файл: rusalochka.jpg -(128 KB, 850x1009, rusalochka.jpg)
128

Прошу прощения за вопрос не в тему, но эта русалочка на картинке рядом с Сырной - тоже какая-то тоходевочка?

>> No.5038514  
Файл: 05a77ca3582c5ecba031b841e16c2fec.jpg -(1133 KB, 1600x865, 05a77ca3582c5ecba031b841e16c2fec.jpg)
1133

>>5038511
Да, Вакасагихиме это.
https://ru.touhouwiki.net/wiki/%D0%92%D0%B0%D0%BA%D0%B0%D1%81%D0%B0%D0%B3%D0%B8%D1%85%D0%B8%D0%BC%D0%B5

>> No.5038588  

>>5037982

> Про миры не понял, но физику как особую традицию математики (со своими понятиями о важном/неважно и строгом/нестрогом) я примерно и воспринимаю, по крайней мере теоретическую её часть, хотя сами физики были бы против такой трактовки.

Хотел сказать, что эти разные традиции образуют разные "скопления мемов", и эти разные скопления являются разными мирами, о каждом из которых можно узнавать новое.

Можно завтра узнать что индукция не работает. Можно узнать какую-то новую физическую теорию. Можно узнать важное событие из произведения. А можно узнать что на балконе завелись муравьи.

И "иерархия" этих миров может меняться, может меняться их "распределение" между друг другом (может муравьи окажутся важнее физического знания или окажутся неразрывно переплетены с математикой).

Само понятия "факта" может отличаться в этих мирах:

В Математике факт вечен, а в физике "факты" это в каком-то смысле (если преувеличивать!) что-то типа хроники песен которые когда-то занимали первое место в чарте...

Но это не важно и вообще ни про что, прости!

>> No.5039503  

Ычан, Ычан, у меня что-то совсем на уровне младшей школы возник вопрос.
Смотри, есть выражение a - k = 2a - p. Очевидно, что k = a - (2a - p).
Но как правильно довести выражение до такого вида, если рассматривать его как уравнение и перемещать члены в разные части, меняя их знак на противоположный?

>> No.5041682  

>>5040755

Кстати, про дуальность есть у постнауки немного:

https://www.youtube.com/watch?v=m92fm6qJ5q4&list=PLh6dVTO7f4FYZtR6RnZU5xaYuS1-f1feE&index=10

> В некотором смысле исследование и поиск исключительных объектов - это логически противоположная деятельность построению общих вселенски глобальных теорий.

Из-за каких-то других мыслей не подумал об этом!

А как обычно выглядит процесс работы математика, процесс думания над чем-то или над доказательством? У шахматиста, наверное, так:

1) Можно посчитать какие-то варианты. Ствол (1 длинный вариант), куст (много мелких) и т.д. ...
2) Можно представить позицию или идею, а потом подумать как реализовать её.
3) Можно предсказать какие вещи будут важны в конце игры (или просто позже).
4) Можно что-то понять о самой динамике хода игры, наверное. У профи и "понтующихся" шахматистов даже много терминов про это: слишком медленно, слишком смело, слишком пассивно... и т.д.
5) Искусство оценивания позиции разное есть.
6) Можно просто маяться дурью играя, ожидая когда возможности появятся.

А как у математика?

>> No.5044080  

>>5035722

> потому что это суть трансляция каких-то внутренних полурелигиозных переживаний в слова

По-меому ваш диалог отличался от чего-то типа "Реальна ли математика?" и даже "Что есть индукция" (к чему всё перешло) и поэтому был оригинален.

Вроде началось с того что вы просто (или принципиально) две разные вещи называли "реальным миром". Но у меня остались ещё неясности:

Новое в физике работает сразу на двух уровнях - новое даёт не только новую теорию, но и новые феномены (не связанные 100% с теорией). (Ты развенчал только первое, вроде)

Твоя точка зрения в общем не понятна/интересна —

в чём смысл твоей критики точности теорий? Критиковать точность = уважать реальность (в каком-тоо смысле), в то же время ты абсолютно её не уважаешь (как показалось)...

Чем является мир, если реальным миром является математика?

Какая вообще разница кто чему больше верит (это ты на эту тему перевёл - оппонент про это высказываться не хотел)? Я понимаю что связь можно провести, но это будет интересная/оригинальная связь. (или я просто отвык от философии и это так же старо как "пещера Платона"!)

И по твоим постам про "кодирование мат. структурами информации" могло сложиться совсем противоположное впечатление о твоих мнениях (что добавляет интерес)

Не знаю что меня задело, прости, может несправедливость к человеку или к теме.

Как понимаю, диалог должен был идти так: (как понял логику)

  • Если узнаем о бракованности индукции - ничего не узнаем о реальном мире.
  • Мы никогда ничего не узнаём о реальном мире, все наши теории убоги - поэтому я вообще не считаю реальный мир реальным.

А тема сути индукции - уже левая(?) 2 с чем-то недели мучает и не отпускает!

Просто и мир унизили и тему мгновенно закрыли как скучную = у меня случился шок (простите!)

>> No.5044081  

>>5044080

т.е. (фикс)
Твой оппонент: — Если узнаем о бракованности индукции - ничего не узнаем о реальном мире.
Ты: — Мы никогда ничего не узнаём о реальном мире, все наши теории убоги - поэтому я вообще не считаю реальный мир реальным.

Может вообще не понял вас обоих.

>> No.5044595  

Чем алгебраическая Геометрия отличается от алгебраической Топологии?

>> No.5044611  
Файл: cirnokiss97.jpg -(114 KB, 1100x800, cirnokiss97.jpg)
114

>>5041682
Я это никогда особо не продумывал, всего и не упомнишь. Всё что ты описал в принципе применимо и к думанию над задачей, иногда удачная метафора/аналогия/ассоциация может дать правильный вектор, это вообще самое важное, любое прозрение типа "на что-то оно отдалённо похоже" обычно оказывается очень полезным. Иногда просто вычитываешь где-то какой-то трюк/технику и понимаешь что если её совсем чуть-чуть модифицировать то она станет применима и в твоём случае (что в принципе редуцируется до предыдущего предложения), поэтому можно читать источники по смежным темам и надеяться что что-то вычитаешь. Можно ещё пытаться максимально обобщить задачу "правильным образом" и надеятся что хорошее обобщение подскажет какие-то новые интуиции. Можно наоборот разбирать частные случаи и надеяться на то же самое.
>>5044080
Ну уж извини, у меня просто эти discovered vs invented, формализм vs платонизм, FOL vs SOL так навязли в зубах что сил нету, просто уже по семисотому кругу одни и те же тезисы, а когда их ещё проговаривают с интонацией в духе "вообще-то всё на самом деле обстоит вот так вот, не путайся в таких простых вещах пожалуйста", то желание всё это обсуждать ещё раз дополнительно на 3 делится.

Обе позиции примерно одинаково легитимны (потому что это тот тип метафизических вопросов, где иначе быть не может). Если кто-то думает, что индукция invented, то пусть будет invented, я не против.
>>5044081
Я уже не помню, но по-моему я как раз говорил что бракованность индукции укажет как раз укажет что-то о реальном мире (и что-то невероятно интересное, на уровне относительности одновременности и замедлений времени Эйнштейна, если даже не выше).
>>5044595
Сеттингами, а алгебраической геометрии пространства "жёсткие" их сложно мять и гнуть даже задача о том вкладывается ли какая-то кривая в какую-то поверхность может быть очень сложной (скажем, чтобы доказать что есть хотя бы одна рациональная кривая на гладкой поверхности Фано нужно проделать нетривиальную работу). В топологии пространства очень мягкие, гомотопические типы - почти самые мягкие объекты в математике, с ними делать вообще что угодно можно. Поэтому разные стили, техники доказательств, системы метафор и иинтуиций. Но, конечно, многие важные алгебраические техники/конструкции используются и там и там и многие интуиции тянут из алг.геометрии в алг.топологию и наоборот.

>> No.5045209  
Файл: ellipse.png -(41 KB, 341x1337, ellipse.png)
41

А описаны ли такие пространства, в которых объекты могут увеличиваться в размерах, проходя через какие-то области?
https://youtu.be/M9gdAt91Wp4 типа как в таком короткометражном фильме?

Ну или вот я нарисовал, можно представить такое двухмерное пространство (на трехмерное тоже можно обобщить) и вот внутри там как снаружи, и если штука двигается к центру, то сзади к ней будет увеличенное ее же отображение приезжать и она как бы будет увеличиваться в этом зацикленном пространстве. И наоборот, если двигаться наружу, оно будет уменьшаться. Как это называется

>> No.5045255  

>>5045209

> А описаны ли такие пространства, в которых объекты могут увеличиваться в размерах, проходя через какие-то области?

Немного не так как у тебя в примере, но допустим у нас есть прямой отрезок. Длину отрезка будем считать как кратчайшее расстояние между его концами.

Далее, представим что отрезок живет на поверхности какого-то двухмерного объекта (например, вытянутой сферы). Т.к. у вытянутой сферы непостоянная кривизна, при перемещении отрезка, его длина (т.е. кратчайшее расстояние между его концов) будет меняться.

По такому принципу работают, например, детекторы гравитационных волн.

Есть еще такой интересный эффект -- длина окружности по геодезической орбите вокруг вращающейся черной дыры не равна 2*pi*r, а может быть чуть меньше, или больше (в зависимости в какую сторону крутиться).

>> No.5045267  

>>5045255
Ну а если говорить о конкретно том, что я нарисовал, как такое пространство можно описать? Какая у этого пространства размерность, кривизна, как его можно обозначить? Есть ли какие-то математические нотации для этого?

>> No.5045278  
Файл: cirnokiss57.jpeg -(7 KB, 259x194, cirnokiss57.jpeg)
7

>>5045209
>>5045267
Называется "пространство орбит R^n \ {0} по действию v -> x v где x>0 вещественное число" (в частности на твоём рисунке n=2 и x=1/2). Чисто топологически это тор конечно, о кривизне/метрике говорить сложно, потому что действие не сохраняет метрику на R^n, а поэтому метрика на пространстве орбит не индуцируется (только конформная структура). Вообще конструкция слишком естественная, поэтому подозреваю что что-то где-то о ней интересного написано должно быть, поищу как время будет.

>> No.5045678  
Файл: ellipse2.png -(17 KB, 255x255, ellipse2.png)
17

>>5045278
А где в твоём описании обозначено то, какой формы эти орбиты в пространстве? Ведь может быть не только квадратные кольца, но и например круглые.

Есть ли некое формальное стандартизированное описание каких-то таких хитровывернутых пространств?

>> No.5045699  
Файл: cirnostrong.png -(113 KB, 480x643, cirnostrong.png)
113

>>5045678
Орбиты в этом пространстве, это множества типа {x^n v | n \in Z}, то есть некие дискретные множества точек. Растяжение в x раз работает таким образом, что переводит квадраты в маленькие квадраты, а окружности в маленькие окружности, и вообще любой рисунок в тот же самый рисунок поменьше, поэтому не очень важно что ты "нарисуешь на фоне".

Моё описание довольно формально мне кажется, пространство орбит ассоциированное с действием группы на множестве - это очень формальная математическая конструкция, формальнее некуда.

Если это был вопрос к формализации твоей картинки не относящийся, то он довольно общий. Есть разные классификации/описания разных типов пространств, ну скажем, гипотеза о Геометризации (та самая которую доказал Перельман на районе) описывает, в некотором смысле, все "естественные трёхмерные геометрии" (а в некотором смысле ещё более естественных геометрий всего 3, которые все знают). Есть ещё классификация экзотических гладких структур на S^7 скажем, много вообще чего есть, и слишком широкий вопрос...

>> No.5045712  
Файл: Calabi-Yau.png -(327 KB, 840x840, Calabi-Yau.png)
327

>>5045699

>Орбиты в этом пространстве, это множества типа {x^n v | n \in Z}, то есть некие дискретные множества точек. Растяжение в x раз работает таким образом, что переводит квадраты в маленькие квадраты, а окружности в маленькие окружности, и вообще любой рисунок в тот же самый рисунок поменьше, поэтому не очень важно что ты "нарисуешь на фоне".

Да, похоже что так, тут это значение не имеет.

И что из себя представляют те непонятные маленькие пространства, которые в рамках теории струн/суперструн натеоретизировали? Там тоже какие-то хитрым образом зацикленные сами на себя пространства, или как это представить? Как вообще к такому пришли? И можешь понятным языком объяснить неспециалисту, что из себя представляют многообразия? Вот то описанное мной пространство с этими самоповторениями снаружи и внутри - это многообразие или нет? А проективная плоскость это многоообразие?

>> No.5045743  
Файл: cirnostrong2.jpeg -(9 KB, 224x224, cirnostrong2.jpeg)
9

>>5045712
Да, в некотором смысле там тоже некоторые зацикленные на себя пространства, которые лежат в 6 "ортогональных осях" к нашему пространству времени, просто они очень маленькие и их не видно (ну, по крайней мере это один из подходов, называемый КК-компактификацией). Придумали сложным образом, но главная мотивация в том что теория с дополнительными измерениями становится стройнее и симметричнее. Иногда эти симметрии подсказывают даже вычислительные инсайты для вычисления пертурбационных рядов, что полезно для классической физики высоких энергий. Какой-то физик из LHC сказал как-то что бозон Хиггса не был бы открыт без вычислительных инсайтов теории струн, потому что софт анализирующий данные не был бы достаточно точен (точно не знаю о каких конкретно инсайтах он говорил, но верить стоит, думаю).

Многообразие это пространство которое на "маленьких масштабах" возле каждой точки выглядит как обычное плоское пространство. Твоя конструкция на маленьких масштабах возле каждой точки выглядит как обычная плоскость (потому что на маленьких масштабах "повтор сзади" и "повтор спереди" не виден), поэтому это многообразия (более того, топологически это двумерный тор, о чём я говорил уже). Проективная плоскость это тоже многообразие. Есть консенсус что многообразия это наиболее естественный геометрический объект для изучения, потому что, во-первых, он обобщает все обычные геометрические фигуры которые мы знаем в школе: тор, куб, сфера, шар, конус, плоские фигуры, а во-вторых позволяет с ними работать "внутренним образом", а не координатно, то есть работать непосредственно со сферой, скажем, а не сферой вложенной в R^3, а когда можно работать с чем-то без координат то обычно выходит продуктивно. Ну как-то так.

Вообще чтобы понимать что такое многообразие не нужно быть специалистом, обычно о них на первом курсе узнают во втором семестре когда многомерный анализ проходят. Поэтому если интерес достаточно сильный, то мне кажется можно и в формальном определении разобораться!

>> No.5045858  

>>5045209 >>5045255 >>5045278

Вы просто супер-команда! Глаз и ум радуется. (Такого в треде ещё не было)

>>5044611

> Ну уж извини, у меня просто эти discovered vs invented, формализм vs платонизм, FOL vs SOL так навязли в зубах что сил нету, просто уже по семисотому кругу одни и те же тезисы

Прости, мне просто показалось диалог шёл или "мог бы пойти" по более интересному пути!

> Если кто-то думает, что индукция invented, то пусть будет invented, я не против.
> Я уже не помню, но по-моему я как раз говорил что бракованность индукции укажет как раз укажет что-то о реальном мире (и что-то невероятно интересное, на уровне относительности одновременности и замедлений времени Эйнштейна, если даже не выше).

А может оппонент вообще не хотел (изначально) как-то высказываться об индукции? Произошло своего рода недоразумение.

Просто тут три варианта: (и я сам в них запутался!)

  • Математика — единственный источник фактов о мире. Так что новый факт об индукции это тривиально новое знание о мире. (1)
  • Математика это буквально часть мира. Так что новый факт об индукции это тривиально новое знание о мире. (2)
  • Новый факт об индукции как-то нетривиально ведёт к новому знанию о мире. (3)(это можно совмещать с другими пунктами)

Если честно я не задумался совсем ни о (2), ни о (3)!

Может это конкретнее—интереснее тезисы чем discovered vs invented? (но не знаком с деталями этих философских тем которые ты назвал!)

>>5035724 >>5035722 >>5035720 >>5035707

Для неискушённого (меня) ваш диалог даже не напоминал об обычной теме "discovered vs invented" (или "реализм/платонизм") — скорее о чём-то более радикальном, о столкновении радикального "реализма" (против самых очевидных истин) и радикального "платонизма" (касающегося именно математики), например:

> И индукция для меня "достовернее" всех записанных физических законов, потому что индукция самоочевидна, а физические законы - это попытка описать какие-то повторяющиеся видимые явления в каком-то формализме (и зачастую описания оказываются плохими при каких-то экстремальных параметрах описываемой системы)

Звучало радикально/интересно, как пункт (1)!

Это радикальнее просто веры в не-открытость математических штук (хотя может быть прямо следует из этой веры).

Хотя сейчас вижу >>5035698 что началось именно с обвинения математики в формализме. (Но он к этому ещё добавил про "не только числа", не хотел именно на это полностью напирать...)

Чтобы внести вклад в тред по теме: какие у математических объектов бывают виды "сопротивления изучению"? Например, есть хаос и эффект бабочки. Есть невыводимость чего-то правдивого в обмен на "силу" теории. Есть нерешаемости какие-то. Есть комбинаторные взрывы.

>> No.5045860  

>>5045858

> и радикального "платонизма"

Хотя такое не требует платонизма! От этого ещё интереснее.

>> No.5046222  
Файл: 1587667836155.png -(23 KB, 904x592, 1587667836155.png)
23

Объясните как это работает

>> No.5046287  

>>5037982
А какие номеры из листков связаны с перестановками? А то вроде всё с группами.

>> No.5046350  
Файл: EQ-8jbLesA4.jpg -(65 KB, 640x494, EQ-8jbLesA4.jpg)
65

>>5040755
Что такое спорадические исключительные объекты?

>Исключительные объекты

Например?

>>5040810
Математика, наверное, сама по себе существует, как живая природа.

>>5044611
А пространства, они на ощупь как?

>> No.5046680  

>>5046350

Зачем ты спрашиваешь то что можно нагуглить или то что потребует от тебя знания такого уровня, какое исключило бы этот вопрос изначально? (попытка - не пытка, не критикую слишком)

>> No.5047162  
Файл: sample-84596f9afbd8e1bf4f48c4a2d7a5d5a3-(...).jpg -(76 KB, 333x806, sample-84596f9afbd8e1bf4f48c4a2d7a5d5a3-(...).jpg)
76

Что если бы вместо "морфизм" было бы "хеншин"? Не по Поливанову так же, как "суши", из-за устоявшегося употребления.

>> No.5047321  
Файл: cirnocirno.jpg -(186 KB, 1280x720, cirnocirno.jpg)
186

>>5045858
Хм, не думал интерпретировал тот разговор именно таким образом, может это и интереснее, возможно действительно моя позиция ближе к (1).

Формализм vs платонизм я немного по-другому понимаю, формализм это грубо говоря позиция что "синтаксис важнее семантики" и сопутствующие этому заявления, что все непротиворечивые теории одинаково равноправны, что всё что "невыразимо" каким-либо образом, того вообще нет и тд А платонизм - это что "семантика важнее синтаксиса", что натуральные числа (или, более радикально, унивёрсум фон Неймана) это какие-то означающие к которым наша интуиция непосредственно подключена, а всякие теории, скажем, ZFC, это непосредственно попытка ухватить эти интуиции каким-либо образом. Ну, конечно есть разные вариации/отступления, но в общем суть такова. В этом же разница между логикой первого и второго порядка, в логике первого порядка мы сначала строим синтаксис, а потом говорим что "семантика это вообще всё что угодно что подходит под синтаксис", а в логике второго порядка мы сначала строим семантику и говорим что "всё что следует семантически должно следовать и синтаксически" но синтаксис из такого подхода становится, мягко говоря, слегка неосязаемым: становятся возможны бесконечные выводы например математически строго это значит что перестаёт работать теорема о компактности, что вообще говоря несколько подрыввает сам смысл слова "синтаксис". Ну это всё такое.

Да, ты так-то почти все и описал, с наскоку могу выделить следующие группы:

препятствия "невыразимости": синтаксическая неполнота сильных теорий, алгоритмическая неразрешимость, неинтегрируемость динамических систем, невыразимость в радикалах корней, невыразимость интегралов в элементарных функциях

препятствия "дикости" (то что ты назвал "комбинаторным взрывом", думаю): дикость задачи о классификации двух матриц с точностью до сопряжения, дикость классификации конечных групп, конечных топологических пространств, 4-многообразий с точностью до гомеоморфизма и тд (Обычно в этом случае задачу сужают чтобы она стала хоть чуть-чуть более адеватной, например, "классифицировать простые конечные группы")

проблемы из-за оснований, невозможность определить "слишком большие объекты" без возни: сюрреальные числа, большие категории, универсальные конструкции по категориям любых размеров, унивёрсумы в теории типов, большие кардиналы и тд

отсутствие хороших свойств у системы, непозволяющих инвокнуть общие теоремы о них: некомпактность, хаотичность, неэргодичность, положительная характеристика, неинтегрируемость, присутствие кручения, неточность (функтора), неотделимость, негеометричность и тд и тд и тд
>>5046350
Кватернионы октавы, исключительные группы Ли, исключительные простые группы, исключительные решётки, исключительные гиперкэлеровые многообразия, исключительные изоморфизмы, исключительные геометрические конфигурации (платоновы/архимедовы тела, периодичные замощения, полуправильные многогранники и тд и тд), исключительные гладкие/комплексные/симплектические/кэлеровы/... структуры на маломерных многообразиях (сферах, скажем), исключительные модельные геометрии, исключительные внешние автоморфизмы (у S_6 скажем). Короче все конфигурации/структуры которые существуют в маленьких размерностях и перестают существовать в больших (не допускают многомерных обобщений).

Топологические - подвижные эластичные и плохо осязаемые, как мыльные пузыри, но не такие хрупкие, гладкие - металлические и гладкие, римановы - на ощупь как бумага и деформируются так же, алгебраические как "растягивающаяся шарнирная бумага".

>>5046287
7.1 7.8а 7.9 8.4аб 8.5 9.7абвг 10.2 абз

>>5047162
Было бы замечательно!

>> No.5047345  

>>5047321

>задачи о классификации двух матриц с точностью до сопряжения

Жорданова форма же, нет?

>> No.5047347  
Файл: cirnoiosys.jpg -(25 KB, 300x300, cirnoiosys.jpg)
25

>>5047345
Так это одной матрицы, привести подходящей сменой базиса к жордановой форме две матрицы одновременно нельзя, иначе мир был бы слишком простой. Есть устоявшийся консенсус что эта задача "дикая", то есть не поддаётся никакому разумному решению, и часто чтобы доказать что какая-то задача классификации каких-нибудь представлений тоже дикая, показывают что она как минимум не проще этой. Например она эквивалентна задачи классификации n матриц с точностью до сопряжения для любого фиксированного n.

>> No.5047868  

А давайте придумаем какую-нибудь непонятную комбинаторную задачу, и попробуем ее решить.

Например, есть вектор натуральных чисел (или массив, ну кому как удобнее) длины N и есть условие, что первый элемент вектора всегда равен 1, второй и все последующие элементы вектора должны быть больше либо равны сумме всех предыдущих элементов вектора, но меньше либо равны произведению всех предыдущих элементов вектора. Получается, первые два элемента такого вектора будут {1,1}, это без вариантов. Третий элемент вектора может быть произведением, т.е. 1 x 1 = 1, может быть суммой 1 + 1 = 2, т.е. два варианта {1,1,1} и {1,1,2}, а для четвертого элемента вектора, если отталкиваться от {1,1,1} то он может быть 1 2 3, а если отталкиваться от {1,1,2} то тогда 2 3 4 - думаю что понятно почему.

Так вот, а можно ли как-нибудь вывести общую формулу нахождения количества корректных вариантов для вектора длины N? Или оценить асимптику?

>> No.5047869  

>>5047868
Хотя не, немного не правильно сформулировал. В общем границы определяются тем, что там получается при произведении или суммировании предыдущих элементов. Если сумма меньше произведения, нижняя граница будет суммой, верхняя произведением. Иначе - наоборот.

>> No.5047875  

А что такое "анти три" из теории групп, о которой вот тут https://youtu.be/r36ApsGwbtk?t=4490 рассказывают? О чем это вообще?

>> No.5047892  
Файл: cirnocirno.jpg -(110 KB, 999x999, cirnocirno.jpg)
110

>>5047875
Это о вычислениях в кольце представлений SU(3), "три" и "антитри" это фундаментальные представления соответствующие корням старшего веса.

>> No.5048545  

>>5047892
А какие-нибудь симметрии в закономерности таблицы Менделеева есть? Есть какая-нибудь математика, которая б там что-то описывала, примерно как в случае с кварками?

>> No.5048573  

>>5048545
Ты не поверишь, но таблица Менделеева не уникальна. Существует множество моделей систематизирующих химические элементы, включая трёхмерные.

>> No.5048575  

>>5047869
Зачем начинать с 1 и усложнять, если можно начать с 2? И тогда первые три элемента определяются однозначно, 2,2,4, четвёртый от 8 до 16 и т.д.

>> No.5048579  
Файл: cirnocool.jpg -(74 KB, 460x952, cirnocool.jpg)
74

>>5048545
Я не физический химик но почти уверен что нет. Атомы слишком сложные штуки чтобы наедятся на то что какие-то алгебраические и комбинаторные методы придутся к месту. Волновая функция атома, которая туда-сюда полностью описывает все его нерелятивистские свойства (а релятивистские тоже супер важны даже в приложениях), в ситуации когда он больше ни с чем не взаимодействует, это решение фермионного уравнения Шрёдингера для n тел (электронов), когда n > 10 его решение становится почти нереальным даже численно. Мне кажется на атомы полезнее смотреть как на очень сложные объекты, о которых мы можем узнавать всё больше и больше информации задавая всё более тонкие и тонкие вопросы и применяя всё более сложные и сложные математические описания, которые бы служили адекватным вычислительным фреймворком для ответа на эти вопросы.

>> No.5048836  

Как связаны представимые функторы и сопряженные функторы?

>> No.5048891  

>>5047321

(1) В Физике ещё актуален вопрос можно ли сделать "самую идеальную самую супер-теорию описывающую самое дно", а в Математике он уже решён (по крайней мере для кого-то)? Интересно. (Или нет и тут всё понятно)

(2) Интересует ли тебя что-нибудь в психологии в математическом плане (если говорить абстрактно совсем/"мечтать")? Может математический факт о чём-то там - в конце концов этим мы воспринимаем математику
https://www.youtube.com/watch?v=rlI1KOo1gp4

Меня бы интересовало например такое: как можно держать целый мир в голове и всё равно деградировать (но это не математический вопрос! или не облачённый в математическую форму)

(3) Мог бы ты привести пример некоммутативности (на подобии квантовой) в реальном или выдуманном мире? Поясню вопрос:

Вроде понятно что какая-то операция может зависеть от порядка (и это не особо интересно)

Вроде понятны совсем бытовые примеры (и это тоже малоинтересно!)
https://en.wikipedia.org/wiki/Commutative_property#Noncommutative_operations_in_daily_life

А потом вдруг дело переходит к тому что ~"у частицы свойство А и свойство Б не могут быть определены одновременно (1) и порядок измерения А и Б повлияет на результат (2)" (и это уже очень интересно!)

Есть ли примеры подобного (даже выдуманные)? Имеют ли (1) и (2) одинаковое значение? (или есть ли ещё примеры где есть обе этих вещи)

Обязана ли такая некоммутативность быть связана со временем?

>> No.5048906  
Файл: cir.jpeg -(10 KB, 189x267, cir.jpeg)
10

>>5048891
1) Ну, для кого-то решён, для кого-то нет. Вообще он решён чисто с прагматической точки зрения для работающих математиков, что и запечатлено в знаменитой фразе Гильберта: "Никто не выгонит нас из рая, построенного для нас Кантором", - математики теперь всё кодируют в одном языке - ZFC (ну, по модулю всяких конструктивизмов, больших кардиналов, аксиомы выбора, континуум гипотезы и тд, но это всё терпимо, можно представить себе мир в котором всё было бы намного хуже) и могут без особой опаски чисто механически инвокать результаты из одной области в другой. Но так-то оттенков философских позиций по вопросу "математическая (или, хотя бы, теоретико-множественная) реальность одна или их много?" огромное количество: наивный V-платонизм (аргумент трансфинитной индукции), потенциализм, ультраформализм (\Pi_1 формализм), платонизм Вудина (программа внутренних моделей, программа Ultimate-L), платонизм Форефмана (программа обобщённых больших кардиналов), мультиверсализм Хамкинса/реализм второго порядка ("псевдоаксиомы" мультивёрса), полнокровный платонизм Балагура и тд и тд Все эти споры конечно очень древней грецией или классической немецкой отдают, где всерьез думали о том "а корень из 2/время/пространство/реальность существует или мы его придумали?", что может даже и неплохо.

2) Ну, мне наверное было бы интересно послушать какие эвристики для запоминания/придумывания того или иного сюжета люди используют, как выбирают на чём сфокусировать внимание, а что ерунда, ну что-то такое.

3) У Терренса Тао https://terrytao.wordpress.com/2007/02/26/quantum-mechanics-and-tomb-raider/ был забавный пост где он объяснял неравенства Белла через вымышленную компьютерную игру. Но как правильно подметили в комментариях: хоть конструкция и супер-остроумная, единственный её недостаток в том, что не видно при чём тут некоммутативность, которая лежит в самом сердце квантовой механики. Но это максимум что я мог вспомнить, вообще мне кажется что люди понимают квантовую механику чисто формально типа как "некоммутативизация классической" и никаких сильно хороших картинок и аналогий нету (хотя у физиков наверное своё мнение по этому поводу будет). (1) и (2) более-менее одинаковое значение имеет, по крайней мере теоретически. Про время не уверен что вопрос понял, но мне кажется что не обязана.
>>5048836
G имеет левый сопряжённый если Hom(X,G(?)) представим для всех X, и имеет правый сопряжённый если Hom(G(?),X) копредставим для всех Х.

>> No.5048915  

>>5048906

>математики теперь всё кодируют в одном языке - ZFC

Это не совсем верно. Никто никогда не начинает работать сразу с аксиоматического уровня. В пейперах основные понятия предметной области считаются известными и не вводятся с нуля. В обычных учебниках алгебры явно озвучивается только аксиома Цермело - потому что нужно как-то доказать теорему о вполнеупорядочивании и лемму Цорна. Типично zfc'шные аксиомы вроде регулярности или схемы подстановки практически не используются. Поэтому с тем же успехом можно считать, что все математики работают в NFU+Inf+выбор, или в любой другой теории множеств, способной выразить алгебру.

>> No.5048917  
Файл: ссс.jpg -(89 KB, 569x916, ссс.jpg)
89

>>5048915
Спасибо, буду теперь знать!

>> No.5048918  
Файл: 61Zt0gZmdtDXfaOFjF4DRQ.jpg -(26 KB, 640x544, 61Zt0gZmdtDXfaOFjF4DRQ.jpg)
26

>>5048917
Ну я серьёзно же. Редко вижу в статьях, чтобы оговаривалось, что работа ведётся внутри именно ZFC.

>> No.5048920  
Файл: cifirno.jpg -(46 KB, 600x847, cifirno.jpg)
46

>>5048918
Я просто не понимаю иногда зачем некоторые сообщения пишут. Ты думаешь я не читал/не писал статьи? Или ты думаешь что я читал, но не отрефлексировал то, что большая часть математики от конкретного кодирования в конкретных основаниях не зависит? (Что, кстати, отражено в цитируемом тобой куске в слове "кодируют", ведь закодировать можно только что-то, что от кодировки не зависит)? Или ты споришь с тем что ZFC де факто стандарт и когда тонкости связанные с основаниями важны, то в 99% случаев работают именно в ZFC? Меня просто некоторые замечания конфузят до ужаса, из-за того что их иллокутивный акт считать не могу, остаётся только замять побыстрее, а то мало ли чего, вдруг сбой в Матрице.

>> No.5048922  

>>5048920

>ты споришь с тем что ZFC де факто стандарт

Да, я спорю с этим. Не вижу причин считать, что ZFC стандарт. Её не используют как стандарт, её обычно не упоминают, а в местах, в которых важны основания, почти всегда используется не ZFC, а что-то другое. Непонятно, почему ты считаешь эту аксиоматику стандартом.

>> No.5048924  
Файл: cirnokicirno.jpg -(1465 KB, 2586x2139, cirnokicirno.jpg)
1465

>>5048922
Понял, ну так я вижу мир, и наверное это прекрасно что кто-то его по-другому видит. Но я думаю моя позиция более мейнстримная, на нлабе, например в статье https://ncatlab.org/nlab/show/ZFC написано: "The most commonly accepted standard foundation of mathematics ", - по истории правок можно заметить что ревизию этой статьи в какие-то моменты делали Майк Шульман и Урс Шребер, оба специалисты в основаниях математики и оба разрабатывают альтернативные подходы к основаниям, они эту строчку не удалили, так что смею предположить что они со мной согласны. Но спорить о том что там стандарт, а что нет мне максимально неинтересно конечно, так что могу признать что твоя точка зрения тоже легитимна и имеет право на жизнь.

>> No.5048926  

>>5048906

1) Прости, не пояснил! Хотя может вопрос глупый и тоже банальнейший. Имею в виду что для платониста (как понял) автоматически существуют вещи которые вообще могут быть не выразимы, плюс любой формализм для платониста это лишь нечто побочное (хотя это я уже мог придумать/приукрасить)...

Плюс мелькала заметка Успенского
(спасибо за названия разных течений!)

> (1) и (2) более-менее одинаковое значение имеет, по крайней мере теоретически. Про время не уверен что вопрос понял, но мне кажется что не обязана.

Просто я такое думал: допустим есть воздушный шарик. В одном месте у него есть цвет и нет веса, а в другом есть вес цвет и нет цвета... это связано с некоммутативностью (пусть и не квантовой)?

(Это для меня значило бы "связь с местом, а не со временем".)

Или как надо модифицировать пример?

Есть ещё игра "Braid", которая уже про время, там есть

"зелёные предметы" - они подвластны влиянию игрока, но не подвластны времени,

поэтому ты можешь что-то натворить а потом перемотать время НО эффекты от того что ты сделал останутся.
https://www.youtube.com/watch?v=pcjbil1TM0o

А есть мир где перемотка времени создаёт теневую реальность, вроде сам автор хотел чтобы это ассоциировалось с квантовой механикой (и может похоже на гробницы из поста Тао)
https://www.youtube.com/watch?v=EYCmacqcGWI

Есть ли в этих операциях со временем что-то некоммутативное (в любом из смыслов?)

Или вот вопрос назрел про один из бытовых примеров:
https://en.wikipedia.org/wiki/Commutative_property#Noncommutative_operations_in_daily_life

> The act of dressing is either commutative or non-commutative, depending on the items. Putting on underwear and normal clothing is noncommutative.

"Некоммутативность" может быть свойством лишь действий? может ли "некоммутативность" каким-то образом заключаться в самом факте что трусы на одежде это не то же что трусы под одеждой?

Обязательно всегда думать о каких-то действиях?

%%Понимаю что глупый вопрос, просто для размышления о коммутативности (как понял) надо сравнить последовательности операций АБ и БА (взяв кубик Рубика и пару поворотов, например), но не всегда бывает что обе операции одинаково интересны/известны.

Например, в паззле Braid есть операция "нажать зелёный рычаг и перемотать время" — а обратная ей неизвестна или слишком банальна, "перемотать время а потом нажать рычаг" это бессмыслица какая-то.%%

И что если после того как одел нормальную одежду трусы уже не надеть (реализуема лишь одна из операций)? Это всё ещё некоммутативность?

>> No.5048927  

>>5048926

Вот из Алисы ещё: (относится ли некоммутативность к такому?)

> — Совсем не одно и то же, — возразил Болванщик. — Так ты еще чего доброго скажешь, будто «Я вижу то, что ем» и «Я ем то, что вижу», — одно и то же!
> — Так ты еще скажешь, будто «Что имею, то люблю» и «Что люблю, то имею», — одно и то же! — подхватил Мартовский Заяц.
> — Так ты еще скажешь, — проговорила, не открывая глаз, Соня, — будто «Я дышу, пока сплю» и «Я сплю, пока дышу», — одно и то же!

И такое про пирог:

> — Ты не умеешь обращаться с Зазеркальными пирогами, — заметил Единорог. — Сначала раздай всем пирога, а потом разрежь его!

В обычном мире поменять так порядок действий вообще нельзя.

>> No.5049113  
Файл: ci.jpg -(314 KB, 1200x1867, ci.jpg)
314

>>5048926
Думаю о действиях скорее нужно думать, чем не нужно, хотя некоммутативность это свойство чисто формальное, т.е. о свойстве формальной операции на формальном множестве, все значимые примеры что мне в голову приходят - они так или иначе о действиях.

Те игры разве что очень отдалённо можно притянуть к формулировке квантовой механике через интеграл по траекториям: формализм в квантовой механике где нужно "суммировать по всем возможным мирам", я думаю это с некоммутативностью связано, но непрямо.

Обычно в математике страются строить сеттинг так, чтобы и А и Б были определены для всех состояний системы, но если уже произошло что не для всех, то считать ли А и Б некоммутирующими если АБ определено, а БА это более менее вопрос соглашений, но мне кажется что да, нужно считать некоммутирующими.

Рассказ Алисы думаю к некоммутативности относится прямым образом!

>> No.5049278  
Файл: ring_chain.png -(11 KB, 1088x968, ring_chain.png)
11

Как можно языком математики формально описать вот такую бесконечную цепочку зацепленных колец? У этой штуки есть очень интересное свойство, что если кольцо в цепи в каком-то месте разорвать или просто удалить, все кольца справа от него могут быть распутаны и цепь распадется с той стороны на кучу колец. Это интересно с той точки зрения, что можно было бы попробовать синтезировать такой полимер, и при малейшем повреждении он бы просто рассыпался на молекулы. В химии вообще открыли такие молекулы с топологической связью - катенаны.

>> No.5049280  

>>5049278
Ну и еще предлагаю подумать над тем, как можно сделать конструкцию, где при удалении одного кольца она б рассыпалась в обе стороны. У меня один вариант решения есть уже.

>> No.5049302  

>>5049278
Напоминает сферу Александера и ожерелье Антуана.

>> No.5049303  

А это ссылка на новый тред >>5049267

>> No.5049305  
Файл: ring_chain.png -(12 KB, 1088x968, ring_chain.png)
12

Хотя вообще та >>5049278 штука как раз должна распадаться в обе стороны при размыкании одного кольца. А вот эта на картинке - уже только в одну. Интересно, описывалось ли уже что-то подобное?




[d | an-b-bro-fr-gf-hr-l-m-maid-med-mi-mu-ne-o-old_o-p-ph-r-s-sci-sp-t-tran-tv-w-x | bg-vg | au-mo-tr | a-aa-abe-azu-c-dn-fi-hau-jp-ls-ma-me-rm-sos-tan-to-vn | misc-tenma-vndev | dev-stat]
[Burichan] [Futaba] [Gurochan] [Tomorrow] [Архив-Каталог-RSS] [Главная]